Advanced Organic Reactions Carmelo J. Rizzo

Advanced Organic Reactions
(Lecture notes)
Carmelo J. Rizzo
Associate Professor of Chemistry, Vanderbilt University
Nashville, Tennessee, USA
Source:
http://www.vanderbilt.edu/AnS/Chemistry/Rizzo/chem223/chem223.htm
SELECTIVITY
SELECTIVITY
Science 1983, 219, 245
Chemoselectivity
preferential reactivity of one functional group (FG) over another
- Chemoselective reduction of C=C over C=O:
H2,
Pd/C
O
O
- Chemoselective reduction of C=O over C=C:
O
OH
NaBH4
O
O
+
NaBH4, CeCl3
OH
only
- Epoxidation:
MCPBA
+
OH
OH
O
O
(2 : 1)
VO(acac)2,
tBuOOH
OH
OH
O
exclusively
Regioselectivity
- Hydration of C=C:
1) B2H6
2) H2O2, NaOH
OH
R
R
OH
R
1) Hg(OAc)2, H2O
2) NaBH4
- Friedel-Crafts Reaction:
O
RCOCl,
AlCl3
R
+
R
O
O
SiMe3
RCOCl,
AlCl3
R
OH
1
SELECTIVITY
- Diels-Alder Reaction:
R
R
O
O
R
+
+
O
major
O
R
minor
O
R
+
+
R
O
minor
major
O
O
+
O
H
OAc
O
O
H
OAc
O
O
OAc
O
O
OAc
OAc
Raney Ni, H2
+
O
H
SPh
O
O
H
O
O
SPh
H
O
Change in mechanism:
SPh
PhSH, H+
R
R
R
SPh
PhSH, (PhCO2)2
Stereochemistry:
Relative stereochemistry: Stereochemical relationship between two or more stereogenic centers
within a molecule
H
H
HO
cholesterol
H
enantiomers
same relative stereochemistry
H
OH
syn: on the same side ( cis)
anti: on the opposite side (trans)
- differences in relative stereochemistry lead to diastereomers.
Diastereomers= stereoisomers which are not mirror images; usually have different physical
properties
2
SELECTIVITY
Absolute Stereochemistry: Absolute stereochemical assignment of each stereocenter (R vs S)
Cahn-Ingold-Prelog Convention (sequence rules)
- differences in absolute stereochemistry (of all stereocenters within the molecule) leads to
enantiomers.
- Reactions can "create" stereocenters
O
Ph
MeMgBr
HO
H
Ph
MeMgBr
CH3
H
H 3C
OH
Ph
H
O
enantiomers
(racemic product)
Ph
H
HO
CH3
Ph
H
MeMgBr
Diastereomeric transition states- not necessarily equal in energy
Me
Me
Me
N
O
Ph
Zn
O
CH3 CH3
Zn
Me
N
O
O
CH3 CH3
H
Zn
H 3C
H
Zn
Ph
H 3C
HO
Ph
CH3
HO
CH3
H
H
Ph
Diastereoselectivity
CH3MgBr
Ph
CH3
Ph
CHO
HO
+
CH3
Ph
H
H OH
anti
syn
Diastereomers
Cram Model (Cram's Rule): empirical
O
O
M
S
R
O
S
M
H 3C
Nu
H
CH3MgBr
CH3MgBr
L
R
L
favored
H
Ph
3
SELECTIVITY
4
Felkin-Ahn Model
S O
O M
L
R
L
Nu
Nu
M R
S
disfavored
favored
Chelation Control Mode
M
OR
O
O
R
S
M
HO
OR
CH3MgBr
CH3MgBr
M
S
favored
Nu
M
R
S
OR
R
HO
TBSO
O
TBSO
MgBr
relative stereochemical control
H O
H O
OBn
OBn
Stereospecific
Stereochemictry of the product is related to the reactant in a mechanistically defined manner; no
other stereochemical outcome is mechanistically possible.
i.e.; SN2 reaction- inversion of configuration is required
Br2
H
H 3C
Br
Br2
H
meso
H
CH3
Br
CH3 Br
+
H
CH3
Br
CH3
H
Br
H
CH3 Br
enantiomers (racemic)
Stereoselective
When more than one stereochemical outcome is possible, but one is formed in excess (even if that
excess is 100:0).
CH3
H2, Pd/C
H
H
α-pinene
O
only isomer
O
O
H
H
O
H
+
CH3
H
not observed
O
O
LDA, CH3I
N
O
S
N
O
+
S
N
O
S
(96 : 4)
Diasteromers
Diastereoselective
Enantiospecific
OXIDATIONS
Oxidations
Carey & Sundberg: Chapter 12 problems: 1a,c,e,g,n,o,q; 2a,b,c,f,g,j,k; 5; 9 a,c,d,e,f,l,m,n; 13
Smith: Chapter 3
March: Chapter 19
I. Metal Based Reagents
1. Chromium Reagents
2. Manganese Rgts.
3. Silver
4. Ruthenium
5. other metals
II Non-Metal Based Reagents
1. Activated DMSO
2. Peroxides and Peracids
3. Oxygen/ ozone
4. others
III. Epoxidations
Metal Based Reagents
Chromium Reagents
- Cr(VI) based
- exact stucture depends on solvent and pH
- Mechanism: formation of chromate ester intermediate
Westheimer et al. Chem Rev. 1949, 45, 419
JACS 1951, 73, 65.
HO
R2CH-OH
HCrO4
-
R
R
H+
Cr
C O
O-
R
O-
O
R
+ HCrO3- +
H+
H
+ H2O
Jones Reagent (H 2CrO4, H2Cr2O7, K2Cr2O7)
J. Chem. Soc. 1946 39
Org. Syn. Col. Vol. V, 1973, 310.
- CrO3 + H2O → H2CrO4 (aqueous solution)
K2Cr2O7 + K2SO4
- Cr(VI) → Cr(III)
(black)
(green)
- 2°- alcohols are oxidized to ketones
R2CH-OH
Jones
reagent
R
acetone
R
O
- saturated 1° alcohols are oxidized to carboxylic acids.
Jones
reagent
RCH2-OH
acetone
O
R
hydration
H
HO OH
Jones
reagent
R
acetone
H
O
R
OH
- Acidic media!! Not a good method for H+ sensitive groups and compounds
5
OXIDATIONS
1) Jones,
acetone
SePh
OH
6
SePh
CO 2CH 3
2) CH2N2
Me 3Si
Me 3Si
JACS 1982, 104, 5558
H17C 8
H17C 8
O
O
O
OH
Jones
acetone
O
O
O
JACS 1975, 97, 2870
O
Collins Oxidation (CrO3•2pyridine)
TL 1969, 3363
- CrO3 (anhydrous) + pyridine (anhydrous) → CrO 3•2pyridine↓
- 1° and 2° alcohols are oxidized to aldehydes and ketones in non-aqueous solution (CH 2Cl2)
without over-oxidation
- Collins reagent can be prepared and isolated or generated in situ. Isolation of the reagent
often leads to improved yields.
- Useful for the oxidation of H+ sensitive cmpds.
- not particularly basic or acidic
- must use a large excess of the rgt.
CrO3•(C 5H5N)2
OH
ArO
H
CH 2Cl 2
O
O
ArO
O
JACS 1969, 91, 44318.
O
O
CrO3 catalyzed (1-2 mol % oxidation with NaIO6 (2.5 equiv) as the reozidant in wet aceteonitrile.
oxidized primary alcohols to carboxylic acids.
Tetrahedron Lett. 1998, 39, 5323.
Pyridinium Chlorochromate (PCC, Corey-Suggs Oxidation)
TL 1975 2647
Synthesis 1982, 245 (review)
CrO3 + 6M HCl + pyridine → pyH+CrO3 Cl- ↓
- Reagent can be used in close to stoichiometric amounts w/ substrate
- PCC is slighly acidic but can be buffered w/ NaOAc
PCC, CH 2Cl 2
OHC
HO
JACS 1977, 99, 3864.
O
O
O
PCC, CH 2Cl 2
OH
O
CHO
TL, 1975, 2647
OXIDATIONS
- Oxidative Rearrangements
Me
OH
Me
PCC, CH 2Cl 2
JOC 1977, 42, 682
O
Me
Me
PCC, CH 2Cl 2
JOC 1976, 41, 380
OH
O
- Oxidation of Active Methylene Groups
PCC, CH 2Cl 2
O
O
O
JOC 1984, 49, 1647
PCC, CH 2Cl 2
O
O
O
- PCC/Pyrazole PCC/ 3,5-Dimethylpyrazole
JOC 1984, 49, 550.
NH
NH
N
N
- selective oxidation of allylic alcohols
OH
OH
PCC, CH 2Cl 2
H
3,5-dimethyl
pyrazole
H
HO
H
O
H
(87%)
Pyridinium Dichromate (PDC, Corey-Schmidt Oxidation)
TL 1979, 399
- Na2Cr2O7•2H2O + HCl + pyridine → (C5H5N)2CrO7 ↓
PDC
PDC
CHO
CH 2Cl 2
OH
DMF
1° alcohol
-allylic alcohols are oxidized to α,β-unsaturated aldehydes
CO 2H
7
OXIDATIONS
- Supported Reagents
Comprehensive Organic Synthesis 1991, 7, 839.
PCC on alumina : Synthesis 1980, 223.
- improved yields due to simplified work-up.
PCC on polyvinylpyridine : JOC, 1978, 43, 2618.
CH 2 CH
cross-link
N
CH 2 CH
R2CH-OH
R2C=O
CH 2 CH
CrO3, HCl
N
N
Cr(III)
N
Cr(VI)O3 •HCl
8
partially
spent
reagent
to remove Cr(III)
1) HCl wash
2) KOH wash
3) H2O wash
CrO3/Et2O/CH2Cl2/Celite
Synthesis 1979, 815.
- CrO3 in non-aqueous media does not oxidized alcohols
- CrO3 in 1:3 Et2O/CH2Cl2/celite will oxidized alcohols to ketone and aldehydes
C 8H17
C 8H17
CrO3
Et2O/CH 2Cl 2/celite
(69%)
HO
Synthesis 1979, 815
O
H2CrO7 on Silica
- 10% CrO3 to SiO2
- 2-3g H2CrO3/SiO2 to mole of R-OH
- ether is the solvent of choice
Manganese Reagents
Potassium Permanganate
KMnO4/18-Crown-6
JACS 1972 94, 4024.
(purple benzene)
O
O
O
K+
O
MnO 4O
O
- 1° alcohols and aldehydes are oxidized to carboxylic acids
- 1:1 dicyclohexyl-18-C-6 and KMnO4 in benzene at 25°C gives a clear purple solution as high
as 0.06M in KMnO4.
O
JACS 1972, 94, 4024
CO 2H
CHO
Synthesis 1984, 43
CL 1979, 443
CHO
OXIDATIONS
9
Sodium Permanganate
TL 1981, 1655
- heterogeneous reaction in benzene
- 1° alcohols are oxidized to acids
- 2° alcohols are oxidized to ketones
- multiple bonds are not oxidized
Barium Permanganate
(BaMnO4)
TL 1978, 839.
- Oxidation if 1° and 2° alcohols to aldehydes and ketones- No over oxidation
- Multiple bonds are not oxidized
- similar in reactivity to MnO2
Barium Manganate
BCSJ 1983, 56, 914
Manganese Dioxide
Review: Synthesis 1976, 65, 133
- Selective oxidation of α,β-unsatutrated (allylic, benzylic, acetylenic) alcohols.
- Activity of MnO2 depends on method of preparation and choice of solvent
- cis & trans allylic alcohols are oxidized at the same rate without isomerization of the double
bond.
OH
OH
HO
HO
MnO 2, CHCl3
J. Chem. Soc. 1953, 2189
JACS 1955, 77, 4145.
(62%)
O
HO
- oxidation of 1° allylic alcohols to α,β-unsaturated esters
OH
MnO2,
ROH, NaCN
CO 2R
OH
CO 2Me
JACS1968, 90, 5616. 5618
MnO 2, Hexanes
MeOH, NaCN
Manganese (III) Acetate
α-hydroxylation of enones
Synthesis 1990, 1119
TL 1984 25, 5839
O
O
Mn(OAc)3, AcOH
AcO
Ruthenium Reagents
Ruthenium Tetroxide
- effective for the conversion of 1° alcohols to RCO2H and 2° alcohols to ketones
- oxidizes multiple bonds and 1,2-diols.
OXIDATIONS
Ph
OH
O
H
O
JOC 1981, 46, 3936
RuO4, NaIO4
OH
CH 3
CO 2H
Ph
CCl 4, H2O, CH3CN
OH
Ph
RuO4, NaIO4
10
Ph
CCl 4, H2O, CH3CN
CO 2H
H
96% ee
CH 3
94%ee
HO
RuO2, NaIO4
O
TL 1970, 4003
CCl 4, H2O
O
O
O
O
Tetra-n-propylammonium Perruthenate (TPAP, nPr4N+ RuO4-)
Aldrichimica Acta 1990, 23, 13.
Synthesis 1994, 639
- mild oxidation of alcohols to ketones and aldehydes without over oxidation
OH
O
TPAP
MeO 2C
MeO 2C
OSiMe 2tBu
OSiMe 2tBu
O
N+
-O Me
TL 1989, 30, 433
(Ph3P)4RuO2Cl3
RuO2(bipy)Cl2
- oxidizes a wide range of 1°- and 2°-alcohols to aldehydes and ketones without oxidation of
multiple bonds.
OH
CHO
CHO
OH
JCS P1 1984, 681.
H
H
Ba[Ru(OH)2O3]
-oxidizes only the most reactive alcohols (benzylic and allylic)
(Ph3P)3RuCl2 + Me3SiO-OSiMe3
- oxidation of benzylic and allylic alcohols
TL 1983, 24, 2185.
Silver Reagents
Ag2CO3 ( Fetizon Oxidation)
also Ag2CO3/celite
- oxidation of only the most reactive hydroxyl
O
OH
Synthesis 1979, 401
O
Ag 2CO 3
O
OH
OH
O
O
OH
O
OH
Ag 2CO 3, C 6H6
O
O
O
JACS 1981, 103, 1864.
mechanism: TL 1972, 4445.
OXIDATIONS
- Oxidation of 2° alcohol over a 1° alcohol
OH
OH
Ag2CO3, Celite
OH
JCS,CC 1969, 1102
(80%)
O
Silver Oxide (AgO2)
- mild oxidation of aldehyde to carboxylic acids
AgO 2, NaOH
RCHO
CHO
RCO 2H
CO 2H
AgO 2
JACS 1982, 104, 5557
Ph
Ph
Prevost Reaction Ag(PhCO2)2, I2
Ag(PhCO 2)2, I2
AcO
OAc
AcOH
AcO
Ag(PhCO 2)2, I2
OH
AcOH, H 2O
Other Metal Based Oxidations
Osmium Tetroxide OsO 4
review: Chem. Rev. 1980, 80, 187.
-cis hydroxylation of olefins
old mechanism:
O
OH
Os
O O
OH
O
OsO 4, NMO
osmate ester
intermediate
cis stereochemistry
- use of R 3N-O as a reoxidant
TL 1976, 1973.
OsO 4, NMO
O
O
O
OH
O
OH
OH
OH
TL 1983, 24, 2943, 3947
Stereoselectivity:
OsO 4
R3
R2
RO
H
R4
OsO 4, NMO
HO H
R2
HO
R3
RO
H R4
11
OXIDATIONS
- new mechanism: reaction is accelerated in the presences of an 3° amine
R1
R1
O
O O
R2
Os
O
O
[2+2]
R3N
R1
R2
O Os
O
12
O
Os O
O
O
R2
O
NR3
[O]
[3+2]
OsO2
R1
O
O
R2
O
[O]
hydrolysis
R1
Os O
R2
+
O
HO
OH
OsO4
- Oxidative cleavage of olefins to carboxylic acids.
JOC 1956, 21, 478.
- Oxidative cleavage of olefins to ketones & aldehydes.
OH
CHO
CHO
OH
OsO 4, NMO
O
O
NaIO4
OH
O
H2O
O
O
O
O
O
O
OAc
O
OAc
OAc
JACS 1984, 105, 6755.
Substrate directed hydroxylations:
-by hydroxyl groups
Chem. Rev. 1993, 93, 1307
HO
OsO4,
pyridine
O
HO
HO
O
HO
+
O
HO
HO
HO
3:1
HO
OsO4,
pyridine
O
HO
O
TMSO
TMSO
HO
CH3
HO
CH3 OH
OsO4, Et2O
HO
OH
CH3
CH3
+
OH
CH3
(86 : 14)
- by amides
AcO
AcO
OH
MeS
OsO4
MeS
OH
HN
O
OAc
CH3 OH
HN
O
OAc
OXIDATIONS
- by sulfoxides
••
••
OMe
O
OsO4
S
OMe OH
O
S
OH
1) OsO4
2) Ac2O
S
HN
OAc
(2 : 1)
••
••
O
13
O
S
O
AcO
O
HN
(20 : 1)
- by sulfoximines
O
Ph S
O
Ph S
OH
MeN
MeN
OsO4, R3NO
O
OH
∆
OH
OH
OH
OH
CH3
Raney nickel
H 3C
OH
OH
OH
CH3
- By nitro groups
PhO2S
PhO2S
1) OsO4
N
NHR
N
+
2) acetone, H
N
NHR
N
O2N
O2N
N
N
N
O
N
O
N
N
HO
HO
NHR
N
NHR
N
N
N
N
O
N
O
- OsO4 bis-hydroxylation favors electon rich C=C.
OsO4
X
OH
OH
X
+
OH
OH
X= OH
= OMe
= OAc
= NHSO2R
- Ligand effect:
80 : 20
98 : 2
99 : 1
60 : 40
OsO4
OH
K3Fe(CN)6, K2CO3
MeSO2NH2, tBuOH/H2O
OH
OH
OsO4 (no ligand)
Quinuclidine
DHQD-PHAL
X
4:1
9:1
> 49 : 1
+
X
(directing effect ?)
(directing effect ?)
OH
OH
OH
OXIDATIONS
Chem. Rev. 1994, 94, 2483.
Sharpless Asymmetric Dihydroxylation (AD)
- Ligand pair are really diastereomers!!
14
dihydroquinidine ester
N
"HO
Ar
OH"
H
H
R3
OR'
R2
R3 OH
0.2-0.4% OsO4
R2
R1
acetone, H 2O, MNO
80-95 % yield
20-80 % ee
OH
R1
H OR'
"HO
Ar
OH"
MeO
Ar =
N
dihydroquinine ester
N
R'= p-chlorobenzoyl
Mechanism of AD:
L
HO
OH
O
O
H 2O
O
O
Os
O
O
O
O
First Cycle
(high enantioselectivity)
O
Os
O
O
Second Cycle
(low enantioselectivity)
[O]
[O]
O
O
Os
O
O
L
Os
O
L
O
O
O
O
Os
O
O
O
O
R 3N
HO
OH
H 2O, L
- K3Fe(CN)6 as a reoxidant gives higher ee's- eliminates second cycle
TL 1990, 31, 2999.
- Sulfonamide effect: addition of MeSO2NH2 enhances hydrolysis of Os(VI) glycolate
(accelerates reaction)
- New phthalazine (PHAL) ligand's give higher ee's
N
Et
Et
O
H
Et
N
N N
N
N
O
H
O
H
OMe
MeO
O
H
MeO
N
OMe
N
N
N
(DHQ)2-PHAL
(DHQD)2-PHAL
JOC 1992, 57, 2768.
Et
N N
OXIDATIONS
15
- Other second generation ligands
N
Et
Et
O
H
MeO
Et
N
Ph
O
N
N
H
N
OMe
Ph
N
N
O
H
O
OMe
N
N
PYR
IND
Proposed catalyst structure:
O
H
O
O
Os
N
MeO
N
Os
"Bystander
quinoline
(side wall)
Asymmetric
Binding
Cleft
O
N
H
H
N
N
O
N
N
N
O
Phthalazine
Floor
OMe
OMe
OMe
O
Corey Model: JACS 1996, 118, 319
Enzyme like binding pocket;
[3+2] addition of OsO4 to olefin.
N
O
O
Os O
N O
O
N
H
N N
O
O
N
DHQL
Rs
RM
RL
H
DHQ
RL large and flat,
i.e Aromatics work particularly well
OXIDATIONS
Olefin
Preferred Ligand
ee's
PYR, PHAL
30 - 97 %
PHAL
70 - 97 %
IND
20 - 80 %
PHAL
90 - 99.8 %
PHAL
90 - 99 %
PHAL, PYR
+ MeSO2NH2
20 - 97 %
R1
R2
R1
R1
R2
R2
R1
R2
R3
R1
H
R2
R3
R1
R4
"AD-mixes" commercially available pre-mix solutions of Os, ligand and reoxidant
AD-mix α
(DHQ)2PHAL, K 3Fe(CN)6, K2CO3, K2OsO4 (0.4 MOL % Os to C=C)
AD-mix β
(DHQD)2PHAL, K 3Fe(CN)6, K2CO3, K2OsO4
O
HO
O
Campthothecin
N
N
O
OMe
N
OMe
OMe
AD
(DHQD)2PYR
O
N
N
O
94 % ee
O
O
OH
OH
OH
- Kinetic resolution (not as good as Sharpless asymmetric epoxidation)
H
Ph
tBu
Ph
H
tBu
H
Ph
H
Ph
AD mix α
30% conversion
Ph
H
OH
OH
tBu
tBu
olefins with axial
dissymmetry
H
Ph
+
OH
OH
+
tBu
(4 : 1)
tBu
enriched
16
OXIDATIONS
17
Asymmetric Aminohydroxylation TL 1998, 39, 2507; ACIEE 1996, 25, 2818, 2813,
preparation of α-aminoalcohols from olefin. Syn addition as with the dihydroxylation
regiochemistry can be a problem
O
Ph
O
N
Na
CO2Me
Ph
O
OH
Cl
Ph
O
NH
+
CO2Me
Ph
K2OsO6H4 (cat)
Ligand
CO2Me
Ph
N
OH
O
Ph
O
Ligand= PHAL
AQN
4:1
1:4
Molybdenum Reagents
MoOPH [MoO5•pyridine (HMPA)]
JOC 1978, 43, 188.
- α-hydroxylation of ketone, ester and lactone enolates.
O
OR'
R
O
+
Mo
O
L
R
O
H
R
R
Pd(OAc) 2,
CH 3CN, 80° C
HO
H
R
O
H
R
- CO 2
O
O Pd
-
TL 1984, 25, 2791
Tetrahedron 1987, 43, 3903
O
OH
2
HO
CO
H
OH
JACS 1989, 111, 8039.
Pd2(DBA) 3•CHCl 3,
CH 3CN, 80° C
OH
O
R
R
Pd(0)
O
H
H
(Tsuji Oxidation)
O
O 2 CO
OH
R'
OH
L
Palladium Reagents
Pd(0) catalyzed Dehydrogenation (oxidation) of Allyl Carbonates
Tetrahedron 1986, 42, 4361
R
O
THF, -78°C
O
H
O
O
Oxidation of silylenol ethers and enol carbonates to enones
O
OTMS
Pd(OAc) 2,
CH 3CN
O
O
O
OTIPS
Ph
O
Pd(OAc) 2,
CH 3CN
(NH 4)2Ce(NO 3)6
DMF, 0°C
O
O
Ph
TL 1995, 36, 3985
R
Oppenauer Oxidation
OXIDATIONS
Organic reactions 1951, 6, 207
Synthesis 1994, 1007
OiPr
+O Al
R1R2CHOH
(CH3)2C=O
OiPr
OiPr
+O Al
O OiPr
H
R1
R2
O
R1
18
+ Al(OiPr)3
R2
Nickel Peroxide
Chem Rev. 1975, 75, 491
Thallium Nitrate (TNN, Tl(NO 3)3•3H2O
Pure Appl. Chem. 1875, 43, 463.
Lead Tetraacetrate Pb(OAc)4
Oxidations in Organic Chemistry (D), 1982, pp 1-145.
Non-Metal Based Reagents
Activated DMSO Review: Synthesis 1981, 165; 1990, 857.
Me
Me
S+
S+
+ E
O-
Me
E
O
Organic Reactions 1990, 39, 297
Nu:
Nu
S
Me
Me
+
+ E-O
Me
E= (CF3CO)2O, SOCl2, (COCl)2, Cl2, (CH3CO)2O, TsCl, MeCl, SO3/pyridine, F 3CSO2H,
PO5, H3PO4, Br2
Nu:= R-OH, Ph-OH, R-NH2, RC=NOH, enols
Swern Oxidation
- trifluoroacetic anhydride can be used as the activating agent for DMSO
O
Me
Me
(COCl) 2
S + O-
CH 2Cl 2, -78°C
Me
R2CH-OH Me
Me
Me
-CO, -CO 2
Cl -
Me
S + Cl
Me
O
R
R
S+ O
Cl
S+ O
Et3N:
Me
R
S
+
O
Me
R
H
B:
O
Cl
O
DMSO, (COCl) 2
OH
Moffatt Oxidation (DMSO/DCC)
O
JACS 1965, 87, 5661, 5670.
Me
C 6H11
S + O-
CF 3CO 2H,
Pyridine
Me
+
C 6H11 N C
TL 1988, 29, 49.
CH 2Cl 2, Et3N
N C 6H11
OH
CO 2Me
O
Me
NH
S
+
O C
R2CH-OH
Me
R
O
H
R
B:
C 6H11
CHO
DCC/ DMSO
CO 2Me
CF 3CO 2H,
Pyridine
JACS 1978, 100, 5565
O
S
SO3/Pyridine
S+ O
N
Me
R
R
Me
S
JACS 1967, 89, 5505.
CO 2Me
HO
H
CONH 2
H
HO
OH
OH
CO 2Me
H
SO 3, pyridine,
DMSO, CH 2Cl 2
CONH 2
H
HO
O
JACS 1989, 111, 8039.
OXIDATIONS
Corey-Kim Oxidation
(DMS/NCS)
19
JACS 1972, 94, 7586.
O
Me
Me
S:
+
S + Cl
N Cl
Me
Me
O
N-Chlorosuccinimide
(NCS)
Acc. Chem. Res. 1980, 13, 419
••
••
••
O O
singlet
"ene" reaction
H
O
Tetrahedron 1981, 37, 1825
hν
•• ••
•O O •
•• ••
triplet
••
Oxygen & Ozone
Singlet Oxygen
Ph3P:
H
O
O
O
OH
Tetrahedron 1981, 1825
1) O2, hν, Ph2CO
2) reduction
Ozone
HO
Comprehensive Organic Synthesis 1991, 7, 541
O
O 3, CH 2Cl 2
O
O
-78°C
O
Ph3P:
O O
NaBH 4
+
O
O
H
Jones
OH
RCOOH
Other Oxidations
Mukaiyama Oxidation
BCSJ 1977, 50, 2773
O
R
PrMgBr
CH OH
R
N
R
N
N
N
R
O
CH O MgBr
O
R
THF
R
OH
Cl
MeO
CH 3
O
O
NH
O
O
SEt
SEt
MeO
N
Cl
O
N N
N
O
MeO
CH 3
OHC
O
NH
OEt
O
tBuMgBr, THF
(70%)
SEt
SEt
MeO
JACS 1979, 101, 7104
OEt
OXIDATIONS
20
O
OH
tBuMgBr, THF
O
N
N
N
N
O
O
O
Dess-Martin Periodinane
JOC 1983, 48, 4155.
- oxidation conducted in CHCl3, CH3CN or CH2Cl2
- excellent reagent for hindered alcohols
- very mild
JACS 1992, 113, 7277.
OAc
OAc
AcO
••
I
O
OAc
R
R2CH-OH
I
O
+
+ 2 AcOH
O
R
O
O
HO
Dess-Martin
O
JOC 1991, 56, 6264
(99%)
RO
RO
Chlorite Ion
-oxidation of α,β-unsaturated aldehydes to α,β−unsaturated acids.
Tetrahedron 1981, 37, 2091
NaClO 2,
NaH2PO 4
OBn
- HClO2
OBn
OBn
OH
H
tBuOH, H 2O
CHO
CO 2H
-O-Cl-O
Selenium Dioxide
- Similar to singlet oxygen (allylic oxidation)
1) SeO2
2) NaBH 4
OAc
OAc
OH
Phenyl Selenium Chloride
O
OLi
PhSeCl
O
SePh
H2O 2
Ph
Se
O-
THF
O
- PhSeOH
H
- PhS-SPh will do similar chemistry however a sulfoxide elimination is less facile than a
selenoxide elinimation.
Peroxides & Peracids
- R3N: → R3N-O
- sulfides → sulfoxides → sulfones
-Baeyer-Villiger Oxidation- oxidation of ketones to esters and lactones via oxygen insertion
Organic Reactions 1993, 43, 251
Comprehensive Organic Synthesis 1991, vol 7, 671.
OXIDATIONS
21
m-Chloroperbenzoic Acid, Peracetic Acid, Hydrogen peroxide
O
O
H
O
O 2N
O
O
O
H
O
R1
R2
O
O
HO
O
NO 2
Cl
R1
H
Ar
O
C R2
O
R1
O
+
R2
ArCO2H
Ar
O
O
- Concerted R-migration and O-O bond breaking. No loss of stereochemistry
- Migratory aptitude roughly follows the ability of the group to stabilize positive charge:
3° > 2° > benzyl = phenyl > 1° >> methyl
JACS 1971, 93, 1491
O
O
mCPBA
O
HO
O
CO2H
O
CHO
O
HO
O
O
CO2H
HO
OH
PGE1
O
O
CH3
O
mCPBA
Tetrahedron Lett. 1977, 2173
Tetrahedron Lett. 1978, 1385
CH3
(80 %)
CH3
CH3
Oxone (postassium peroxymonosulfate)
Tetrahedron 1997, 54, 401
oxone
RCHO
RCOOH
acetone (aq)
Oxaziridines
reviews: Tetrahedron 1989, 45, 5703; Chem. Rev. 1992, 92, 919
O
N C
R
R3
R2
- hydroxylation of enolates
O
R
O
Base
R
R'
O
_
R
R'
O
O
PhSO2
O
R
Ph
N
R
R'
+ PhSO2N=CHPh
HO
Ph
O
_
R'
O
R'
_
NSO2Ph
R
+ PhSO2N=CHPh
Ph
R'
NHSO2Ph
By-product
supresed by using
bulkier oxaziradine
such as camphor
oxaziradine
OXIDATIONS
22
Asymmetric hydroxylations
O
O
NaN(SiMe3)2, THF
MeO 2C
HO
Tetrahedron 1991, 47, 173
MeO 2C
OMe
OMe
N
Ar
MeO
O
SO 2 O
MeO
KN(SiMe3)2
CO2Me
(67% ee)
O
O
OH
CO2Me
OH
O
OH
OH
N
SO2 O
MeO
MeO
MeO
O
OH
OH
(>95% ee)
- hydroxylation of organometallics
R-Li or R-Mg → R-OH
JACS 1979, 101, 1044
- Asymmetric oxidation of sulfides to chiral sulfoxides.
JACS 1987, 109, 3370.
Synlett, 1990, 643.
Remote Oxidation (functionalization)
Barton Reaction
Comprehensive Organic Synthesis 1991, 7, 39.
NOCl, CH2Cl2
pyridine
OH
hν
O
NO
- NO
•
O
•
OH
H
•
•NO
JACS 1975, 97, 430
OH
OH
NO
N
N
ketone
oxidation state
HO
C5H11
perhydrohistricotoxin
Epoxidations
Peroxides & Peracids
- olefins → epoxides
Tetrahedron 1976, 32, 2855
- α,β-unsaturated ketones, aldehydes and ester → α,β-epoxy- ketones, aldehydes and esters
(under basic conditions).
O
(CH 2)n
tBuOOH
triton B, C6H6
O
O
(CH 2)n
JACS 1958, 80, 3845
OXIDATIONS
O CO 2Me
CO 2Me
mCPBA, NaHPO3
TL 1988, 23, 2793
O
O
H
H
O
O
Henbest Epoxidation- epoxidation directed by a polar group
OH
OH
OH
mCPBA
+
O
OAc
O
10:1 diastereoselection
OH
OAc
mCPBA
+
O
O
1:4 diastereoselection
O
Ph
O
NH
Ph
NH
"highly selective"
mCPBA
O
Ar
O
H
H
O
proposed transition state:
-OH directs the epoxidation
O
O
H
- for acyclic systems, the Henbest epoxidation is often less selective
Rubottom Oxidation:
JOC 1978, 43, 1588
O
OTMS
LDA, TMSCl
TMSO
mCPBA
O
H2O
O
OH
Sharpless Epoxidation
tBuOOH w/ VO(acac)2, Mo(CO)6 or Ti(OR) 4
Reviews:
Comprehensive Organic Synthesis 1991, vol 7, 389-438
Asymmetric Synthesis 1985, vol. 15, 247-308
Synthesis, 1986, 89.
Org. React. 1996, 48, 1-299.
Aldrichimica Acta 1979, 12, 63
review on transition mediated epoxidations: Chem. Rev. 1989, 89, 431.
- Regioselective epoxidation of allylic and homo-allylic alcohols
- will not epoxidize isolated double bonds
- epoxidation occurs stereoselectively w/ respect to the alcohol.
23
OXIDATIONS
- Catalysts: VO(acac)2; Mo(CO)6; Ti(OiPr)4
- Oxidant: tBuOOH; PhC(CH3)2OOH
VO(acac)2
tBuOOH
OH
OH
O
OH
OH
(CH2)n
O
(CH2)n
ring size
5
6
7
8
9
VO(acac)2
>99%
>99
>99
97
91
MoO2(acac)2
-98
95
42
3
mCPBA
84
95
61
<1
<1
Acyclic Systems:
L
M
1,3-interaction
O
R3
A1,2-strain
tBu
O
O
R1
R3
L
Rc
Rt
R1
Rt
R2
Rc
O
O
M
R2
L
A1,3-strain
Major influences:
A1,2-Strain between Rg and R1
A1,3 -strain between R2 and Rc
1,3-interactions between L and R1
O
L
(Rg and R2)
(R1 and Rc)
(L and R2)
VO(acac)2,
tBuOOH
O
OH
+
O
OH
OH
(4 : 1)
tBu
CH3
H
O
L
M
H
H
L
O
O
M
L
O
O
L
tBu
O
H 3C
H
H
H
24
OXIDATIONS
VO(acac)2,
tBuOOH
O
OH
+
O
OH
OH
(19 : 1)
tBu
L
H 3C
H
H
H 3C
O
H
M
L
H
O
O
O
H 3C
H
O
M
L
L
H
O
tBu
CH3
SiMe3
SiMe3
VO(acac)2,
tBuOOH
O
OH
SiMe3
+
OH
O
OH
(> 99 : 1)
- Careful conformational analysis of acyclic systems is needed.
Homoallylic Systems
L
L
O
O
OH
V
OtBu
O
OH
dominent stereocontrol element
Titanium Catalyst structure:
RO2C
OR
Ti
CO2R
O
O
O
RO
O
OR
O
Ti
OR
OR
O
OR
CO2R
O
OR
Ti
CO2R
O
O
O
RO
O
OR
O
CO2R
Ti
O
RO
Ti
O
O
CO2R
O
CO2R
O
O
tBu
OR
Disfavored
O
O
tBu
OR
Ti
O
Favored
O
CO2R
25
OXIDATIONS
26
Asymmetric Epoxidation
tBuOOH, Ti(OiPr), (+) or (-) Diethyl Tartrate, 3Å molecular sieves
Empirical Rule
R1
(+)- DET epoxidation from the bottom
(-)- DET epoxidation from the top
R2
R3
OH
Catalytic system: addition of molecular sieves to "soak" up any water with 3A sieves, 5-10 mol %
catalyst is used.
Preparation of Allylic Alcohols:
R
CO2R'
[(CH3)2CHCH2]2AlH
Na (MeOCH2CH2O)2AlH2
R
(DIBAL)
OH
(REDAL)
R CHO
R C C CH2OH
R
CO2R'
R
[(CH3)2CHCH2]2AlH
H2, Lindlar's Catalyst
OH
"In situ" derivatization of water soluble epoxy-alcohol
(-)-DIPT
O
OH
(R)-glycidol
OH
water soluble
O
(+)-DIPT
O
O
OH
OH
(S)-glycidol
O
O S
NO2
organic soluble
O
Alkoxide opening of epoxy-alcohol product
reduced by use of Ti(OtBu)4 and catalytic conditions
OO
R
OH
O
OH
OH
from
Ti(OiPr)4
R
Stoicheometric vs Catalytic epoxidation:
(+)-DET
Ti(OiPr)4
tBuOOH
O
OH
stoicheometric:
catalytic (6-7 mol %)
in situ deriv. with PNB
OH
85% ee
47% yield
>95% ee
78% yield
92 % ee >98 %ee after 1 recrystallization
(+)-DET
Ti(OiPr)4
tBuOOH
R
R
OH
yields: 50 - 100 %
ee:
> 95%
O
OH
OXIDATIONS
Ring Opening of Epoxy-Alcohols
OH
REDAL
R
O
AE
R
OH
OH
1,3-Diol
R
OH
DIBAL
R
OH
OH
1,2-Diol
Two dimensional amplification
OH
OH
OH
(+)-DIPT, Ti(OiPr)4,
tBuOOH, 3A sieves
+
(90 % ee)
O
OH
O
OH
95 : 5
major
major
O
O
minor
minor
major
minor
95 : 5
95 : 5
OH
OH
O
O
OH
90 %
(>99.5 % ee)
OH
O
O
OH
meso
9.75%
(+)-DIPT, Ti(OiPr)4,
tBuOOH, 3A sieves
OH
R
+
R
OR
Ti
CO2R
O
O
O
RO
O
O
tBu
OR
H
Ti
O
O
R
CO2R
O
OH
0.25 %
Kinetic Resolution of Allylic Alcohols
OH
OH
O
CO2R
OH
27
OXIDATIONS
R3
R4
R3
kinetic resolution
-20 °C, 0.5 - 6 days
OH
R2
R4
R4
O
+
OH
R2
R1
R3
R2
R1
OH
R1
40 - 50 % yield
> 99 % ee
40 - 50 % yield
high ee
Reiterative Approach to the Synthesis of Carbohydrate
OR
OR
OR
(+)-DET, Ti(OiPr)4,
tBuOOH, 3A sieves
DIBAL
(MeO)2P(O)CH2CO2Me
NaH
CHO
OH
CO2Me
OR
OR
HOO
OH
PhS
OR
OH
OR
OH
acetone, H+
O
mCPBA, Ac2O
O
-
HO
Pummerer
O
SPh
SPh
PhS-
O
O
O
O
DIBAL
OAc
CHO
OR
OR
OR
O
O
O
CHO
HO
H
HO
H
HO
H
HO
SPh
O
CHO
H
CH2OH
L-glucose
Jacobsen Aysmmetric Epoxidation
JACS 1990, 112, 2801; JACS 1991, 113, 7063; JOC 1991, 56, 2296.
- Reaction works best for cis C=C conjugated to an aromatic ring
H
H
H
N
N
H
N
NaOCl
Mn
Mn
O
O Cl O
tBu
O
O
tBu
tBu
O
N
5 mol % Cat. ,NaOCl,
H2O, CH2Cl2
tBu
O
(98% ee)
O
86% ee
O
Methyltrioxoruthenium (MTO) Ru(VII)
Sharpless et al. JACS 1997, 117, 7863, 11536.
Ph
0.5 mol % MTO Ru (VII),
pyridine, CH2Cl2
1.5 eq. 30% H2O2 (aq.)
Ph
O
28
OXIDATIONS
Oxaziridines
- Asymmetric epoxidation of olefins
29
Tetrahedron 1989 45 5703
CH3
Ph
O2 O
N S N
*
(Murray's Reagent)
C6F5
*
Ph
Dioxiranes
*
Reviews: Chem. Rev. 1989, 89, 1187; ACR 1989, 27, 205
Org. Syn. 1996, 74, 91
O
KHSO 5
O
"oxone"
O
- epoxidation of olefins
O
OTBS
TBSO
TBSO
-
OTBS
O
O
O
TBSO
TBSO
CH 2Cl 2, acetone
(100%)
JOC 1990, 55, 2411
O
Asymmetric epoxidation JACS 1996, 118, 491.
oxidation of sulfides to sulfoxides and sulfones
oxidation of amines to amine-N-oxides
oxidation of aldehydes to carboxylic acids
hydroxylation of enolates
1) LDA
2) Cp 2TiCl2
3)
O
OH
H
O
JOC 1994, 59, 2358
O
O
- bis-trifluoromethyldioxirane, much more reactive
JACS 1991, 113, 2205.
F3C
O
F3C
O
- oxidation of alcohols to carbonyl compounds. 1° alcohols give a mixture of aldehydes and
carboxylic acids.
- Insertion into 3° C-H bonds to give R3C-OH
DCC-H2O2
JOC 1998, 63, 2564
R
R N C N R
H2O2, MeOH
H
N
R
N
C
H
O
O
O
O
R
R
+ H
N
R
C
N
R
H
REDUCTIONS
Carey & Sundberg Chapter 5 problems: 1a,b,c,d,f,h,j; 2; 3a-g, n,o; 4b,j,k,l; 9; 11;
Smith: Chapter 4 March: Chapter 19
30
Reductions
1. Hydrogenation
2. Boron Reagents
3. Aluminium Reagents
4. Tin Hydrides
5. Silanes
6. Dissolving Metal Reductions
Hydrogenations
Heterogeneous Catalytic Hydrogenation
Transition metals absorbed onto a solid support
metal: Pd, Pt, Ni, Rh
support: Carbon, alumina, silica
solvent: EtOH, EtOAc, Et2O, hexanes, etc.
-
Reduction of olefins & acetylenes to saturated hydrocarbons.
Sensitive to steric effects and choice of solvent
Polar functional groups, i.e. hydroxyls, can sometimes direct the delivery of H2.
Cis addition of H2.
R1
R1
R2
R2
H
H2, Pd/C
R1
H
R2
R1
R2
- Catalyst can be "poisoned"
- Directed heterogeneous hydrogenation
O
O
H
H2, Pd/C
O
O
OH
MeO
O
OH
MeO
H2, Pd/C
H
O
O
CO2Me
MeO
O
CO2M2
(86 : 14)
MeO
Lindlar Catalyst
( Pd/ BaSO4/ quinoline)- partially poisoned to reduce activity; will only
reduce the most reactive functional groups.
acetylenes + H2, Pd/BaSO4/ quinoline → cis olefins
Acid Chlorides + H2, Pd/BaSO4 → Aldehydes
R
SiMe 3
(Lindlar Reduction)
(Rosemund Reduction)
Org. Rxn. 1948, 4, 362
H2, Pd/BaSO4, Quinoline
SiMe 3
TL 1976, 1539
R
O
O
H2, Pd/BaSO4, pyridine
CO 2Me
CO 2Me
JOC 1982, 47, 4254
REDUCTIONS
HO
OH
H2, Lindlar Catalyst
CH 2Cl 2: MeOH: Quinoline
(90:9.5:0.5)
8π e-
JACS 1982, 104, 5555
6π edisrotatory
conrotatory
HO
OH
HO
H
H
HO
OH
H
OH
Ease of Reduction: (taken from H.O. House Modern Synthetic Reactions, 2nd edition)
R COCl
R CHO
R NO 2
R NH 2
R
R'
R
R CH 2-OH
R CHO
R CH CH
R CH 2 CH 2 R'
R'
O
R
HO H
R'
Ar
R'
R
O
R
R'
Ar
CH 3
+
HO R
R CH 2 NH 2
R C N
O
R CH 2-OH
R
+
HO R'
OR'
O
R'
R
R
N
CH 2
R'
N
R'
R'
requires high
temperature & pressure
R CO 2- Na+
no reaction
Raney Nickel Desulfuriztion ,
Reviews:
Org. Rxn. 1962, 12, 356; Chem. Rev. 1962, 62, 347.
R
R S
R
Raney Nickel
R
H
R
H
(CH 2)n
O
R S
31
REDUCTIONS
O O
O
O O
HO
32
O
HO
Raney Nickel
JOC 1987, 52, 3346
EtOH
(74%)
H
S
H
S
Homogeneous Catalytic Hydrogenation
- catalyst is soluble in the reaction medium
- catalyst not "poisoned" by sulfur
- very sensitive to steric effects
- terminal olefins faster than internal; cis olefins faster than trans
R
R
>
>
R
R
R
R
>
R
R
R
>
R
R
>>
R
R
R
- (Ph3P)3RhCl (Wilkinson's Catalyst); [R3P Ir(COD)py]+ PF6- (Crabtree's Catalyst)
(Ph3P)3RhCl, H2
OH
OH
JOC 1992, 57, 2767
C 6H6
(92%)
Directed Hydrogenation
Review: Angew. Chem. Int. Ed. Engl. 1987, 26 , 190
- Diasterocontrolled hydrogenation of allylic alcohols directed by the -OH group
-
+
O K
O- K+
PPh3
O
PPh3
Rh H
H
(Ph3P)3RhCl, H2
H
MeO
MeO
Ph Ph
P
Rh+
P
Ph Ph
Brown's Catalyst
BF4-
Ir +
P(C 6H11)3
N
PF6 -
(Crabtree's Ctalyst)
JACS 1983, 105 , 1072
Regioselective Hydrogenation- allylic and homoallylic alcohols are hydrogenated faster than
isolated double bonds
MeO2C
OH
MeO2C
OH
REDUCTIONS
33
mechanism:
L
+
M
H2
+
M
L
L
L
+
OH
L
S
M
L
S
O
H
lose
H
H
H2
(oxidative
addition)
reductive
elimination
OH
migratory
insertion
HO
H
L
M
L
+
H
H
M
L
S
L
O
H
Diastereoselective Hydrogenation: since -OH directs the H2, there is a possibility for control of
stereochemistry
- sensitive to: H2 pressure
catalyst conc.
substrate conc.
solvent.
Regioselective Hydrogenation- allylic and homoallylic alcohols are hydrogenated faster than
isolated double bonds
HO
"Ir"
(20 mol %)
HO
O
O
H
Brown's Catalyst
OH
JACS 1984, 106, 3866
H2 (640 psi), CH2Cl 2
H
OMe
OMe
Me
(24 : 1)
OH
OMe
H
Me
OMe
Brown's Catalyst
H2 (1000 psi)
O
TL 1987, 28 , 3659
O
OH
OH
Selectivity is often higher with lower catalyst concentration:
OH
OH
OH
20 mol %
Catalyst
2.5 mol %
Catalyst
50 : 1
150 : 1
33 : 1
52 : 1
OH
REDUCTIONS
34
Olefin Isomerization:
CH3
(3 : 1)
OH
OH
olefin
isomerization
O
OH
major
product
- Conducting the hydrogenation at high H 2 pressures supresses olefin isomerization and often
gives higher diastereoselectivity.
Other Lewis basic groups can direct the hydrogenation. (Ir seems to be superior to Rh for
these cases)
OMe
CO2Me
OMe
Ir
CO2Me
+
Ir+
99 : 1
O
O
CO2H
Ir+ 7 : 1
Rh+ 1 : 1
O
O
N
O
O
N
O
130 : 1
1:1
Acyclic Examples
Rh+ (2 mol %)
OH
Me
CH 3
JCSCC 1982, 348
H2 (15 psi)
Me
L
L M
OH
(97:3)
H
O
OH
H
H
Ph
H 3C
H
H
L
CH3
Ph
M
L
Ph
anti
H
H
O
H
32 : 1
CO2H
> 99 : 1
O
> 99 : 1
Rh+
OH
1,2-strain
Ph
syn
REDUCTIONS
L
L
R3
H
OH
R3
35
H
O
M
R2
R1
OH
favored
R3
H
syn
R1
R2
R1
R2
R3
H
L
H
R1
R2
M
OH
disfavored
R3
1,2-strain
R1
R2
O
H
L
anti
- Supression of olefin isomerization is critical for acyclic stereocontrol !
L
L
OH
R2
H
O
M
OH
R2
H
R1
CH3
R1
CH3
R2
H
R1
anti
R2
olefin
isomerization
OH
R2
OH
L
L
R1
H
O
M
R2
H
H
R1
R2
CH2R2
R1
H
- Rh+ catalyst is more selective than Ir + for acyclic stereoselection.
Acyclic homoallylic systems:
HO
R1
R3
R2
relative
stereochemistry
is critical
A
E
R2
A
O
H
M
R3
1,3-strain
R3
L
L
E
1,2-strain R2
O
H
L
M
L
syn
REDUCTIONS
Rh+
OH
(20 mol %)
36
OH
H2 (15 psi)
OBz
OBz
TBSO
32 : 1
TBSO
Tetrahedron Lett. 1985, 26, 6005
Rh+ (20 mol %)
OH
OH
H2 (15 psi)
OBz
OBz
TBSO
8:1
TBSO
Asymmetric Homogeneous Hydrogenation
- Chiral ligands for homogeneous hydrogenation of olefins and ketones
H
O
P
OMe
MeO
PPh2
PPh2
O
P
H
DIOP
PPh2
PPh2
PPh2
PPh2
PPh2
PPh2
R= -CH3 PROPHOS
= -Ph
PHENPHOS
= -C6H11 CYCPHOS
CHIRAPHOS
DBPP
DIPAMP
Ph2P
PPh2
PPh2
X
PPh2
CO2tBu
BPPM
PPh2
CO 2H
Ph
HN
PPPFA
PPh2
P
BINAP
DIPHEMP
Rh (I) L*, H2
CO 2H
Ph
HN
Ph
(95% ee)
O
CO 2Me
Ph
O
CO 2H
NHAc
O
NH 2
HO
O
OH
L-DOPA
DIOP
DIPAMP
PPPFA
BINAP
NORPHOS
BPPM
85% ee
96% ee
93% ee
100% ee
95% ee
91% ee
PPh2
PPh2
PPh2
PPh2
DUPHOS
Fe
N
X= CH2 DPCP
X= N-R PYRPHOS
CAMPHOS
P
PPh2
PPh2
PPh2
PPh2
NORPHOS
NMe2
ACR 1983, 16, 106.
REDUCTIONS
37
General Mechanism: J. Halpern Science 1982, 217, 401 Asymmetric Synthesis 1985, vol 5, 41.
CO2Me
Ph
S
P
Rh
P
Ph
NHAc
S
P
Rh
NH
O
P
(fast equilibrium)
CO2Me
CH3
rate
limiting
CO2Me
Ph
H2
(slow)
NHAc
Ph
P
H
Rh
P
O
S
Ph
H
CO2Me
NH
S
P
(fast)
P
CO2Me
H NH
Rh
O
CH3
CH3
Detailed Mechanism:
P
*
P
MeO2C
+
S
Ph
CO2Me
NHAc
NH
P
*
S
Rh
Ph
Rh
P
P
Ph
CH3
O
CO2Me
HN
H 3C
P
O
major complex
minor complex
H2
(slow)
MeO2C
H
H
P
* P
H2
(slow)
NH
NH
Ph
Rh
CH3
O
H 3C
Rh
*
S
H
CH3
NH
Ph
H
N
H 3C
O
P
P *
S
CO2Me
P
H
Rh
O
CH3
O
CO2Me
H
Ph
S
P
*
Rh
CO2Me
Ph
(R)
Minor product
HN
MeO2C
O
S
P
Rh
*
H
P
Ph
MeO2C
Ph
H
N
CH3
O
(S)
Major product
REDUCTIONS
+
MeO2C
H
NH
H
Ph
Rh
P
P
Free Energy
*
NH
38
CH3
O
+
CO2Me
H
H
Ph
+
CO2Me
HN
Rh
O
P
P *
P
Ph
H 3C
H 3C
Rh
*
P
O
minor complex
MeO2C
*
Ph
Rh
P
+
NH
P
CH3
O
major complex
Reaction Coordinate
Ph Ph
O
O
P
Ru
O
P
O
Ph Ph
ACR 1990, 23, 345.
BINAP
O
O
(BINAP)RuAc2,
H2 (100 atm)
O
O
O
R
O
O
50 °C, CH2Cl 2
(BINAP)RuAc2,
H2 (100 atm)
R
O
50 °C, CH2Cl 2
(95 - 98 % ee)
(94 % ee)
CO 2H
Ru(AcO)2(BINAP),
H2 (135 atm), MeOH
(97% ee)
MeO
CO 2H
97 % ee
MeO
(S)-naproxen
MeO
MeO
MeO
(BINAP)RuAc2,
H2 (4 atm)
NAc
MeO
MeO
NAc
NH
MeO
OMe
OMe
(95 - 100 % ee)
OMe
OMe
OMe
OMe
Tetrahydropapaverine
Directed Asymmetric Hydrogenation
(BINAP)Ru (II), H2
OH
CHO
OH
(96 - 99 % ee)
OH
OH
HO
J. Am. Chem. Soc. 1987, 109, 1596
O
Vitamin E
REDUCTIONS
39
Kinetic Resolution by Directed Hydrogenation
CF 3SO 3-
Rh+
MeO Ph
P
P
Ph
CO 2Me
MeO 2C
H2 (15 psi), L*Rh+
Et
Ph
OMe
CO 2Me
MeO 2C
0 °C
(~60% conversion)
R
R=
OMe
R
> 96 % ee
82 %
93 %
Hydrogenation of Carbonyls
1,3-diketones:
O
R1
O
O
R1
OH
H2
O
H
OH
OH
R2
R2
R1
O
H
O
H
syn
anti
Ru2Cl 4(BINAP)
Et3N, H2 (100 atm)
MeO 2C
99 : 1
16 : 1 (90 % ee)
32 : 1
49 : 1
R1
R2
R1
R2
Directed
Reduction
M
O
R1
anti : syn=
OH
R1
O
R2
OH
syn
anti
H
O
M
OH
R2
-CH3
-CH2CH3
-iPr
-CH2CH3
R2
R2
+
R1
R2=
OH
R1
OH
R2
-CH3
-CH3
-CH2CH3
-CH2CH3
O
R2
Ru (II)
H2 (700 psi)
O
R1
O
R1
R2
O
R1=
OH
O
OH
JACS 1988, 110 , 6210
MeO 2C
(98% ee)
Decarbonylations
O
(Ph3P)3RhCl
O
(Ph3P)3RhCl
R H
R
H
- CO
R Cl
R
Cl
- CO
REDUCTIONS
40
OHC
(Ph3P)3RhCl
Fe
Fe
Fe
PhCH 3, ∆
JOC 1990, 55, 3688
Fe
Diimide
HN=NH
Review:
Organic Reactions 1991, 40J. Chem. Ed. 1965, 254
- Only reduces double bonds
- Syn addition of H 2
- will selectivley reduce the more strained double bond
- Unstable reagent which is generated in situ
K+O2C-N=N-CO2K+ + AcOH → H-N=N-H
H2N-NH2 + Cu2+ + H2O2 → H-N=N-H
HN=NH
ACIEE 1965, 271
(76%)
O
O
+-
CO 2MeK
O 2C N N CO 2- K+
CO 2Me
AcOH, MeOH
(95%)
NO 2
JACS 1986, 108 , 5908
NO 2
O
HN
HN
hν (254 nm)
S
NH
NH
+ COS + CO
O
O
S
O
N N
H H
TL 1993, 34, 4137
hν, 16 hr
(96%)
Metal Hydrides
Review on Metal Hydride Selectivity:
Chem Soc Rev. 1976, 5 , 23
Comprehensive Organic Synthesis 1991, vol 8, 1.
Boron Hydrides
Review:
Chem. Rev. 1986, 86 , 763.
NaBH4
reduces ketones and aldehydes
LiBH4
reduces ketones, aldehydes, esters and epoxides. THF soluble
LiBH4/TMSCl
stronger reducing agent. ACIEE 1989, 28, 218.
Zn(BH4)2
reduces ketones and aldehydes
R4N BH4
organic soluble (CH2Cl2) borohydrides. Synth Commun. 1990, 20, 907
LiEt3BH
reduces ketones, aldehydes, esters, epoxides and R-X
Li s-Bu3BH
reduces ketones, aldehydes, esters and epoxides (hindered borohydride)
Na(CN)NH3
reduces iminium ions, ketones and aldehydes
Na(AcO)3BH
reduces ketones and aldehydes (less reactive)
NaBH2S3
reduces ketones and aldehydes
REDUCTIONS
41
Sodium Borohydride NaBH4
- reduces aldehydes and ketones to alcohols
- does not react with acids, esters, lactones, epoxides or nitriles.
- Additives can increase reactivity.
Sodium Cyanoborohydride Na (CN)BH3
Reviews: Synthesis 1975, 136; OPPI 1979, 11 , 201
- less reactive than NaBH4
- used in reductive aminations (Borch Reduction)
Na(CN)BH3 reduces iminium ions much more quickly than ketones or aldehydes
R"-2NH, MeOH,
AcO - NH4+ , pH~ 8
R
O
R
R'
O
CHO
R
N
R'
JACS 1971, 93 , 2897
R"-2NH, MeOH,
AcO- NH4+
N
H
Na(CN)BH3
N
R'
R'
R
R
R H
Na(CN)BH3
N+
N
- Related to Eschweiler-Clark Reaction
H2CO, HCO 2H
R NH 2
R
or
N
H
Me
H2CO, H2/Pd
- Reduction of tosylhydrazones gives saturated hydrocarbon
O
H
H
1) TsNHNH 2, H+
2) Na(CN)BH3
TL 1978, 1991
(90%)
H
HO
H
HO
1) TsNHNH 2, H+
2) Na(CN)BH3
O
(100%)
JOC 1977, 42 , 3157
- migration of the olefin occurs w/ α,β-unsaturated ketones
O
JACS 1978, 100 , 7352
(75%)
- Epoxide opening
O
OH
NaBH3CN,
BF3•OEt2, THF
OH
HO
JOC 1994, 59, 4004
NaBH2S3
REDUCTIONS
Synthesis 1972, 526 Can. J. Chem. 1970, 48 , 735.
Lalancette Reduction
42
NaBH4/ NiCl 2
Chem. Pharm. Bull. 1981, 29 , 1159; Chem. Ber. 1984, 117 , 856.
Ar-NO2 → Ar-NH2
Ar-NO → Ar- NH2
R2C=N-OH → R2CH-NH2
NaBH4 / TiCl 4
Synthesis 1980, 695.
R-COOH → R-CH2-OH
R-COOR' → R-CH2-OH
R-CN → R-CH2-NH2
R-CONH2 → R-CH2-NH2
R2C=N-OH → R2CH-NH2
R-SO2-R' → R-S-R'
NaBH4 / CeCl 3
Luche Reduction
reduced α,β-unsaturated ketones in a 1,2-fashion
OH
NaBH 4/ CeCl3
R
R
NaBH 4
R
R
O
OH
O
R
+
R
R
R
R
R
R
H
R
JCSCC 1978, 601
JACS 1978, 100 , 2226
O
OH
CO 2Me
NaBH 4/ CeCl3
C 5H11
CO 2Me
MeOH
C 5H11
OH
OH
- selective reduction of ketones in the presence of aldehydes.
OH
O
CO 2Me
CO 2Me
NaBH 4/ CeCl3
EtOH, H2O
CHO
CHO
O
CeCl 3
H2O
1) NaBH 4, CeCl3
2) work-up
R
JACS 1979, 101 , 5848
OH
OH
O
OH
CHO
NaBH 4/ CeCl3
EtOH, H2O
(78%)
CHO
REDUCTIONS
Zinc Borohydride
Zn(BH4)2
Synlett 1993, 885.
ZnCl2 (ether) + NaBH 4 → Zn(BH4)2
- Ether solution of Zn(BH4)2 is neutral- good for base sensitive compounds
- Chelation contol model
Zn
RO
O
R1
H
O
OR
OH
H B
H
H
R2
OR
R1
R2
H
R1
R2
Zn(BH4)2,
Et2O, 0°C
OH
OH
OH
O
H
H-
Me
TL 1983, 24 , 2653, 2657, 2661
O
O
Zn
R
Na + (AcO)3BH , Me4N + (AcO)3BH
Review:
OPPI 1985, 17 , 317
- used in Borch reductive amination
TL 1990, 31 , 5595; Synlett 1990, 537
- selective reduction of aldehydes in the presence of ketones
O
Bu4N (AcO) 3BH
C 6H6, ↑↓
O
OH
CHO
(77%)
TL 1983, 24 , 4287
AcO
O
Bu4N (AcO) 3BH
C 6H6, ↑↓
CHO
Ph
H
O
OAc
B
OH
O
Ph
Ph
OH
-hydroxyl-directed reduction of ketones
TL 1983, 24 , 273; TL 1984, 25 , 5449
OH
O
Me
O
OH
Ph
N
Me
Me
OH
Me4N (AcO) 3BH,
CH 3CN, AcOH, −40°C
Me
O
Ph
N
Me
O
O
Me
O
O
TL 1986, 27 , 5939
JACS 1988, 110 , 3560
(98:2)
OAc
H
O
R
B
O
OAc
OAc
H
R
R
H
OAc
H
O
R
minor
major
OH
B
O
O
Na+ BH(OAc)3
OH
OH
50 : 1
43
REDUCTIONS
OH
O
O
OH
Na+ BH(OAc)3
OH
O
CO2R
CO2R
OH
Na+ BH(OAc)3
44
OH
OH
CO2R
HO
OBn
BnO
OBn
BnO
Me
MeO
OBn
O
OBn
Me4N (AcO) 3BH,
CH 3CN, AcOH, −40°C
O
Me
MeO
Me
O
MeO
O
O
Me
Me
OH
O
N
OH
O
Me
O
O
HO
OH
OH
Me
FK-506
OH
OH
TL 1989, 30 , 1037
(Ph3P)2Cu BH4
reduction of acid chlorides to aldehydes
reduction of alkyl and aryl azides to amines
JOC 1989, 45, 3449
J. Chem. Res. (S) 1981, 17
R4N BH4 organic soluble borohydride (CH2Cl2)
R4N= BnEt3N or Bu4N
Heterocylces 1980, 14, 1437, 1441
reduction of amides to amines
reduction of nitriles to amines
BnEt3N BH4 / Me 3SiCl
reduction of carbolxylic acids to alcohols
LiBH4/ Me 3SiCl
Synth. Commun. 1990, 20, 907
ACIEE 1989, 28, 218.
Alkyl Borohydrides
Selectrides
M + HB
3
M + = Li (L-selectride)
K (K-selectride)
LS-selectride
Li + HB
3
- hindered reducing agent
increased selectivity based on steric considerations
CO 2H
O
CO 2H
OH
L-selectride
THF
JACS 1971, 93, 1491
R
R
HO
HO
OH
OH
REDUCTIONS
45
- selective 1,4-reductions of α,β-unsaturated carbonyl cmpds.
JOC 1975, 40 , 146; JOC 1976, 41 , 2194
O
O
K-selectride, THF
(99%)
- 1,4-reduction generates an enolate which can be subsequently alkylated.
O
O
a) K-selectride, THF
b)
Br
K+ HBPh3
Syn. Comm. 1988, 18 , 89.
- even greater 1,4-selectivity
Li + HBEt3 (Super Hydride)
- very reactive hydride source
- reduces ketones, aldehydes, esters, epoxides and C-X (alkyl halides and sulfonates)
O
HO
Li Et3BH, THF
HO
HCA 1983, 66 , 760
HO
H
H
HO
HO
CH 3
OH
HO
1) TsCl, pyridine
2) Li Et3BH, THF
HCA 1988, 71 , 872
HO
H
H
Boranes
Hydroboration
H2O 2, NaOH
B 2H6
B
R
R'
R
B 2H6
R'
B
R
B 2H6
H
HO H
H
H3O +
H
R
B
H2O 2, NaOH
R
R'
H
H
R-CH 2CHO
H
- BH3 reduces carboxylic acids to 1° alcohols in the presence of esters, nitro and cyano groups.
- BH3 reduces amides to amines
HO 2C
BH 3•SMe 2
O
O
THF
HO
O
O
- Boranes also reduce ketones and aldehydes to the corresponding alcohols.
REDUCTIONS
Hindered Boranes
Disiamyl Borane (Sia2BH)
B
H
Thexyl Borane
B
H
H
B
9-BNN
=
B
H
B
H
O
Catecholborane
BH
O
BH
BH
Pinylborane
2
2
B
B
H
Alpine Borane
BCl
BCl
IPC 2BCl (DIP-Cl)
2
2
B
H
Borolane
Ph
Oxazaborolidine
N B
H
Ph
O
Asymmetric Reduction of Unsymmetrical Ketones Using Chiral Boron Reagents
Review:
Synthesis 1992, 605.
Alpine Borane
Midland Reduction
JACS 1979, 111 , 2352;
JACS 1980, 112 , 867
review:
Chem. Rev. 1989, 89 , 1553.
O
alpine-borane
THF, 0°C
OH
Tetrahedron 1984, 40 , 1371
(94% ee)
46
REDUCTIONS
B
H
B
=
9-BBN
B
B
α-pinene
9-BBN
B
H
Mechanism:
OH
O
RL
Rs
RL
Rs
- works best for aryl- and acetylenic ketones
- because of steric hindrance, alpine-borane is fairly unreactive
Chloro Diisopinylcamphenylborane (DIP-Cl, Ipc2BCl)
H.C. Brown
Review:
ACR 1992, 25 , 16. Aldrichimica Acta 1994, 27 (2), 43
BCl
2
- Cl increases the Lewis acidity of boron making it a more reactive reagent
- saturated ketones are reduced to chiral alcohols with varying degrees of ee.
O
OH
I
CO2tBu
Ipc2B-Cl,
THF
I
CO2tBu
JOC 1992, 57, 7044
PrO
PrO
OMe
(90 % ee)
OMe
Borolane
(Masamune's Reagent)
JACS 1986, 108 , 7404; JACS 1985, 107, 4549
B
H
B
O
OH
H
MeSO 3H, pentane
(80% ee)
Asymmetric Hydroboration:
a)
B
H
b) H2O 2, NaOH
OH
(99.5% ee)
(99.5% ee)
47
REDUCTIONS
Oxazaborolidine
(Corey)
JACS 1987, 109 , 7925; TL 1990, 31, 611l ; TL 1992, 33 , 4141
Ph
Ph
O
N B
Ph
N B
H
Ph
O
+ BH3
CH 3
Catalytic
O
OH
R
R
> 90 % ee
OH
O
92 % ee
O
OH
I
86 % ee
I
O
OH
93 % ee
Ph Ph
O
O
N B
Me
OH
BH 3•THF, -0°C
TL 1988, 29 , 6409
(90% ee)
CF 3
O
OH
Cl
O
Cl
• HCl
N
H
CH 3
94 % ee
Fluoxetine (Prozac)
O
O
O
O
90 : 10
BzO
BzO
O
Aluminium Hydrides
1. LiAlH 4
2. AlH3
3. Li (tBuO)3AlH
4. (iBu)2AlH
DIBAL-H
5. Na (MeOCH2CH2O)2AlH2
REDAL
OH
48
REDUCTIONS
49
Chem. Rev. 1986, 86, 763
Org. Rxn. 1951, 6, 469.
Lithium Aluminium Hydride LiAlH4 (LAH)
- very powerful reducing agent
- used as a suspension in ether or THF
- Reduces carbonyl, carboxylic acids and esters to alcohols
- Reduces nitrile, amides and aryl nitro groups to amines
- opens epoxides
- reduces C-X bonds to C-H
- reduces acetylenic alcohols trans-allylic alcohols
LAH
OH
R
OH
R
H2N
LAH, THF
N
H
NH 2
H2N
(62%)
NH 2
N
H
NH 2
Lindlar/ H2
H2N
O
N
H
HO
CO 2Me
OH
LAH, THF, ↑↓
TL 1988, 29 , 2793.
(100%)
O
O
H
O
H
O
BINAL-H
(Noyori)
- Chiral aluminium hydride for the asymmetric reduction of prochiral ketones
OH
OH
1) LiAlH4
2) ROH
H
O
Li +
Al
O
OR
R= Me, Et, CF 3CH 2-
BINOL
BINAL-H
O
O
O
BINAL-H,THF
Tetrahedron 1990, 46 , 4809
-100 to -78°C
HO
(94% ee)
Intermediate for 3-Component Coupling Strategy to Prostaglandins
I
O
Li+ O CO 2Me
RO
Li+ RCu
RO
OTBS
O
OTBS
O
CO 2H
CO 2Me
HO
RO
OTBS
OH
PGE 2
REDUCTIONS
Alane
50
AlH3
LiAlH 4 + AlCl3 → AlH3
- superior to LAH for the 1,2-reduction of α,β-unsaturated carbonyls to allylic alcohols
OMe
Ph
1) AlH3, ether, 0°C
2) H3O +
O
O
HO
O Me
O
JACS 1989, 111 , 6649
Me
O
OH
O
Diisobutyl Aluminium Hydride
DIBAL or DIBAL-H
Al
H
- Reduces ketones and aldehydes to alcohols
- reduces lactones to hemi-acetals
O
Al
O
OH
CHO
work up
DIBAL
O
(CH 2)n
O
OH
O
(CH 2)n
(CH 2)n
(CH 2)n
lactol
(stable complex)
- reduces esters to alcohols
- under carefully controlled reactions conditions, will partially reduce an ester to an aldehyde
Al
O
DIBAL
R CO 2Me
if complex
is unstable
R C OMe
fast
R CHO
R CH 2 OH
H
if complex
is stable
R CHO
O
O
O
OBn O
DIBAL, CH 2Cl 2
O
HO
OH
HO
HO
H
O
HO
OBn O
OBn
OH
OBn
OH
JACS 1990, 112 , 9648
OBn
OBn
DIBAL, CH 2Cl 2
CO 2iPr
iPrO2C
OHC
OBn
O
R
OBn
O
TMS-Cl, Et3N
OH
CHO
CH2Cl2
R
O
DiBAl-H
OSiMe3
CH2Cl2, -78°C
TL 1998, 39, 909
R
H
Reduction of O-Methyl hydroxamic acids
O
R
O
R'-M
R'
R
O
OMe
N
Me
TL 1981, 22 , 3815
DIBAL or LAH
R
H
REDUCTIONS
51
Sodium Bis(2-Methoxyethoxy)Aluminium Hydride
REDAL
Organic Reactions 1988, 36, 249
Organic Reactions 1985, 36, 1.
MeO
MeO
Na+
H
O
Al
O
H
- "Chelation" directed opening fo allylic epoxides
OH
O
REDAL
R
OH
DIBAL-H
R
R
OH
Ph
1,2-diol
OH
REDAL
O
OH
Ph
TL 1982, 23 , 2719
OH
1,3-diol
Sharpless
epoxidation
OH
OH
DME
LiAlH4
AlH3
OH
OH
+
OH
O
JOC 1988, 53 , 4081
Ph
2 : 98
95 : 5
OH
O
OH
BnO
+
OH
BnO
OH
BnO
OH
REDAL
DIBAL
O
OH
Me
HO
OH
O
O
Me
OH
OH
150 : 1
1 : 13
OH
OH
Me
OH
HO
O
OH
Me
Amphotericin B
HO
O
NH 2
Me
Me
O
Me
O
OH
Me
OH
O
O
O
sugar
sugar
Me
Erythromycin A
Li+ (tBuO)3AlH
Lithium Tri(t-Butoxy)aluminium Hydride
- hindered aluminium hydride, will only react with the most reactive FG's
O
R
Li(tBuO)3AlH
Cl
R
H
Me
Cl
O
N+
R-CO 2H
O
Me
H
R
pyridine, -30°C
O
H
Me
N+
Me
O
Li(tBuO)3AlH
CuI (cat), -78°C
R
TL 1983, 24 , 1543
H
Meerwein-Ponndorf-Verley Reduction: opposite of Oppenauer oxidation
Synthesis 1994, 1007
Organic Reactions 1944, 2, 178
O
H
O
H
Al(O-Pr)3, iPrOH
O
O
O AcHN
O
O AcHN
OH
REDUCTIONS
52
Asymmetric M-P-V Reduction
Bn
Ph
X
O
Ph
N
O
Sm
X
O
OH
I
JACS 1993, 115, 9800
iPrOH, THF
X= H, Cl, OMe
yield: 83-100 %
96% ee
Dissolving Metal Reductions
Birch Reductions
reduction of aromatic rings
Organic Reactions 1976, 23, 1.
Tetrahedron 1986, 42, 6354. Comprehensice Organic Synthesis 1991, vol. 8, 107.
- Li, Na or K metal in liquid ammonia
H H
H H
M, NH3
_•
•
R
R
R
R
H
H H
- position of the double bond in the final product is dependent of the nature of the substituent
R
R
R
R= EWG
R= ERG
- ketones and nitro groups are also reduced but esters and nitrile are not.
- α,β-unsaturated carbonyl cmpds are reduced in a 1,4-fashion to give an enolate which can be
subsequently used to trap electrophiles
Me
HO
Me
Me
O
O
K, NH3,
MeOH, -78°C
Me
HO
O
JOC 1991, 56 , 6255
O
(92%)
O
Me
Me
OH
Me
Me
O O
O O
a) Li, NH3, tBuOH
b) CH 2O
JOC 1984, 59 , 3685
O
O
HO
Other Metals
- Mg
Mg, MeOH
X
X
X- CN, CO2R, CONR'2
H
H
Mg, MeOH
EtO2C
(98%)
TL 1987, 28 , 5287
EtO2C
- Zn reduction of α-halocarbonyls
O
O
Br
Zn, PhH,
DMSO, MeI
Me
JACS 1967, 89, 5727
REDUCTIONS
Cl
R
Zn
O
X
Y
Zn, AcOH
R
R'
X= Cl, Br, I
Y= X, OH
R
R-OH
O
CH
53
CH R'
"Copper Hydrides"
LAH or DIBAL-H + MeCu → "CuH"
- selective 1,4-reduction of α,β-unsaturated ketones (even hindered enones)
O
O
1) MeCu, DIBAL,
HMPA, THF, -50°C
2) MeLi
3)
JOC 1987,52 , 439
Br
O
O
MeCu, DIBAL, HMPA
THF, -50°C
H
H
JOC 1986,51 , 537
(85%)
O
H
H
O
H
[(Ph3P)CuH]6
Stryker Reagent
JACS 1988, 110 , 291 ; TL 1988, 29 , 3749
- 1,4-reduction of α,β-unsaturated ketones and esters; saturated ketones are not reduced
- halides and sulfonates are not reduced
- 1,4-reduction gives an intermediate enolate which can be trapped with electrophiles.
O
Br
O
[(Ph3P)CuH] 6, THF
TL 1990, 31 , 3237
Silyl Hydrides
- Hydrosilylation
Et3SiH + (Ph3P)3RhCl (cat)
- selective 1,4-reduction of enones, 1,2-reduction of saturated ketones to alchohols.
O
Et3SiH,
(Ph3P)3RhCl (cat)
O SiEt3
H3O +
O
TL 1972, 5085
J. Organomet. Chem. 1975, 94 , 449
O
O
O
iPr3SiH, Et2O
Si O
2
Pt
2
OSi(iPr)3
O
JOC 1994, 59, 2287
O
(87%)
- Buchwald Reduction
JACS 1991, 113 , 5093
- catalytic reagent prepared from Cp2TiCl2 + nBuLi and stoichometric (Et)3SiH in THF will
reduce ester, ketones and aldehydes to alcohols under very mild conditions.
- α,b− unsaturated esters are reduced to allylic alcohols
- free hydroxyl groups, aliphatic halides and epoxides are not reduced
REDUCTIONS
54
Clemmensen Reduction
Organic Reactions 1975, 22, 401
Comprehensive Organic Synthesis 1991, vol 8, 307.
- reduction of ketones to saturated hydrocarbons
Zn(Hg), HCl
H
O
Wolff-Kishner Reduction
H
Organic Reactions 1948, 4, 378
Comprehensive Organic Synthesis 1991, vol. 8, 327.
- reduction of ketones to saturated hydrocarbons
H2N-NH2, KOH
H
O
H
Radical Deoxygenation
Review:
Tetrahedron 1983, 39 , 2609
Chem. Rev. 1989, 89, 1413.
Comprehensive Organic Synthesis 1991, vol. 8, 811
Tetrahedron 1992, 48, 2529
W. B. Motherwell, D. Crich Free Radical Chain Reactions in Organic Synthesis
(Academic Press: 1992)
- free radical reduction of halide, thio ethers, xanthates, thionocarbanates by a radical chain
mechanism.
nBu3Sn-H, AIBN
Ph-CH3 ↑↓
R3C-H
R3C-X
S
X= -Cl, -Br, -I, -SPh,
S
S
Ph , O
O
Barton-McCombie Reduction
JCS P1 1975, 1574
R3C-X → R3C-H
N N
CN
SMe , O
CN
AIBN
N
N
X= -OC(=S)-SMe, -OC(=S)-Im, -OC(=S)Ph
O
O
O
O
HO
NaH, imidazole,
THF, CS2, MeI
O
O
O
O
O
O
O
nBu3SnH, AIBN
PhCH3, reflux
O
O
O
O
(85%)
O
SMe
S
Xanthate
R
R
Cl
a) Ph
nBu3SnH, AIBN
PhCH3, reflux
NMe2 , THF
+
S
b) H2S, pyridine
HO
(73%)
Ph
(90%)
R
O
Thionobenzoates
S
Ph
O
O
O
HO
AcHN
OBn
N
N
N
(CH2Cl)2
59%
Ph
N
N
N
O
O
O
O
AcHN
OBn
S
Thiocarbonyl Imidazolides
nBu3SnH, AIBN
PhCH3, reflux
(57%)
Ph
O
O
AcHN
O
OBn
REDUCTIONS
- Cyclic Thionocarbonates: deoxygenation of 1,2- and 1,3-diols to alcohols
55
S
OH
N
O
HO
MeO
MeO
S
N
O
O
MeO
N
N
HO
O
JCS P1 1977, 1718
MeO
(61%)
MeO
OMe
nBu3SnH, AIBN
PhCH3, reflux
O
MeO
OMe
OMe
- Thionocarbonate Modification (Robbins)
JACS 1981, 103 , 932; JACS 1983, 105 , 4059.
iPr
iPr
O
O
Si
PhOC(S)Cl,
pyridine, DMAP
B
iPr
iPr
O
nBu3SnH, AIBN
PhCH3, reflux
B
O
O
> 90%
Si O
iPr iPr
O
Si
OH
iPr
iPr
O
B
O
O
(58-78%)
Si O
iPr iPr
O
Si
Si O
iPr iPr
OPh
S
S
OAr
O
O
O
CH3
nBu3SnH,
AIBN, PhCH3
O
O
O
Tetrahedron 1991, 47, 8969
O
88-91%
O
O
O
O
Best method for deoxygenation
of primary alcohols
Ar= 2,4,6-trichlorophenyl,
4-fluorophenyl
- N-Phenyl Thionocarbamates
iPr
iPr
O
O
Si
U
PhNCS
NaH, THF
S
O
Si O
iPr iPr
Tetrahedron 1994, 34, 10193.
iPr
iPr
O
U
Si O
iPr iPr
NHPh
(TMS)3SiH, AIBN
PhH, reflux
S
O
(78%)
O
O
Si
O
(93%)
iPr
iPr
O
O
Si
U
O
Si O
iPr iPr
NHPh
Thiocarbamates
- Methyl oxylates
OAc
O
OAc
O
MeO2CCOCl
THF
OC
nBu3SnH, AIBN
PhCH3, reflux
OC
O
HO
CO2Me
CO2Me
MeO2C
O
OAc
O
OC
CO2Me
Methyl Oxylate Esters
- Water Soluble Tin Hydride: [MeO(CH2)2O(CH2)3]3SnH / 4,4'-Azo(bis-4-cyanovaleric acid)
TL 1990, 31 , 2957
- Silyl Hydride Radical Reducing Agents
- replacement for nBu3SnH
(Me3Si)3SiH
Chem Rev. 1995, 95, 1229.
JOC 1991, 56 , 678; JOC 1988, 53 , 3641; JACS 1987, 109 , 5267
Ph2SiH2 / Et3B / Air
TL 1990, 31 , 4681; TL 1991, 32 , 2569
- hypophosphorous acid as radical chain carrier
JOC 1993, 58, 6838
REDUCTIONS
- Photosensitized electron transfer deoxygenation of m-trifluoromethylbenzoates
JACS 1986, 108, 3115, JOC 1996, 61, 6092, JOC 1997, 62, 8257
iPr
iPr
O
O
Si
U
N-methylcarbazole,
hν, iPrOH, H2O
O
Si O
iPr iPr
O
iPr
iPr
(85%)
O
Si
O
CF3
O
U
O
Si O
iPr iPr
Dissolving Metal : JACS 1972, 94, 5098
O
OH
O
nBuLi,
(Me2N)2P(O)Cl
O
P(NMe2)2
Li, EtNH2,
THF, tBuOH
O
O
H
CH3
O
O
H
O
Radical Decarboxylation: Barton esters
Aldrichimica Acta 1987, 20 (2), 35
S
hn or ∆
N
S
R
N
nBu3SnH, ∆
O
O
R
- CO2
Radical Deamination
Comprehensive Organic Synthesis 1991, vol. 8, 811
Reduction of Nitroalkanes
JOC 1998, 63, 5296
NO2
Bu3SnH, PhSiH3
O
O
initiator, PhCH3 (reflux)
(75%)
O
O
R H
H
56
PROTECTING GROUPS
Carey & Sundberg Chapter 13.1 problems # 1; 2; 3a, b, c ;
Smith: Chapter 7
57
Protecting Groups
T.W. Greene & P.G.M. Wuts, Protective Groups in Organic Synthesis (2nd edition) J.
Wiley & Sons, 1991.
P. J. Kocienski, Protecting Groups, Georg Thieme Verlag, 1994
1.
2
3.
4.
Hydroxyl groups
Ketones and aldehydes
Amines
Carboxylic Acids
- Protect functional groups which may be incompatible with a set of reaction
conditions
- 2 step process- must be efficient
- Selectivity
a. selective protection
b. selective deprotection
Hydroxyl Protecting Groups
Ethers
Methyl ethers
R-OH → R-OMe
Formation: Cleavage: -
difficult to remove except for on phenols
CH2N2, silica or HBF4
NaH, MeI, THF
AlBr3, EtSH
PhSe Ph2P Me3SiI
OMe
O
OH
O
AlBr3, EtSH
TL 1987, 28 , 3659
O
O
OBz
Methoxymethyl ether MOM
R-OH → R-OCH2OMe
OBz
stable to base and mild acid
Formation: - MeOCH2Cl, NaH, THF
- MeOCH2Cl, CH2Cl2, iPr2EtN
Cleavage
- Me2BBr2
TL 1983, 24 , 3969
PROTECTING GROUPS
Methoxyethoxymethyl ethers (MEM)
R-OH → R-OCH2OCH2CH2OMe
stable to base and mild acid
Formation: - MeOCH2CH2OCH2Cl, NaH, THF
- MeOCH2CH2OCH2Cl, CH2Cl2, iPr2EtN
Cleavage
- Lewis acids such as ZnBr2, TiCl4, Me2BBr2
TL 1976, 809
S
B Cl
MEM-O
HO
S
C5H11
O-Si(Ph)2tBu
O
C5H11
TL 1983, 24 , 3965, 3969
O
O-Si(Ph)2tBu
- can also be cleaved in the presence of THP ethers
Methyl Thiomethyl Ethers (MTM)
R-OH → R-OCH2SMe
Stable to base and mild acid
Formation: - MeSCH2Cl, NaH, THF
Cleavage: - HgCl2, CH3CN/H2O
- AgNO3, THF, H2O, base
Benzyloxymethyl Ethers
(BOM)
R-OH → R-OCH2OCH2Ph
Stable to acid and base
Formation: - PhOCH2CH2Cl, CH2Cl2, iPr2EtN
Cleavage: - H2/ PtO2
- Na/ NH3, EtOH
Tetrahydropyranyl Ether
R-OH
(THP)
O
H
+
, PhH
Formation
Cleavage:
R
O
O
Stable to base, acid labile
- DHP (dihydropyran), pTSA, PhH
- AcOH, THF, H2O
- Amberlyst H-15, MeOH
Ethoxyethyl ethers (EE)
JACS 1979, 101 , 7104; JACS 1974, 96 , 4745.
R-OH
O
H+
R
Benzyl Ethers
(R-OBn)
R-OH → R-OCH2Ph
Formation:
Cleavage:
O
O
(R-OEE)
base stable, acid labile
stable to acid and base
- KH, THF, PhCH2Cl
- PhCH2OC(=NH)CCl3, F3CSO3H
- H2 / PtO2
- Li / NH3
JCS P1 1985, 2247
58
PROTECTING GROUPS
2-Napthylmethyl Ethers (NAP)
JOC 1998, 63, 4172
formation: 2-chloromethylnapthalene, KH
cleavage: hydrogenolysis
OH
O
ONAP
BnO
OH
H2, Pd/C
-
OH
BnO
(86%)
p- Methoxybenzyl Ethers
(PMB)
Formation:
- KH, THF, p-MeOPhCH2Cl
- p-MeOPhCH2OC(=NH)CCl3, F3CSO3H
Cleavage:
O
59
TL 1988, 29 , 4139
H2 / PtO2
Li / NH3
DDQ
Ce(NH4)2(NO3)6 (CAN)
e-
o-Nitrobenzyl ethers
Review: Synthesis 1980, 1; Organic Photochemistry, 1987, 9 , 225
O 2N
NaH, THF
R-OH
Cl
R
O
NO 2
Cleavage:
- photolysis at 320 nm
NO 2
HO
HO
HO
O
HO
hν, 320 nm,
pyrex, H2O
O
HO
HO
OH
O
OH
JOC 1972, 37 , 2281, 2282.
OH
p-Nitrobenzyl Ether
TL 1990, 31 , 389
-selective removal with DDQ, hydrogenolysis or elctrochemically
9-Phenylxanthyl- (pixyl, px)
TL 1998, 39, 1653
Formation:
Ph
Ph
Cl
OR
pyridine
ROH
+
O
Removal:
Ph
O
Ph
OR
OH
hν (300 nm)
ROH
O
CH3CN,H 2O
+
O
Trityl Ethers
-CPh3 = Tr
R-OH → R-OCPh3
- selective for 1° alcohols
- removed with mild acid; base stable
formation: - Ph3C-Cl, pyridine, DMAP
- Ph3C + BF4Cleavage: - mild acid
PROTECTING GROUPS
60
Methoxytrityl Ethers
JACS 1962, 84 , 430
- methoxy group(s) make it easier to remove
R1
(p-Methoxyphenyl)diphenylmethyl ether
4'-methoxytrityl
MMTr-OR
R2
Di-(p-methoxyphenyl)phenylmethyl ether
4',4'-dimethoxytrityl
DMTr-OR
C O R
Tri-(p-methoxyphenyl)methyl ether
4',4',4'-trimethoxytrityl TMTr-OR
R3
Tr-OR < MMTr-OR < DMTr-OR << TMTr-OR
O
O
HN
R O
HN
O
80% AcOH (aq)
N
O
HO
O
20°C
N
O
HO OH
HO OH
R = Tr 48 hr.
R= MMTr
2 hr.
R= DMTr
15 min.
R= TMTr
1 min.
(too labile to be useful)
Oligonucleotide Synthesis (phosphoramidite method - Lessinger)
Review: Tetrahedron 1992, 48 , 2223
S
I
L
I
C
A
S
I
L
I
C
A
OH
OH
OH
S
I
L
I
C
A
O
Si (CH 2)3 NH 2
O
O
DMTrO
O
O
DMTrO
O
B
O
Cl 3CCOOH
S
O
B'
O
P
O
CN
O
CN
O
DMTrO
O
O
Base
O
B
O
P
-
O
B
O
coupling
S
DMTrO
O
O
HO
B'
O
S
B'
O
O
O
I2, H2O
Si (CH 2)3 N C (CH 2)2 C OH
H
HO
B'
N (iPr)2
O
B
Si (CH 2)3 N C (CH 2)2 C O
H
O
O
O
P
O
O
B'
O
O
O
S
I
L
I
C
A
DMTrO
O
P
OO-
Cl 3CCOOH
O
B
O
P
O
B
O
O
S
O
Repeat Cycle
OO
S
O
PROTECTING GROUPS
Silyl Ethers
Synthesis 1985, 817 Synthesis 1993, 11 Synthesis 1996, 1031
R-OH → R-O-SiR3
formation: - R3Si-Cl, pyridine, DMAP
- R3Si-Cl, CH2Cl2 (DMF, CH3CN), imidazole, DMAP
- R3Si-OTf, iPr2EtN, CH2Cl2
61
Trimethylsilyl ethers
Me3Si-OR
TMS-OR
- very acid and water labile
- useful for transiant protection
Triethylsilyl ethers
Et3Si-OR
TES-OR
- considerably more stable that TMS
- can be selectively removed in the presence of more robust silyl ethers with with F - or
mild acid
O
OH
H2O/ACOH/THF
(3:5:11), 15 hr
TESO
OTBS
O
(97%)
Liebigs Ann. Chem. 1986, 1281
OTBS
Triisopropylsilyl ethers
iPr3Si-OR
TIPS-OR
- more stabile to hydrolysis than TMS
Phenyldimethylsilyl ethers
J. Org. Chem. 1987, 52 , 165
t-Butyldimethylsilyl Ether tBuMe 2Si-OR
TBS-OR
TBDMS-OR
JACS 1972, 94 , 6190
- Stable to base and mild acid
- under controlled condition is selective for 1° alcohols
t-butyldimethylsilyl triflate
tBuMe 2Si-OTf
TL 1981, 22 , 3455
- very reactive silylating reagent, will silylate 2° alcohols
cleavage:
- acid
- F- (HF, nBu4NF, CsF, KF)
TBSO
HO
CO2Me
HF, CH3CN
CO2Me
(70%)
O
OTBS
O
HO
JCS Perkin Trans. 1 1981, 2055
t-Butyldiphenylsilyl Ether tBuPh2Si-OR
TBDPS-OR ∑-OR
- stable to acid and base
- selective for 1° alcohols
- Me3Si- and iPr 3Si groups can be selectively removed in the presence of TBS or TBDPS
groups.
- TBS can be selectively removed in the presence of TBDPS by acid hydrolysis.
TL 1989, 30 , 19
PROTECTING GROUPS
cleavage
- F- Fluoride sources: -
nBu 4NF (basic reagent)
HF / H2O /CH3CN
HF•pyridine
SiF4. CH2Cl2
TL 1979, 3981.
Synthesis 1986, 453
TL 1992, 33 , 2289
Me
O Si
OH
tBu
Me
JOC 1981, 46 ,1506
TL 1989, 30 , 19.
AcOH / THF/ H2O
Ph
O Si tBu
O Si
Me
Ph
tBu
Ph
Ph
Me
tBu Si
62
Me
tBu
O
Si
Me
OTHP
O
OH
PPTS / EtOH
JACS 1984, 106 , 3748
Esters
R-OH → R-O2CR'
Formation: - "activated acid", base, solvent, (DMAP)
Activated Acids Chem. Soc. Rev. 1983, 12, 129 Angew. Chem. Int. Ed. Engl. 1978, 17, 569.
RCO2H → "activated acid" → carboxylic acid derivative (ester, amide, etc.)
Acid Chlorides
O
O
O
O
N
N
+
N
R
R
OH
R
R
N
Cl
+
N
N
acyl pyridinium ion
(more reactive)
1. SOCl2
2. PCl5
3. (COCl)2
Anhydrides
O
O
P2O 5
2
R
OH
R
O
O
R
Activating Agents:
Carbonyl Diimidazole
O
O
O
R
OH
N
+
N
R
N
N
N
N
Acyl Imidazole
NH
+
CO
+
N
PROTECTING GROUPS
63
Dicyclohexylcarbodiimide
C 6H11
O
NH
O
R
+
OH
R
N C N
O
O
Nu:
O C
N
R
+C 6H11
Nu
N
H
N
H
C 6H11
C 6H11
Ketene formation is a common side reaction- scambling of chiral centers
C 6H11
O
NH
R
R
O C
N
H
C O
"ketene"
C 6H11
Hydroxybenzotriazole (HOBT) - reduces ketene formation
C 6H11
O
R
O
N
NH
+
O C
N
N
N
O N
R
N
N
OH
C 6H11
N-Hydroxysuccinimide (NHS)
O
C 6H11
O
R
O
O
NH
HO
+
O C
N
R
N
O N
O
C 6H11
O
2,2'-Dipyridyl Disulfide (Aldrithiol, Corey Reagent)
Aldrichimica Acta 1971, 4 , 33
O
R
OH
O
Ph3P:
+
N
S S
R
N
+
S
+
N
N
Ph3P=O
SH
Mukaiyama's Reagent (2-Chloro-1-methyl pyridinium Iodide or 2-Fluoro-1methyl pyridinium p-toulenesulfonate)
Aldrichimica Acta 1987, 20 , 54
Chem. Lett. 1975, 1045; 1159; 1976, 49; 1977, 575
O
+
R
OH
F
+
N
Me
O
TsO R
O
+
N
I-
Me
Acetates
R-OH → R-O2CCH3
- stable to acid and mild base
- not compatable with strong base or strong nucleophiles such as organometallic
reagents
Formation: - acetic anhydride, pyridine
- acetyl chloride, pyridine
PROTECTING GROUPS
Cleavage:
-
K2CO3, MeOH, reflux
KCN, EtOH, reflux
NH3, MeOH
LiOH, THF, H2O
enzymatic hydrolysis (Lipase)
OAc
64
Org. Rxns. 1989, 37, 1.
OAc
Porcine Pancreatic
Lipase
TL 1988, 30 , 6189
OAc
OH
(96% ee)
Chloroacetates
- can be selectively cleaved with Zn dust or thiourea.
O Me
O OR
O
HO
AcO
Me
OAc
O
Cl
Me
O OR
Cl
H2NNHCOSH
O
AcO
Me
O
JCS CC 1987, 1026
O
OAc
O
HO
Cl
O
O
OH
O
Trifluoroacetates
Formation: - with trifluoroacetic anhydride or trifluoroacetyl chloride
Cleavage: - K2CO3, MeOH
Pivaloate
(t-butyl ester)
- Fairly selective for primary alcohols
Formation: - tbutylacetyl chloride or t-butylacetic anhydride
Cleavage: - removed with mild base
Benzoate
(Bz)
- more stable to hydrolysis than acetates.
Formation: - benzoyl chloride, benzoic anhydride, benzoyl cyanide (TL 1971, 185) ,
benzoyl tetrazole (TL 1997, 38, 8811)
Cleavage: - mild base
- KCN, MeOH, reflux
1,2 and 1,3- Diols
Synthesis 1981, 501
Chem. Rev. 1974, 74, 581
R2
O
OH
R1
R
OH
Isopropylidenes
R3
H+ , -H2O
R2
R3
O
O
R
R1
(acetonides)
H+
OH
R
R1
OH
acetone or
OMe
MeO OMe
or
Me
Me
O
O
R
R1
- in competition between 1,2- and 1,3-diols, 1,2-acetonide formation is usually favored
- cleaved with mild aqueous acid
PROTECTING GROUPS
65
Cycloalkylidene Ketals
- Cyclopentylidene are slightly easier to cleave than acetonides
- Cyclohexylidenes are slightly harder to cleave than acetonides
O
(CH 2)n -or-
OH
OMe
MeO
(CH 2)n
(CH 2)n
R1
R
O
H+ , -H2O
OH
O
R
R1
Benzylidene Acetals
PhCHO
-orPhCH(OMe)2
OH
R1
R
Ph
O
O
+
H , -H2O
OH
R
R1
- in competition between 1,2- and 1,3-diols, 1,3-benzylidene formation for is usually
favored
- benzylidenes can be removed by acid hydrolysis or hydrogenolysis
- benzylidene are usually hydrogenolyzed more slowly than benzyl ethers or olefins.
p-Methoxybenzylidenes
- hydrolyzed about 10X faster than regular benzylidenes
- Can be oxidatively removed with Ce(NH 4)2(NO3)6 (CAN)
OMe
OBn
BnO
O
MeO
BnO
OH
(95%)
O
O
OBn
Ce(NH 4)2(NO3)6
CH 3CN, H2O
MeO
O
OH
Other Reactions of Benzylidenes
- Reaction with NBS
(Hanessian Reaction)
H
O
Ph
O
O
HO
NBS, CCl 4
O
Br
Ph
HO
OMe
O
HO
Org. Syn. 1987, 65, 243
O
HO
OMe
- if benzylidene of a 1° alcohol, then 1° bromide
- Reductive Cleavage
Ph
O
Na(CN)BH3,
TiCl4,CH 3CN
O
OH
MeO 2C
MeO 2C
O
MeO
CO 2Me
O
O
Ph
O
CO 2Me
Synthesis 1988, 373.
OBn
O
TMS-CN
BF3•OEt2
OH
Tetrahedron 1985, 41, 3867
MeO
O
O
H
Ph
CN
PROTECTING GROUPS
Ph
O
O
BnO
OH
DIBAL-H
TL 1988, 29 , 4085
O
O
OMe
OMe
Carbonates
O
OH
R1
R
(Im)2CO
O
OH
O
R
R1
- stable to acid; removed with base
- more difficult to hydrolyze than esters
Di-t-Butylsilylene
(DTBS)
TL 1981, 22 , 4999
- used for 1,3- and 1,4-diols; 1,2-diols are rapidly hydrolyzed
- cleaved with fluoride (HF, CH 3CN -or- Bu4NF -or- HF•pyridine)
- will not fuctionalize a 3°-alcohol
OH
(t-Bu)2SiCl 2, Et3N
CH 3CN, HOBT
O
tBu
Si
O
OH
1,3-(1,1,3,3)-tetraisopropyldisiloxanylidene (TIPDS)
- specific for 1,3- and 1,4-diols
- cleaved with fluoride or TMS-I
tBu
TL 1988, 29 , 1561
O
O
HN
HN
HO
O
N
iPr2Si(Cl)-O-Si(Cl)iPr2
pyridine
O
O
Si
O
N
O
O
Si
HO OH
O OH
Ketones and Aldehydes
- ketones and aldehydes are protected as cyclic and acyclic ketals and acetals
- Stable to base; removed with H3O+
R
O
MeOH, H+
R1
R
R1
(CH 2OH)2, H+
PhH, -H2O
-or(CH2OSiMe 3)2,
TMS-OTf, CH2Cl 2
CH 2(CH 2OH)2,
H+ , PhH, -H2O
OMe
OMe
R
O
TL 1980, 21 , 1357
R1 O
1,3-dioxolanes
R O
R1 O
1,3-dioxanes
66
PROTECTING GROUPS
67
Cleavage rate of substituted 1,3-dioxanes:
Chem. Rev. 1967, 67 , 427.
R O
R O
R O
>
R1 O
>>
R1 O
R1 O
- Ketal formation of α,β-unsaturated carbonyls are usually slower than for the
saturated case.
O
O O
CH 2(CH 2OH)2,
H+ , PhH, -H2O
O
O
Fluoride cleavable ketal:
O
O
LiBF4
O
O
(88%)
O
Me3Si
TL 1997, 38, 1873
O
O
Base cleavable ketal:
HO
O
SO2Ph
SO2Ph
DBU, CH2Cl 2
OH
R1
R2
O
pTSA, C6H6
R1
O
TL 1998, 39, 2401
O
R1
R2
R2
Carboxylic Acids
Tetrahedron 1980, 36, 2409. Tetrahedron 1993, 49, 3691
Nucelophilic Ester Cleavage:
Organic Reactions 1976, 24, 187.
Esters
Alkyl Esters
formation: - Fisher esterification (RCOOH +R'OH + H+)
- Acid Chloride + R-OH, pyridine
- t-butyl esters: isobutylene and acid
- methyl esters: diazomethane
Cleavage: - LiOH, THF, H2O
- enzymatic hydrolysis
Org. Rxns. 1989, 37, 1.
- t-butyl esters are cleaved with aqueous acid
- Bu 2SnO, PhH, reflux (TL 1991, 32, 4239)
OH
MeO 2C
CO 2Me
O
R
Pig Liver Esterase
pH 6.8 buffer
Bu2SnO, PhH, ↑↓
OR'
OH
MeO 2C
CO 2H
O
R
OH
TL 1991, 32, 4239
R= Me, Et, tBu
9-Fluorenylmethyl Esters
(Fm)
TL 1983, 24 , 281
- cleaved with mild base (Et2NH, piperidine)
DCC
RCO 2H
+
O
OH
R
O
PROTECTING GROUPS
2-Trimethylsilyl)ethoxymethyl Ester
(SEM)
HCA 1977, 60 , 2711.
- Cleaved with Bu 4NF in DMF
DCC
RCO 2H
HO
+
O
R
SiMe 3
O
O
SiMe 3
O
- Cleaved with MgBr2•OEt2 TL 1991, 32, 3099.
2-(Trimethylsilyl)ethyl Esters
JACS 1984, 106 , 3030
- cleaved with Fluoride ion
DCC
RCO 2H
HO
+
R
O
SiMe 3
SiMe 3
O
Haloesters
- cleaved with Zn(0) dust or electrochemically
DCC
RCO 2H
HO
+
CCl 3
R
O
CCl 3
O
Benzyl Esters
RCO2H + PhCH2OH → RCO2Bn
Formation: Cleavage: -
DCC
Acid chloride and benzyl alcohol
Hydrogenolysis
Na, NH3
Diphenylmethyl Esters
DCC
RCO 2H
HO
+
R
O
CHPh 2
CHPh 2
O
Cleavage:
- mild H3O+
- H2, Pd/C
- BF3•OEt2
o-Nitrobenzyl Esters
- selective removed by photolysis
Orthoesters
Synthesis 1974, 153
TL 1983, 24 , 5571
Chem. Soc. Rev. 1987, 75
O
RCOCl
O
BF 3•OEt2
+
R
OH
- Stable to base; cleaved with mild acid
O
R
O
O
O
O
68
PROTECTING GROUPS
69
Amines
Carbamates
9-Fluorenylmethyl Carbamate
(Fmoc)
Acc. Chem. Res. 1987, 20 , 401
- Cleaved with mild base such as piperidine, morpholine or dicyclohexylamine
NaHCO 3
H2O, dioxane
+
R2NH
O
O
Cl
O
R2N
O
2,2,2-Trichloroethyl Carbamate
O
O
Cl 3C
O
R2NH, pyridine
Cl 3C
Cl
O
N
R
R
- Cleaved with zinc dust or electrochemically.
O
N
O
CCl 3
S
Te Te S
TL 1986, 27 , 4687
NaBH 4
EtO2C
NH
EtO2C
S
S
2-Trimethylsilylethyl Carbamate
(Teoc)
- cleaved with fluoride ion.
O
O
Me 3Si
O
O
+
O N
Me 3Si
R2NH
O
N
R
R
O
SiMe 3
OH
Cl
O
MeO
OTBS
O
N
Cl
Bu4NF, THF
O
CH 3
H
N
MeO
O
CH 3
(100%)
H
O
SEt
H
O
SEt
OEt
OEt
SEt
SEt
JACS 1979, 101 7104
t-Butyl Carbamate
(BOC)
O
O
O
R2NH
Cleavage:
tBuO
O
OtBu
R2N
OtBu
- with strong protic acid (3M HCl, CF3COOH)
- TMS-I
O
B Cl
Allyl Carbamate
(Alloc)
O
TL 1985, 26 , 1411
TL 1986, 27 , 3753
PROTECTING GROUPS
O
O
R2NH
O
O
O
O
R2N
O
- removed with Pd(0) and a reducing agent (Bu3SnH, Et 3SiH, HCO2H)
O
HN
1) Pd(OAc) 2, Et3N, Et3SiH
2) H3O +
O
CO 2Me
Benzyl Carbanate
NH 2
TL 1986, 27 , 3753
CO 2Me
(Cbz)
O
BnO
O
Cl
R2NH
Cleavage:
-
R2N
O
Ph
Hydrogenolysis
PdCl 2, Et3SiH
TMS-I
BBr3
hν (254 nm)
Na/ NH3
m-Nitrophenyl Carbamate
JOC 1974, 39 , 192
NO 2
O
R2N
O
- removed by photolysis
Amides
Formamides
- removed with strong acid
R2NH
+
O
HCO 2Et
R2N
H
Acetamides
- removed with strong acid
R2NH
+
O
Ac2O
R2N
Trifluoroacetamides
Cleavage: - base (K2CO3, MeOH, reflux)
- NH3, MeOH
R2NH
Sulfonamides
p-Toluenesulfonyl
+
O
(CF3CO) 2O
R2N
(Ts)
R2NH
pTsCl, pyridine
R2N
CH 3
SO 2
CF 3
70
PROTECTING GROUPS
Cleavage:
Ts
- Strong acid
- sodium Naphthalide
- Na(Hg)
N
N
Na(Hg), MeOH
Na2HPO 4
Ts
H
N
N
H
JOC 1989, 54 , 2992
(65%)
N
N
Ts
H
Trifluoromethanesulfonyl
Tf
Tf
N
N
N
NH
Na, NH3
HN
JOC 1992, 33, 5505
NH
N
HN
Tf
Tf
Trimethylsilylethanesulfonamide
(SES)
TL 1986, 54 , 2990; JOC 1988, 53, 4143
- removed with CsF, DMF, 95°C
SO 2Cl
Me 3Si
R2NH
Et3N, DMF
R2N
O
SiMe 3
S
O
tert-Butylsulfonyl (Bus) JOC 1997, 62, 8604
R-NH2
tBuSOCl,
Et3N, CH2Cl2
mCPBA
-orRuCl3, NaIO4
O
R 2N
S
tBu
R2N SO2tBu
CF3SO3H,
CH2Cl2, anisole
R-NH2
71
C-C BOND FORMATION
72
Carbon- Carbon Bond Formation
1.
Alkylation of enolates, enamines and hydrazones
C&S: Chapt. 1, 2.1, 2.2 problems Ch 1: 1; 2; 3, 7; 8a-d; 9; 14 Ch. 2: 1; 2; 4)
Smith: Chapt. 9
2.
Alkylation of heteroatom stabilized anions C&S :Chapt. 2.4 - 2.6)
3.
Umpolung Smith: Chapt. 8.6
4.
Organometallic Reagents
C&S: Chapt. 7, 8, 9 problems ch 7: 1; 2; 3, 6; 13 Ch. 8: 1; 2
Smith: Chapt. 8
5.
Sigmatropic Rearrangements . C&S Chapt. 6.5, 6.6, 6.7 # 1e,f,h,op
Smith Chapt. 11.12, 11.13
Enolates
Comprehensive Organic Synthesis 1991, vol. 2, 99.
- α-deprotonation of a ketone, aldehyde or ester by treatment with a strong nonnucleophillic base.
- carbonyl group stabilizes the resulting negative charge.
O
H
H
O-
O
B:
H -
R
H
H
R
H
R
H
- Base is chosen so as to favor enolate formation. Acidity of C-H bond must be greater
(lower pKa value) than that of the conjugate acid of the base (C&S table 1.1, pg 3)
O
H 3C
O
H 3C
CH3
pKa = 20
unfavorable enolate
concentration
MeO- pKa = 15
tBuO- pKa = 19
more favorable
enolate concentration
O
CH2
OEt
pKa = 10
- Common bases: NaH, EtONa, tBuOK, NaNH2, LiNiPr2, M N(SiMe3)2,
Na CH2S(O)CH3
Enolate Formation:
- H+ Catalyzed (thermodynamic)
O
OH
H+
- Base induced (thermodynamic or kinetic)
O
:B
O-
+
B:H
H
Regioselective Enolate Formation
Tetrahedron 1976, 32, 2979.
- Kinetic enolate- deprotonation of the most accessable proton (relative rates of
deprotonation). Reaction done under essentially irreversible conditions.
O - Li+
O
LDA, THF, -78°C
C-C BOND FORMATION
typical conditions: strong hindered (non-nucleophilic) base such as LDA
R2NH pKa= ~30
73
Li
N
Ester Enolates- Esters are susceptible to substitution by the base, even LDA can be
problematic. Use very hindered non-nucleophillic base (Li isopropylcyclohexyl amide)
O
O
OR'
LDA, THF, -78°C
N
E+
R
R
O
O- Li+
N
Li
OR'
R
OR'
THF, -78°C
R
- Thermodynamic Enolate- Reversible deprotonation to give the most stable enolate:
more highly substituted C=C of the enol form
O - K+
O - K+
O
tBuO- K+,
tBuOH
kinetic
thermodynamic
typical conditions: RO- M+ in ROH , protic solvent allows reversible enolate
formation. Enolate in small concentration (pKa of ROH= 15-18 range)
- note: the kinetic and thermodynamic enolate in some cases may be the same
- for α,β-unsaturated ketones
O
thermodynamic
site
kinetic
site
Trapping of Kinetic Enolates
- enol acetates
1) NaH, DME
2) Ac2O
Ph
O
Ph
+
Ph
O
O
kinetic
O
isolatable
separate & purify
CH3Li, THF
Regiochemically
pure enolates
O
CH3Li, THF
Ph
Ph
O- Li+
O- Li+
- silyl enolethers
Synthesis 1977, 91.
1) LDA
2) Me3SiCl
Ph
O
C-C BOND FORMATION
Acc. Chem. Res. 1985, 18, 181.
Ph
+
Ph
OTMS
OTMS
kinetic
isolatable
separate & purify
CH3Li, THF
-orBu4NF -or- TiCl4
Ph
Ph
Geometrically
pure enolates
CH3Li, THF
O- M+
O- M+
- tetraalkylammonium enolates- "naked" enolates
- TMS silyl enol ethers are labile: can also use Et3Si-, iPr3Si- etc.
- Silyl enol ether formation with R 3SiCl+ Et3N gives thermodyanamic silyl
enol ether
- From Enones
1) MeLi
2) E+
1) Li, NH3
2) TMS-Cl
O
O
TMSO
OSiMe3
H
H
E
O
OSiMe3
TMS-Cl, Et3N
TMS-OTf
Et3N
O
OSiMe3
Li, NH3, tBuOH
TMS-Cl
- From conjugate (1,4-) additions
O
O- Li+
O
E+
(CH3)2CuLi
E
Trap or use directly
- From reduction of α-halo carbonyls
O
Br
Zn or Mg
O- M+
Alkylation of Enolates (condensation of enolates with alkyl halides and epoxides)
Comprehensive Organic Synthesis 1991, vol. 3, 1.
1° alkyl halides, allylic and benzylic halides work well
2° alkyl halides can be troublesome
3° alkyl halides don't work
74
C-C BOND FORMATION
O
75
O
a) LDA, THF, -78°C
b) MeI
Me
- Rate of alkylation is increased in more polar solvents (or addition of additive)
O
(Me2N)3P
R
NMe2
R= H DMF
R-CH3 DMA
HMPA
O
O
O
S
H 3C
O
CH3N
CH3
CH3N
NMe2
NCH3
Me2N
DMSO
TMEDA
Mechanism of Enolate Alkylation: SN2 reaction, inversion of electrophile stereochemistry
X
C
180 °
M+ -O
Alkylation of 4-t-butylcyclohexanone:
O
O
R
E
R
equitorial anchor
E
H
H
A
E
tBu
favored
A
Chair
tBu
R
R
B
O
O- M+
H
O
E
tBu
B
Twist Boat
R
E
on cyclohexanone enolates, the electrophile approaches from an "axial" trajectory. This
approach leads directly into a chair-like product. "Equitorial apprach leads to a higher
energy twist-boat conformation.
Alkylation of α,β-unsaturated carbonyls
O- M+
R1
O
R2
Kinetic
R1
E
O
R1
H
R2
E
H
R2
H
H
O- M+
R1
O
R2
H
E
R1
R2
H
Thermodynamic
E
C-C BOND FORMATION
76
Stork-Danheiser Enone Transposition:
- overall γ-alkylation of an α,β-unsaturated ketone
O
O
LDA
PhCH2OCH2Cl
HO CH3
CH3Li
PhO
OMe
H3O
PhO
CH3
+
PhO
OMe
OMe
O
J. Org. Chem. 1995, 60, 7837.
Chiral enolates- Chiral auxilaries.
D.A. Evans JACS 1982, 104 , 1737; Aldrichimica Acta 1982,15 , 23.
Asymmetric Synthesis 1984, 3, 1.
- N-Acyl oxazolidinones
O
O
R
H 2N
OH
Me
Ph
O
N
Me
Ph
norephedrine
O
O
H 2N
R
OH
O
N
valinol
O
O
R
R
LDA, THF
O
N
O
O
N
O
LiOH, H2O, THF
O
R
OH
Et-I
Me
Ph
O
O
R
N
O
Complimentary Methods
for enantiospecific alkylations
Me
Ph
major product
(96:4)
O
O
LDA, THF
R
N
O
LiOH, H2O, THF
O
R
OH
Et-I
Diastereoselectivity: 92 - 98 %
for most alkyl halides
major product
(96:4)
Enolate Oxidation Chem. Rev. 1992, 92, 919.
R
O
O
O
N
O
NaN(SiMe3)2,
THF, -78°C
R
N
N
(88 - 98 % de)
SO2Ph
O
LDA, THF
R
O
tBuO
O
OH
O
Ph
O
Boc
N
N
OtBu
O
O
N
N
HN
Boc
O
1) HO2) CH2N2
3) TFA
4) Raney Ni
(94 - 98 % de)
O
R
OMe
NH2
C-C BOND FORMATION
O
R
Bu
Bu
O
B
N
O
Bu2BOTf, Et3N
O
O
N
N
O
N
1) LiOH
2) H2, Pd/C
O
O
R
N3
R
D- amino acids
O
O
N
R
KN(SiMe3)2, THF
O
OH
NH2
Ph
O
O
Ph
Ph
O
O
N3-
Br
Ph
R
O
O
NBS
R
R
77
O
N
O
N3
SO2N3
Ph
Ph
Oppolzer Camphor based auxillaries
Tetrahedron, 1987, 43, 1969.
diastereoselectivities on the order of 50 : 1
SO2Ph
N
O
R
Ar
Ar
N
O
R
O
SO2Ph
O
R
N
O
SO2N(C6H11)2
O
S
O2
R
H
O
H
LDA, NBS
Et2Cu•BF3
O
O
SO2N(C6H11)2
O
H
O
Br
O
SO2N(C6H11)2
HO
O
SO2N(C6H11)2
Asymmetric Acetate Aldol
O
S
O
N
O
TIPSO
H
1) Br
Sn(OTf)2, CH2Cl 2,
R3N, -40°C
2) TIPS-OTf, pyridine
3) NH3
O
Br
NH2
J. Am. Chem. Soc. 1998, 120, 591
J. Org. Chem. 1986, 51, 2391
85 %, 19:1 de
Chiral lithium amide basess
CH3
MeO
CO2Et
OMe
Ph
N
Li
THF, -78°C
(CH3)2C=O
CH3
OMe
MeO
O
OMe O
NH2
O
(72% ee)
C-C BOND FORMATION
H
N
O
Ph
N
H
But
O
N
Li
Li
N
(97 % ee)
THF, HMPA
TMSCl
tBu
OTMS
Ph
N
tBu
N
Me
Lewis Acid Mediated Alkylation of Silyl Enolethers- SN1 like alkylations
OTMS
O
tBu-Cl, TiCl4,
CH2Cl2, -40°C
CH3
(79%)
SPh
OTMS
R
note: alkylation with a
3° alkyl halide
C(CH3)3
O
Cl
SPh
ACIEE1978, 17, 48
TL 1979, 1427
O
Raney Ni
R
R
(95 %)
TiCl4, CH2Cl2, -40°C
(78%)
Enamines
Gilbert Stork
Tetrahedron 1982, 38, 1975, 3363.
- Advantages: mono-alkylation, usually gives product from kinetic enolization
O
O
N
N
"Thermodynamic"
"Kinetic"
O
O
N
H
can not become coplanar
O
O
••
N
+
N
R-I
R
H2O
O
E
H+, (-H2O)
enamine
-Chiral enamines
O
N
Imines
E
Isoelectronic with ketones
Me
O
Ph
N
Li
OMe
N
LDA, THF, -20°C
Ph
1) E
2) H3O+
O
E
E = -CH3, -Et, Pr,
PhCH2-, allylee 87 - 99 %
78
Hydrazones
C-C BOND FORMATION
isoelectronic with ketones Comprehensive Organic Synthesis 1991, 2, 503
O
N
N
N
Me2N-NH2
-N
N
LDA, THF
79
N
-
+
H , (-H2O)
N
E+
N
O
hydrolysis
E
E
- Hydrazone anions are more reactive than the corresponding ketone or aldehyde
enolate.
- Drawback: can be difficult to hydrolyze.
- Chiral hydrazones for asymmetric alkylations (RAMP/SAMP hydrazones- D. Enders
"Asymmetric Synthesis" vol 3, chapt 4, Academic Press; 1983)
OMe
MeO
N
N
H 2N
SAMP
RAMP
N
LDA
OMe
N
N
NH2
O
O3
OMe
N
H
I
OTBS
(95 % de)
TBSO
TBSO
N
1) LDA
2) Ts-CH3, THF
-95 - -20 °C
OMe 3) MeI, 2N HCl
N
O
CH3
(100 % ee)
Me
O
Li
R1
E (C,C)
MeO
••
N N
R2
H
R1
N N
R2
Z (C,N)
H
E
E
Aldol Condensation
Comprehensive Organic Synthesis 1991, 2, 133, 181.
O
H
R
a) LDA, THF, -78°C
b R'CHO
O
β-hydroxyl aldehyde
(aldol)
OH
H
R'
R
- The effects of the counterion on the reactivity of the enolates can be important
Reactivity Li+ < Na+ < K+ < R4N+
addition of crown ethers
C-C BOND FORMATION
- The aldol reaction is an equilibrium which can be "driven" to completion.
M
O- M+
O
+
R
RCHO
O
H
R'
O
work-up
OH
H
R'
80
R'
R
R
In the case of hindered enolates, the equillibrium favors reactants. Mg2+ and Zn2+
counterions will stabilize the intermediate β-alkoxycarbonyl and push the equillibrium
towards products. (JACS 1973, 95, 3310)
O
O- M+
OH
PhCHO, THF
M= Li
M= MgBr
Ph
16% yield
93% yield
- Dehydration of the intermediate β-alkoxy- or β-hydroxy ketone can also serve to drive
the reaction to the right.
O
O
O
tBuO- Na +, tBuOH
O
JACS 1979, 101 , 1330
O
H
O
H
O
Enolate Geometry
- two possible enolate geometries
O - Li+
O - Li+
O
LDA, THF, -78°C
H
+
H
Z - enolate
E - enolate
- enolate geometry plays a major role in stereoselection.
OM
Z -enolate
R
R2
1
O
R3CHO
R
R
OM
R
H
1
3
erythro
(syn)
R2
H
E -enolate
OH
1
O
R3CHO
R
OH
1
R
R2
R
3
threo
(anti)
2
- Zimmerman-Traxler Transition State : Ivanov condensation
JACS 1957, 79 , 1920.
+
O
-
Ph
H
+
MgBr
- +
PHCHCO2 MgBr
Br
H
O
Ph
O
Mg
Ph
H
"pericyclic" T.S.
OMgBr
C-C BOND FORMATION
81
Analysis of Z-enolate stereoselectivity
R2
O
R3
O
M
R2
R2 O
R3
M
O
R3
O
M
O
O
H
H
R1
H
R1
H
H
R3
R1
H
R1
OH
R2
erythro (syn)
favored
R2
O
H
R2 O
H
R2
M
O
M
R1
R3
O
M
O
R3
R1
R3
R1
OH
R1
H
R3
H
H
O
H
O
R2
threo (anti)
disfavored
Analysis of E-enolate stereoselectivity
R3
H
O
R3
M
R
O
3
O
R2
H
R1
H
O
O
M
R2
H
H
R2
H
R
OH
R1
R3
R2
threo (anti)
favored
H
H
O
O
R1
H
R1
O
M
O
M
H
H
O
M
O
H
O
2
R1
R3
R2
R2
R3
R3
R1
O
M
O
OH
R1
R1
R3
R2
erthro (syn)
disfavored
Analysis of Boat Transition State for Z-Enolates
R2
O
R3
O
O
M
R3
H
R1
H
R3
R1
H
O
HO
O
M
R2
R2
R1
H
Favored Chair
Boat
H
O
R2
O
O
O
M
H
HO
R1
H
R3
R1
Disfavored Chair
R3
R3
R2
staggered
O
R2
R1
H
Boat: R1-R2
1,3-interaction is gone
M
C-C BOND FORMATION
82
Analysis of Boat Transition State for E-Enolates
R3
H
O
M
R3
O
O
O
HO
R1
R2
R1
H
H
R3
O
M
H
R2
R1
R2
Favored Chair
Boat
H
H
O
O
O
M
O
HO
R3
H
R1
R2
R3
O
M
H
R2
R1
R3
staggered
R1
R2
Disfavored Chair
Boat: R1-R2
1,3-interaction is gone
Summary of Aldol Transition State Analysis:
1. Enolate geometry (E- or Z-) is an important stereochemical aspect. Z-Enolates
usually give a higher degree of stereoselection than E-enolates.
2. Li+, Mg 2+, Al3+= enolates give comparable levels of diastereoselection for kinetic
aldol reactions.
3. Steric influences of enolate substituents (R1 & R2) play a dominent role in kinetic
diastereoselection.
O- M+
O
Path A
R2
R1
R1
R3
Path
B
H
R2
O- M+
O
H
R1
HO
R1
Path A
R2
HO
R3
R2
When R1 is the dominent steric influence, then path A proceeds. If R2 is the dominent
steric influence then path B proceeds.
4. The Zimmerman-Traxler like transition state model can involve either a chair or boat
geometry.
Noyori "Open" Transition State for non-Chelation Control Aldols
Absence of a binding counterion. Typical counter ions: R4N+, K+/18-C-6, Cp2Zr2+
- Non-chelation aldol reactions proceed via an "open" transition state to give syn aldols
regardless of enolate geometry.
Z- Enolates:
R1
O-
R1
Favored
H
H
R3
R2
R3
O-
R1
H
R3
O
R2
H
H
R2
HO
R1
Favored
R1
O-
O-
R3
R2
Syn Aldol
O
H
HO
R3
H H R R2
3
O
O
O
Disfavored
H
R3
H R2
O
O
R1
H
O-
R1
O-
R1
H
R2
O
Disfavored
R3
R2
Anti Aldol
C-C BOND FORMATION
83
E- Enolate:
-
O
-
R1
-
R1
O
O
favored
H
R3
R3
R2
H
R3
H
R1
-
O
H
R3
O
Syn Aldol
R1
R1
O
H
R2
HO
R3
H H R3 R2
R1
H
O
O
R3
R2
R2
O
favored
disfavored
H
R1
H
-
O
HO
H
H R2
O
O
-
R1
O
O
disfavored
R3
R2
R2
Anti Aldol
NMR Stereochemical Assignment.
Coupling constants (J) are a weighted average of various conformations.
H
O
O
R1
Syn Aldol
JAB = 2 - 6 Hz
HB
R3
R2 HA
60 °
60 °
HA
H
O
O
HB
HB
R3
R2
R2
O
R3
OH
R3
O
R1
R2
R1
HA
HA
H
R1
O
HB
non H-bonded
O
H
OH
B
R1
Anti Aldol
JAB = 1 - 10 Hz
R3
R2 HA
60 °
HA
H
O
O
R3
R3
OH
O
R2
R2
O
HB
HA
HB
HB
R1
R2
R1
60 °
HA
H
R1
O
R3
non H-bonded
Boron Enolates:
Comprehensive Organic Synthesis 1991, 2, 239. Organic Reactions 1995, 46, 1;
Organic Reactions 1997, 51, 1. OPPI 1994, 26, 3.
- Alkali & alkaline earth metal enolates tend to be aggregates- complicates
stereoselection models.
- Boron enolates are monomeric and homogeneous
- B-O and B-C bonds are shorter and stronger than the corresponding Li-O abd Li-C
bonds (more covalent character)- therefore tighter more organized transition state.
Generation of Boron Enolates:
O
R2B-X
iPrEtN
OBR2
X= OTf, I
R= Bu, 9-BBN
C-C BOND FORMATION
R3N:
_
H
R1
+ BL2OTf
O
R2
H
R3N:
OBL2
Z-enolate
OBEt2
_
H
+ BL2OTf
O
H
R1
R2
R2
R1
R1
R2
E-enolate
O
OBR2
R 3B
R
OSiMe3
OBR2
R2B-X
+ Me3 Si-X
O
OBR'2
R' 3B
N2
R
Hooz Reaction
R'
R
Diastereoselective Aldol Condensation with Boron Enolates
O
O
OBEt2
Ph
Ph
Ph
pure
Z-enolate
R2
R1
OH
R3CHO
R1
R3
R2
Z-enolate
OBEt2
O
R1
R3
R2
R2
generally
> 95 : 5
syn : anti
OH
R3CHO
R1
R
100% Syn Aldol
O
OBEt2
OBEt2
RCHO
generally
~ 75 : 25
anti : syn
E-enolate
Asymmetric Aldol Condansations with Chiral AuxilariesD.A. Evans et al. Topics in Stereochemistry, 1982, 13 , 1-115.
- Li+ enolates give poor selectivity (1:1)
- Boron and tin enolates give much improved selectivity
Bu
Bu
O
Me
B
O
N
Bu2BOTf,
EtNiPr2 , -78°
O
O - O +
N
O
OH
RCHO
R
O
O
N
Me
> 99:1 erythro
O
O
1) Bu2BOTf,
EtNiPr2 , -78°
Me
N
O
2) RCHO
Ph
OH
R
O
X
Me
O
84
C-C BOND FORMATION
L
L
B
_
O
H
+
O
R
L
L
B
_
O
O
O
N
RCHO
H
R
O
B
_
O
+
O
R
O
N
L
L
B
_
O
O
+
N
L
L
L
L
O
O
B
_
O
O
+
+
R
N
N
O
O
O
O
preferred conformation
R2
O
O
L
H
O
B
R3
L
R3
N
H
L
H
R3
O
B
N
L
O
O
O
O
Favored
O
O
R2
Disfavored
O
O
OH
N
O
R3
O
OH
N
R2
R3
R2
Oppolzer Sultam
L 2B
O
R2
N
O
R2
N
S
O2
S
O2
O
S
O
N
O
S
O2
OH
O
R3CHO
R2
R3
N
S
O2
R2
R3
N
1) LDA
2) Bu3SnCl
R3
Sn
OH
O
R3CHO
R2
85
C-C BOND FORMATION
Chiral Boron
BOTf
O
OH
StBu
O
Ph
OH
StBu
iPrEt2N,
PhCHO,-78°C
when large,
higher E-enolate
selectivity
O
ArO2SN
SPh
Ph
StBu
1 : 33
(> 99 % ee)
Ph
Ph
R
+
O
B
Br
NSO2Ar
OH
O
Ph
OH
SPh
+
Ph
SPh
R
iPrEt2N,
PhCHO,-78°C
O
R
> 95 : 5
(> 95 % ee)
• In general, syn aldol products are achievable with high selectivity, anti aldols are
more difficult
Mukaiyama-Aldol- Silyl Enol Ethers as an enolate precursors.
Lewis acid promoted condensation of silyl ketene acetals (ester enolate equiv.) with
aldehydes: proceeds via "open" transition state to give anti aldols starting from either
E- or Z- enolates.
OSiMe3
RCHO, TiCl4,
CH2Cl2, -78°C
OH
OH
CO2Et
R
OEt
+
CO2Et
R
CH3
CH3
R= iPr
(anti : syn) = 100 : 0
C6H11
94 : 6
Ph
75 : 25
OSiMe3
RCHO, TiCl4,
CH2Cl2, -78°C
OH
R
OEt
OH
CO2Et
+
CO2Et
R
CH3
CH3
R= iPr
(anti : syn) = 52 : 48
C6H11
63 : 37
Ph
67 : 33
Asymmetric Mukiayama Aldol:
Ph
H 3C
O
OSiMe3
NMe2
RCHO, TiCl4,
CH2Cl2, -78°C
OH
R
O
OH
Rc
+
R
O
Rc
(90-94% de)
syn : anti = 85 : 15
selectivity insenstivie to enolate geometry
86
C-C BOND FORMATION
Ph
N SO Ph
2
O
O
iPrCHO, TiCl4,
CH2Cl2, -78°C
HO
96 % de
anti : syn = 93 : 7
Rc
OSitBuMe2
CH3
O
RCHO, TiCl4,
CH2Cl2, -78°C
O
87
HO
+ Syn product
Rc
CH3
OSitBuMe2
SO2N(C6H11)2
E-Enolate
R= Ph
% de= 90
nPr
85
iPr
85
anti : syn = 91 : 19
94 : 6
98 : 2
Z-Enolate
R= iPr
% de= 87
anti : syn = 97 : 7
Mukaiyama-Johnson Aldol- Lewis acid promoted condensation of silyl enol ethers with acetals:
OSiMe3
OH
O
TiCl4 or SnCl4
Mukaiyama-Johnson Aldol
R
RCHO or RCH(OR')2
CH2Cl2, -78°C
via Ti or Sn enolate
O
O
TiCl4, CHCl2, -78 °C
O
O
O
HO
O
O
O
OTMS
+
Cl4Ti
O+
O
Cl4Ti
O
O
OSiMe3
OTMS
Ph
TiCl4,
(CH3)2C(OEt)2
(78 %)
O
OEt
Ph
Fluoride promoted alkylation of silyl enol ethers
Acc. Chem. Res. 1985, 18, 181
O
OSiMe3
nBu4NF, THF, MeI
C-C BOND FORMATION
88
Meyer's Oxazolines:
O
(ipc)2BOtf
iPrEt2N, Et2O
N
1) RCHO
2) 3N H2SO4
3) CH2N2
O
H 3C
CO2Me
OH
(ipc)2B
Ester
equiv.
R
+
CO2Me
(~ 30%)
N
H 3C
R
OH
R= nPr
%ee (anti) = 77 anti : syn = 91 : 9
C6H11
84
95 : 5
tBu
79
94 : 6
Anti-Aldols by Indirect Methods:
SePh
O
PhSe
C6H11
1) (C5H7)2BOTf
R3N
1) TBS-Cl
2) DiBAl-H
3) NaIO4
4) CH2N2
O
CO2Me
HO
CH3
O3
R
R
HO
Anti Aldol
Product
CHO
OTBS
CO2Me
OTBS
O
1) LDA, THF,
-78 °C
2) RCHO
N
OH
1) HIO6
2) CH2N2
O
N
MeO2C
R
CH3
R
CH3
Anti Aldol
O
MOMO
O
O
MeO
O
MOMO
1) LDA, THF,
-78 °C
N
O
R
syn
aldol
CH3
3) TsCl
4) Ba(CN)BH3
C6H11
HO
chiral
auxillary
CO2Me
R
R
2) RCHO
OTBS
1) HF
2) [O]
OTBS
N
N
KBEt3H, Et2O,
-78 °C
O
MeO
CH3
CH3
CH3
syn : anti
1 : 99
OMOM
CH3
OMOM
2) RCOCl
HO
O
MOMO
Zn(BH4)2
HO
N
CH3
syn : anti
97 : 3
CH3
OMOM
Syn Aldols by Indirect Methods:
O
O
O
N
O
1) LDA, THF,
-78 °C
O
O
O
Zn(BH4)2
O
2) RCOCl
O
N
R
CH3
O
OH
N
R
CH3
syn : anti = 100 : 1
C-C BOND FORMATION
Aldol Strategy to Erythromycin:
O
9
10
8
11
OH
12
13 O
1
O
3
2
1
6
OH
4
Erythromycin
seco acid
CO2H
5
15
14
4
7
OH
OH
O
OH
OH
3
OH
2
[O]
[O]
syn
aldol
Erythromycin
aglycone
3
CHO
CO2H
OH
O
syn
aldol
OH
syn
aldol
4
+
CHO
OH
1
CHO
CHO
O
+
CO2H
OH
syn
aldol
CHO
2
+
CHO
O
1
HO2C
O
O
O
O
LDA,
CH3CH 2COCl
N
O
O
OH
OH
O
OH
3
5
9
11
13
O
O
O
TiCl4, iPr 2EtN,
CH2Cl2
O
N 1
83%, (96:4)
O
N
90% (> 99:1)
O
O
1) 9-BBN, THF
2) Swern oxid.
3
5
73% (85:15)
O
O
O
5
O
O
Ph
O
CHO
N
Ph
Ph
O
1) Zn(BH3)2
2) (H3C)2C(OMe)2
CSA
OH
CHO
Ph
O
N
O
(100%)
Ph
O
OH
O
O
O
Sn(OTf)2,
Et3N, CH2Cl2
O
N
CH3CH 2CHO
O
O
OH
N 9
11
13
1) Na BH(OAc)3
2) TBS-OTf, 2,6-lutidine
3) AlMe3, (MeO)MeNH•HCl MeO
72% (>99:1)
84% (> 96:4)
Ph
O
OH
OTBS
11
13
EtMgBr
86%
8
Ph
1) PMBC(NH)CCl3
TfOH
2) (PhMe2Si)2NLi,
TMS-Cl
48 %
PMBO
TMSO
OTBS
O
N
CH3
OH
OTBS
89
C-C BOND FORMATION
X
L
H3C
H
Sn
H
O
L O
X
O
O
O
X
H
O
O
O
N
O
3
H3C
CH3
O
CHO
7
O
R
H
H
anti-syn
+
O
O
13
1) Zn(BH3)2
2) DDQ
O
O
OH
O
N
OTBS
O
O
O
OH
PMBO
O
3
5
7
9
13
70%
O
O
O
p-MeOC 6H4
O
OTBS
1) LiOOH
2) TBAF
O
N
O
O
O
O
OH
Cl3C6H2COCl
iPr2EtN, DMAP
HO
63%
Ph
11
OTBS
1) NaH, CS2, MeI
2) nBu3SnH, AIBN
N
Ph
p-MeOC 6H4
O
O
Ph
O
95%
O
O L
83% (95:5)
p-MeOC 6H4
O
L
Ti
O
J. Am. Chem. Soc.
1990,112, 866
L
H3C
CH3
BF3•OEt2,
CH 2Cl2, -78 °C
OTBS
11
Ph
O
H
CH3
CH3
X
CH3 CH3
8
O
Disfavored
O
H3C
H
OH
X
PMBO
TMSO
5
O
Disfavored
O
O
L
CH3
H
anti-syn
H
L
CH3
Sn
H
O
O L
O
Ti
CH3 CH3
H
CH3
CH3
X
L
R
H
O
L
OH
13
p-MeOC 6H4
(86%
O
9
O
9
10
1) Pd(OH)2, iPrOH
2) PCC
3) 1M HCl, THF
8
11
7
O
12
O
58 %
13
O
4
1
O
OH
5
6
5
13
11
OH
1
3
O
3
2
O
OH
O
Michael Addition
- 1,4-addition of an enolate to an α,β-unsaturated carbonyl to give 1,5-dicarbonyl
compounds
-
O
+
O
O
O M
Ph
R
Ph
R
Organometallic Reagents
Grignard reagents:
O
R-Br
Mg(0)
OH
R-MgBr
R
THF
O
O
OH
R-MgBr
THF
R
often a mixture of
1,2- and 1,4-addition
+
R
90
C-C BOND FORMATION
91
O
OH
R-MgBr
O
R-MgBr
1,2-addition
R
THF, CeCl3
O
1,4-addition
CuI,THF, -78C
R
Organolithium reagents
- usually gives 1,2-addition products
- alkyllithium are prepared from lithium metal and the corresponding alkyl halide
vinyl or aryl- lithium are prepared by metal-halogen exchange from the
corresponding vinyl or aryl- haidide or trialkyl tin with n-butyl, sec-butyl or tbutyllithium.
Li(0)
R-Br
R-Li
Et2 O
X
Li
nBu-Li
X= Br, I, Bu3Sn
Et2 O
Organocuprates
Reviews: Synthesis 1972, 63; Tetrahedron 1984, 40 , 641; Organic Reactions 1972, 19 , 1.
- selective 1,4-addition to α,β-unsaturated carbonyls
CuI, THF
2 R-Li
R2CuLi
O
O
R2CuLi
R
- curprate "wastes" one R group- use non transferable ligand
_
MeO
MeO
Cu
Li+
Cu R
R-Li
non-transferable
ligand
Other non transferable ligands
_
_
+
Bu3P Cu R
Li
Me2S Cu R
Li+
_
_
NC Cu R
Li
+
F3B Cu R
Li+
22Li
Cu R
S
Mixed Higher Order Cuprate
B. Lipshutz Tetrahedron 1984, 40 , 5005
Synthesis 1987, 325.
+
CN
Addition to Acetals
O R
CH3
n-C6 H13
(n-C6H13)2CuLi
H3C
O
Tetrahedron Asymmtetry 1990, 1, 477.
O
BF3•OEt 2
R
1) PCC
2 NaOEt
OH
LA
O
O
H
CH3
O
O
R
CH3
Nu:
H
CH3
OH
Chiral axulliary is destroyed 99 % ee
LA
R
n-C6 H13
O
H
R
O
Nu
TL 1984, 25, 3087
C-C BOND FORMATION
92
TMS
O
1) TiCl4
O
2) [O]
3) TsOH
JACS 1984, 106, 7588
OH
98 % ee
Stereoselective Addition to Aldehydes
- Aldehydes are "prochiral", thus addition of an organometallic reagent to an aldehydes
may be stereoselective.
- Cram's Rule JACS 1952, 74 , 2748; JACS 1959, 84 , 5828.
empirical rule
O
R
1
*
OH
-
M
S
1) "R2 - "
2) "H + "
R1
R
M
S
2
L
L
O
OH
M
S
M
R2
L R1
-
R
S
1
L
R
2
- Felkin-Ahn TL 1968, 2199; Nouv. J. Chim. 1977, 1 , 61.
based on ab initio calculations of preferred geometry of aldehyde which considers the
trajectory of the in coming nucleophile (Dunitz-Burgi trajectory).
O
L
R2
R
S
vs.
M
R2
-
L
-
S
1
O
M
better
R
1
worse
- Chelation Control Model- "Anti-Cram" selectivity
- When L is a group capable
of chelating a counterion such as alkoxide groups
+
M
OH
O
R
OR'
R1
*
S
M
2
R
1
OR' "Anti-Cram" Selectivity
M
S
M+
O OR'
HO
R2
M
R
1
-
M
S
OR'
R2
R1
S
Umpolung - reversal of polarity Aldrichimica Acta 1981, 14, 73; ACIIE 1979, 18, 239.
i.e: acyl anion equivalents are carbonyl nucleophiles (carbonyls are usually electophillic)
O
R
Benzoin Condensation
O-
KCN
PhCHO
Ph
H
CN
usually
-
R
O
+
Comprehensive Organic Synthesis 1991, 1, 541.
OH
Ph - CN
Cyanohydrin anion
PhCHO
HO
Ph
OCN
Ph
-O
Ph
CN
O
OH
Ph
Ph
OH
Ph
Benzoin
C-C BOND FORMATION
93
Thiamin pyrophosphate- natures acyl anion equivalent for trans ketolization reactions
H
NH2
NH2
_
+
N
N
N
H 3C
+
S
N
N
S
OPO3PO3
H 3C
H 3C
N
OPO3PO3
H 3C
Thiamin pyrophosphate
CHO
H
H 2C
OH
H
+
H
OH
H 2C
OPO3
thiamin-PP
O
HO
OH
H 2C
OH
H 2C
+
OPO3
OPO3
glyceraldehyde-3-P
(C3 aldose)
D-ribulose-5-P
(C5 ketose)
D-ribose-5-P
(C5 aldose)
H
OH
H
OH
CHO
O
OH
H
H 2C
OH
H
H
OH
H
OH
H
OH
H 2C
OPO3
sedohepulose-7-P
(C7 ketose)
Trimethylsilycyanohydrins
O
R
TMSO
TMS-CN
CN
R
H
LDA, THF
TMSO
H
CN
acyl anion
equivalent
_
R
O
NC
OMs
OEE
NaHMDS,
THF, -60°C
CSA, tBuOH
CN
O
Tetrahedron Lett. 1997, 38, 7471
(72%)
OEE
O
O
O
O
O
Dithianes
B:, THF
S
S
S
R
H
R
O
Hg(II)
R'-I
S
-
S
S
R
R'
R
R'
Aldehyde Hydrazones
B:
H
N
R
N
tBu
R
H
Heteroatom Stabilized Anions
Sulfones
R
LDA, THF
S
O
H
O
R
E
E
(Dithiane anion is an example)
O
_
Ph
tBu
N
E+
N
Ph
R
S
O
O
R'
R'
OH
R'
Al(Hg)
R'
R'
OH
R'
Ph
R
O
S
O
R
O
Sulfoxides
O
R'
_
Ph
R
S
O
LDA, THF
Ph
R
S
O
R'
R'
OH
Raney Ni
R'
Ph
R
R'
R'
S
O
R
OH
C-C BOND FORMATION
Epoxide Opening
94
Asymmetric Synthesis 1984, 5, 216.
Basic (SN2) Condition
Nu:
R
R
Nu
Steric Approach Control
O
HO
Acid (SN1-like) Condition
R
R
Nu: attachs site that best
stabilizes a carbocation
OH
O+
Nu
Nu
H
OH
O
OH
BnO
OH
BnO
+
OH
BnO
TL 1983, 24, 1377
OH
Me2CuLi
AlMe3
O
6:1
1:5
OH
Me3Al
JACS 1981, 103, 7520
S
S
OH S
_
O
JOC 1974, 3645
S
O
S
Ph
+
S
_
S
O
OH
Ph
(69 %)
1) TBS-Cl
2) MeI, CaCO 3, H+
S
OH
Tetrahedron Lett. 1992, 33, 931
Ph
Cyclic Sulfites and Sulfates (epoxide equivalents)
Synthesis 1992, 1035.
O
OH
R2
R1
SOCl2, Et3N
O
S
O
R1
OH
O
RuCl3, NaIO4
O
S
O
O
R1
Nu:
R2
O
R1
R2
R2
sulfate
sulfite
O
O
O
S
-
O
S
O
H
H2O
R2
Nu
R1
HO
H
R2
Nu
R1
H2O
O
O
O
S
O
R1
O
R2
Nu:
O S
O
R1
OH
R2
Nu
H
Nu:
Nu2
R1
H
R
Nu1 2
C-C BOND FORMATION
CO2Me
O SO2
O
2) TBS-Cl
CH3
OBn
MeO
OMe
O SO2
carpenter Bee
pheromone
O
CO2Me
O
OMe
OBn
OMe
H
N
H3C
OMe
NCH3
OMe
1) Ac2O
2) HCl
∆
O
OH
MeO
OBn
R2
1) H2, Rh
2) HF
OTBS
1) (CH3)2CuLi
CO2Me
R1
NaH
O
O SO2
CO2Me
CO2Me
R2
R1
MeO2C
MeO
meso
OBn
OBn
HO
OBn
OMe
MeO
OMe
MeO
NCH3
BnO
NCH3
BnO
OAc
Irreversible Payne Rearrangement
OH
O
OH
O
O SO2
OH
Bu 4NF
OTBS
O
O
Payne Rearrangement of 2,3-epoxyalcohols
Sigmatropic Rearrangements
Nomenclature:
σ bond
that breaks
Asymmetric Synthesis 1984, 3, 503.
1
2 3 R
1
3 R
2
∆
1
2 3 R
1
R
R
H
σ bond
that breaks
σ bond
that forms
[3,3]-rearrangement
3
4
1
3
2
3
2
Aldrichimica Acta 1983, 16, 60
2
∆
5
4
1
R
H
1
1
[1,5]-Hydogen migration
5
σ bond
that forms
3,3-sigmatropic Rearrangements
Cope Rearrangemets- requires high temperatures
R
∆
R
Organic Reaction 1975, 22, 1
95
C-C BOND FORMATION
96
Chair transition state:
CH3
CH3 Z
220 °C
H 3C H
H 3C
E
H
E,Z (99.7 %)
Z,Z (0 %)
E,E (0.3 %)
CH3
H
HH
H 3C
H 3C
CH3
H
CH3 Z
E
H 3C
H 3C
E
E,Z (0 %)
H 3C
Z
Z,Z (10 %)
E,E (90 %)
"Chirality Transfer"
H
Ph R
S
CH3
E
Ph
CH3
CH3
(87 %)
Diastereomers
Ph
Ph
H 3C
H 3C
Z
CH3
R
(13 %)
H
R
E
Ph
R
CH3
R
H
E
Z
CH3
Ph
CH3
Diastereomers
Ph
Ph
H 3C
H 3C R
Z
Z
H
S
CH3
- anion accelerated (oxy-) Cope- proceeds under much milder conditions (lower
temperature) JACS 1980, 102 , 774; Tetrahedron 1978, 34, 1877; Organic Reactions 1993, 43,
93; Comprehensive Organic Synthesis 1991, 5, 795. Tetrahedron 1997, 53, 13971.
C-C BOND FORMATION
O
OMe
OH
97
KH, DME, 110°C
OMe
KH
OH
O
O-
Ring expansion to medium sized rings
OH
O
KH, ∆
9-membered
ring
Claisen Rearrangements - allyl vinyl ether to an γ,δ-unsaturated carbonyl
Chem. Rev. 1988, 88, 1081.; Organic Reactions 1944, 2, 1.; Comprehnsive Organic Synthesis
1991, 5, 827.
∆
O
O
O
OH
CHO
O
220 °C
Hg(OAc)2
JACS 1979, 101 , 1330
O
O
O
H
H
H
O
O
O
Chair Transition State for Claisen
E-olefin
R
O
O
R
H
X
X
X=H
X= OEt, NMe2, etc
R
O
1,3-diaxial
interaction
X
R
E/Z = 90 : 10
E/Z = > 99 : 1
Z-olefin
R
X
X
H
O
O
new stereogenic
centers
R
old stereogenic
center
O
O
H
X
R
X
C-C BOND FORMATION
98
- Chorismate Mutase catalyzed Claisen Rearrangement- 105 rate enhancement over
non-enzymatic reaction
CO2H
O
CO2H
HO2C
Chorismate
mutase
J. Knowles
JACS 1987, 109, 5008, 5013
O
CO2H
OH
OH
Chorismate
Prephenate
- Claisen rearrangement usually proceed by a chair-like T.S.
HO2C
H
HO2C
O
H
CO2H
H
H
O
Chair
T.S
OH
OH
Opposite
stereochemistry
H
H
O
CO2H
CO2H
CO2H
H
Boat
T.S
H
OH
O
CO2H
OH
OH
OH
OH
+
O
J. Org. Chem. 1976, 41, 3497, 3512
J. Org. Chem. 1978, 43, 3435
O
+
O
R
R
O
H
H
O
s CH3
R
O
R
H
CH3
H
s
CH3
O
H
CH3
CH3
OH
CH3
OH
O
CH3
H
CH3
CO2R
CH3
OH
CH3
OH
Tocopherol
94 - 99 % ee
hydrophobically accelerated Claisen - JOC 1989, 54, 5849
C-C BOND FORMATION
Johnson ortho-ester Claisen:
EtO OEt
OH
OEt
O
H3C-C(OEt)3
∆
OEt
[3.3]
O
O
- EtOH
H+
Ireland ester-enolate Claisen.
Aldrichimica Acta 1993, 26, 17.
OTMS
O
LDA, THF
TMS-Cl
O
OH
[3.3]
O
O
O
LDA, THF
TMS-Cl
OBn
O
Me
CO2H
Me
Me
Me
JOC 1983, 48, 5221
OBn
Eschenmoser
NMe2
R
OH
EtO
OEt
∆
O
O
NMe2
NMe2
BF3
R
R
"Chirality Transfer"
R
R
N
O
N
R
Ph
N
O
aldehyde
oxidation state
O
Ph
Ph
(86 - 96 % de)
R= Et, Bn, iPr, tBu
[2,3]-Sigmatropic Rearrangement
H
Z
Comprehensive Organic Synthesis 1991, 6, 873.
H
H
:X Y
X Y:
R
R
R
H
X
Y:
R
R1
R
H
-Wittig Rearrangement
:X
Y
Organic Reactions 1995, 46, 105
_
base
O
O
SnR3
BuLi
:X Y
R
R1
X Y:
E
HO
_
O
Synthesis 1991, 594.
99
C-C BOND FORMATION
TBDPSO
TBDPSO
KH, 18-C-6,
Me3SnCH2I
TBDPSO
nBuLi
H
MeO
H
O
MeO
H
O
MeO
OH
O
O
O
SnMe3
TBDPSO
+
O
H
MeO
H3C
O
OH
(58%)
(42%)
CH3
Ph
_
O
CH3
H3C
CH3
CH3
H
(87 %)
OH
Ph
H
_
O
CH3
Ph
H3C
Sulfoxide Rearrangement
R
J. Am. Chem. Soc.
1997, 119, 10935
Li
TBDPSO
H
MeO
100
R
S
-
O
S
(13 %)
OH
Ph
(MeO) 3P
HO
O
O
O
CO2Et
CO2Et
(MeO) 3P
Ph
S
HO
O-
Ene Reaction Comprehensive Organic Synthesis 1991, 5, 1; Angew. Chem. Int. Ed. Engl. 1984, 23,
876; ; Chem. Rev. 1992, 28, 1021.
H
H
- Ene reaction with aldehydes is catalyzed by Lewis Acids (Et2AlCl)
R
R
H
O
H
O
H
OH
CHO
JOC 1992, 57, 2766
Et2AlCl
CH2Cl2 -78°C
Ph
O
O
O
Ph
O
O
H
OH
SnCl4
Ph
O
SnCl4
99.8 % de
O
OH
+ syn isomer
H3C
(94 : 6)
C-C BOND FORMATION
O
TiCl2
O
OH
O
H
CO2Me
CO2Me
PhS
+
H
OH
OH
O
R
Tetrahedron Lett.
1997, 38, 6513
(97% ee)
+
+
PhS
CO2Me
CO2Me
R
syn (90 % ee)
Angew. Chem. Int. Ed. Engl. 1989, 28, 38
CH3
CH3
C6H13
CH3
CO2Me
R
(9 : 1)
anti (99 % ee)
- Metallo-ene Reaction
PhS
C6H13
H2O
C6H13
(10 %)
H 3C
ClMg
Cl
C6H13
MgCl
H 3C
C6H13
H2O
H 3C
ClMg
H 3C
(> 1%)
intramolecular
BrMg
+
(11 : 1)
+
MgBr
BrMg
1)
CH3
1) Mg(0), Et2 )
2) 60 °C
Li
CHO 2) SOCl2
MgCl
MgCl
Cl
CH3
O
CH3
OH
Cl
SOCl2
CH3
CH3
MgCl
O2
H
1) PCC
2) MeLi
3) O3
CH3
CHO
4) KOH
5) H2
6) Ph3 P=CH2
MgCl
H3C
H
H
O
1) Mg(0), Et2 )
2) 60 °C
Capnellene
H
OH
101
C-C BOND FORMATION
Synthesis of Phyllanthocin
O
A. B. Smith et al. J. Am. Chem. Soc. 1987, 109, 1269.
O
O
(Me3Si)2 NLi
O
N
Ph
O
O
1) LAH
2) BnBr
N
Br
Ph
CH3
CH3
BnO
1) O3
2) H2, Lindlar's
BnO
MeAlCl
CHO
BnO
1) MEM-Cl
2) O3
H
BnO
BnO
CH3
O H
OH
CHO
OMEM
O
1)
2)
-
O
O
2) H
O
MEMO
3) Swern
O
1) DBU
2) H2, Pd/C
O
3) RuO4
O
O
O
O
(CH3 )2S(O)CH 2-
CH
O 3
O
BnO
O
O
O
1) LDA, TMSCl
2) BnMe3 NF, MeI
O
O
HO2C
O
BnO
O
O
BnO
O
+
O
H3O +
O
1) ZnCl2
BnO
CH3
O
CH3
MeO2C
O
O
O
Ph
Phyllanthocin
102
C=C BOND FORMATION
C=C Bond Formation
1.
2.
3.
4.
5.
6.
7.
8.
9.
10
11.
103
C&S Chapt. 2 # 5,6,8,9,12
Aldol Condensation
Wittig Reaction
(Smith, Ch. 8.8.A)
Peterson Olefination
Julia-Lythgoe Olefination
Carbonyl Coupling Reactions (McMurry Reaction)
(Smith Ch. 13.7.F)
Tebbe Reagent
Shapiro and Related Reaction
β−Elimination and Dehydration
From Diols and Epoxides
From Acetylenes
From Other Alkenes-Transition Metal Catalyzed Cross-Coupling and Olefin
Metathesis
Aldol Condensation -Aldol condensation initially give β-hydroxy ketones which under
certain conditions readily eliminated to give α,β-unsaturated carbonyls.
OMe
OMe
LDA, THF, -78°C
O
O
OMe OR
O
O
O
Tetrahedron
1984, 40, 4741
CHO
-78C → RT
OMe OR
O
Robinson Annulation : Sequential Michael addition/aldol condensation between
a ketone enolate and an alkyl vinyl ketone (i.e. MVK) to give a cyclohex-2-en-1one
JOC 1984, 49 , 3685
Synthesis 1976, 777.
O
O
O
O
O
L-proline
Weiland-Mischeler Ketone
(chiral starting material)
Mannich Reaction - α,β-unsaturated carbonyls (α-methylene carbonyls)
O
H2C=O, Me2NH•HCl
Me
H 2C N
+ Me
O
Cl-
Mannich Reaction
C=C BOND FORMATION
104
Wittig Reaction
review: Chem. Rev. 1989, 89, 863.
mechanism and stereochemistry: Topic in Stereochemistry 1994, 21, 1
- reaction of phosphonium ylide with aldehydes, ketones and lactols to give
olefins
R
X
Ph3P
R
X
O
R1
strong
base
+
PPh3
R
-
+
PPh3
R
Ph3P O
PPh3
phosphorane
ylide
+
Ph3P O -
R2
_
- Ph3P=O
R2
R
R
R1 2
R
R
R1 2
R
R1
oxaphosphetane
betaine
- Olefin Geometry
O
L
S
Z- olefin
Ph3P
L
S
O
L
S
E- Olefin
Ph3P
S
L
- With "non-stabilized" ylides the Wittig Reaction gives predominantly Z-olefins.
Seebach et al JACS
-
O
O
O
L
S
L
S
S
L
L
S
O
+ PPh3
S
- Ph3P=O
L
L
S
S
S
S
S
L
L
Z-olefins
- "Stabilized ylides" give predominantly E-olefins
O
+
Ph3P
L
O PPh3
Ph3P
L
S
S
PPh3
+
+ PPh3
L
L
_
L
CO2Me
S
S
L
CO2Me
C=C BOND FORMATION
105
- Betaine formation is reversible and the reaction becomes under thermodynamic
control to give the most stable product.
- There is NO evidence for a betaine intermediate.
- Vedejs Model:
R
O
H
R
Ph
Ph P
Pukered (cis)
oxaphosphetane
(kinetically favored)
Cis
H
Ph
H
R
Ph
Ph P
R
H
Planar (trans)
oxaphosphetane
(thermodynamically favored)
Trans
O
Ph
Phosphonate Modification (Horner-Wadsworth-Emmons)
(EtO)3P
R
X
Arbazov Reaction
R
_
R
P(OEt)2
P(OEt)2
O
O
- R is usually restricted to EWG such as CO2H, CO2Me, CN, SO2Ph etc. and the
olefin geometry is usually E.
- Still Modification
TL 1983, 24, 4405.
(CF3CH2O)2P
CO2Me
O
- CF3CH2O- groups make the betaine less stable, giving more Z-olefin.
O
CHO
Me3Si
(CF3CH2O)2P
-
CO2Me Me3Si
+
CO2Me
(Me3Si)2N K , THF,
18-C-6
Peterson Olefination
JACS 1988,
110, 2248
Z:E = 25:1
review: Synthesis 1984, 384
Organic Reactions 1990, 38 1.
-
O
O
Me3Si
CO2Me
LDA, THF Me3Si
_
CO2Me
R1
R2
R1
Me3Si
MeO2C
Silicon can stabilize
an α- negative charge
O
R1
R2
Me3Si
MeO2C
- Me3Si-O
R1
R2
H
usually a mixture
of E and Z olefins
CO2Me
R2
H
C=C BOND FORMATION
Julia-Lythgoe Olefination
106
TL 1973, 4833 Tetrahedron 1987, 43, 1027
OH
SO2Ph
LDA, THF, -78°C
OTBS
1) MsCl, Et N
OTBS 2) Na(Hg),3MeOH
TL 1991,
32, 495
mixture
of olefins
CHO
TBSO
R
PhO2S
R
R
Tetrahedron Lett. 1993, 34, 7479
O
O
H
O
O
1) tBuLi, THF,
HMPA, -78°C
2) Na(Hg) NaHPO4
THF, MeOH, -40°C
OBz
OSitBuPh2
HO
O
OSitBuPh2
HO
O
HO
O
O
O
OH
OH
OH
Milbemycin α1
OR
OTBDPS
O
SmI2,
HMPA/THF
O
R= H, Ac
O
SO2Ph
O
O
OBz
+
PhO2S
O
O
OTBDPS
Synlett 1994, 859
75 % yield
E:Z = 3 : 1
Ramberg-Backlund Rearrangement
Br
OSiPh2tBu
1) Na2S•Al2O3
2) mCPBA
OSiPh2tBu
O
OSiPh2tBu
S
SOCl2
OSiPh2tBu
O
Br
Cl
O
OR
S
MeLi, THF
OR
O
O
OR
S
- SO2
OR
OR
O
thiiarane dioxide
OR
JACS 1992, 114, 7360
Carbonyl Coupling Reactions (McMurry Reaction)
Reviews: Chem. Rev. 1989, 89, 1513.
- reductive coupling of carbonyls with low valent transition metals, Ti(0)
or Ti(II), to give olefins
O
"low-valent Ti"
R1
R2
R1
R1
R2
R2
+
R1
R2
R2
R1
usually a mixture
of E and Z olefins
excellent method for the preparation of strained (highly substituted) olefins
- Intramolecular coupling gives cyclic olefins
C=C BOND FORMATION
107
CHO
"low-valent Ti"
JACS 1984, 106, 723
CHO
OMe
OMe
TiCl4, Zn,
pyridine
(86 %)
OMe
OHC
Tetrahedron Lett. 1993, 34, 7005
OMe
OH OH
O
Tebbe Reagent
Cp2Ti(CH2)ClAlMe2
- methylenation of ketones and lactones. The later gives cyclic enol ethers.
H2
C
Ti AlMe2
Cp Cl
Cp
O
O
O
200 °C
O
- Cp2TiMe2 will also do the methylenation chemistry
JACS 1990, 112, 6393.
Shapiro and Related Reactions
Organic Reactions 1990, 39, 1 : 1976, 23, 405
- Reaction of a tosylhydrazone with a strong base to give an olefin.
NNHTs
Et
N
2 eq. nBuLi
_
N
Et
N N
Ts
Et
Et
- N2
_
THF
Me
_
Me
Me
E
_
E+
Me
Et
Me
Bamford-Stevens Reaction- initial conversion of a tosylhydrazone to a diazo
intermediate
H
H
R1
R2
R3
H
R1
R2
base
R1
R2
R3
NNHTs
N
N
_
R3
+N
_
N
Ts
a: aprotic- decomposition of the diazo intermediate under aprotic conditions
gives and olefin through a carbene intermediate.
H
R1
R2
R3
H
- N2
R1
R2
+N
_
N
R1
R3
••
R3
R2
Carbene
b. protic- decomposition of the diazo intermediate under protic conditions an
olefin through a carbonium ion intermediate.
H
R1
R2
H
R3
+N
_
N
H+
R1
R2
R3
+ N
N
H
- N2
H
R1
R2
R3
+ H
-H
R1
+
R2
R3
C=C BOND FORMATION
108
β- Eliminations
Anti Eliminations
- elimination of HX from vicinal saturated carbon centers to give a olefin,
usually base promoted.
- base promoted E2- type elimination proceeds through an anti-periplanar
transition state.
B:
H
R1
R3
R2
R4
R1
R2
R3
R4
X
- typical bases: NaOMe, tBuOK, DBU, DBN, DABCO, etc.
N
N
N
N
N
N
DBN
DBU
DABCO
- X: -Br, -I, -Cl, -OR, epoxides
O
O
O
B:
OH
R
R
O
R
N
AlEt2
R'
OH
R
- or TMS-OTf, DBU
"unactivated"
BCSJ 1979, 52, 1705
JACS 1979, 101, 2738
R'
Syn Elimination
- often an intramolecular process
H
R1
R2
R1
R2
X
R3
R4
R3
O
X=
R4
O
Se
S
Ph
Ph
Cope Elimination- elimination of R2NOH from an amine oxide
OLI
O
+
Me2N=CH2 I -
O
NMe2
THF, -78°C
ON
Me2
mCPBA
THF
O
O
1) Me2CuLi
+
2) Me2N=CH2
CF3CO2-
NMe2
JACS 1977,
99 , 944
Tetrahedron
1977, 35 , 613
O
- Me2NOH
C=C BOND FORMATION
Selenoxide Elimination
109
Organic Reactions 1993, 44, 1.
O
N SePh
R'
R
R
Bu3P: O
R'
R
- or NO2
OH
mCPBA
R'
H
SeAr
- ArSeOH
R'
R
JACS 1979,
101, 3704
Se
O
SeAr
SeCN
H 3C
H3CHN
O H 3C
N
H 3C
PhSeO2H,
PhH, 60 °C
O
N
H3CHN
(82%)
Ph
O H 3C
O
O
JOC 1995, 60, 7224
JCS P1 1985, 1865
N
N
Ph
O
Dehydration of Alcohols
- alcohols can be dehydrated with protic acid to give olefins via an E 1
mechanism.
- other reactions dehydrate alcohols under milder conditions by first converting
them into a better leaving group, i.e. POCl3/ pyridine, P2O5
Martin sulfurane; Ph2S[OCPh(CF3)2]2 JACS, 1972, 94, 4997
dehydration
occurs under very mild, neutral conditions, usually gives the most stable olefin
OH
MSDA, CH2Cl2
BzO
JACS 1989,
111 , 278
BzO
H
H
Burgess Reagent (inner salt) JOC, 1973, 38, 26 occurs vis a syn elimination
_
+
MeO2CNSO2NEt3
H
R1
R2
_
+
MeO2CNSO2NEt3
R3
R4
MeO2C
- N SO2
H
O
PhH, reflux
OH
Olefins from Vicinal Diols
Corey-Winter Reaction
R1
R2
R4
R3
R1
R4
R2
R3
JACS 1963, 85, 2677; TL 1982, 1979; TL 1978, 737
S
OH
R'
Im2C=S
O
O
R
R3P:
R
OH
R
R'
R'
C=C BOND FORMATION
110
- vic-diols can be converted to olefins with K2WCl6 JCSCC 1972, 370; JACS
1972, 94, 6538
- This reaction worked best with more highly substituted diols and give
predominantly syn elimination.
- Low valent titanium; McMurry carbonyl coupling is believed to go through
the vic-diol. vic-diols are smoothly converted to the corresponding olefins under
these conditions.
JOC 1976, 41 , 896
Olefins from Epoxides
Ph2P -
O
R1
H
HO H
H
R2
R2
HO H
MeI
H PPh2
R1
R2
H PPh2Me
+
R1
+
PPh2Me
HO
R2
-
NC-Se -
O
H
H
R1
R2
NCO H
R1
H
R1
R2
H
H
"inversion "
of R groups
H
R1
R1
-Ph2MeP=O
R2
-
H SeCN
OR
2
R1
R1
-
NCOR
2
H
H
R1
R2
H
H
R2
H Se
Se -
NCSe
O H
Se
R1
H
H
R2
H
"retention"
of R groups
From Acetylenes
- Hydrogenation with Lindlar's catalyst gives cis-olefins
R
R'
Co2(CO)8
R
H2, Rh
R'
R
Co2(CO)6
R'
Bu3SnH
Tetrahedron Lett. 1998, 39, 2609
H
SnBu3
R
R'
From Other Olefins
Sigmatropic Rearrangements
- transposition of double bonds
Birch Reduction
Tetrahedron 1989, 45, 1579
OMe
OMe
H3O+
O
H
C=C BOND FORMATION
111
Olefin Isomerization- a variety of transition metal (RhCl3•H2O) catalyst will
isomerize doubles bonds to more thermodynamically favorable configurations
(i.e. more substituted, trans, conjugated)
RhCl3•3H2O
JOC 1987,
52 , 2875
EtOH
OH
OH
Cl
Cl
Ti
JACS 1992
114 , 2276
+
up to 80% ee
Olefin Inversion
Tetrahedron 1980, 557
- Conversion of cis to trans olefins
- Conversion of trans to cis- olefins
R
R'
R
hν, PhSSPh
R'
OH
OH
I2, CH2Cl2
CO2Me
HO
C5H11 OH
CO2Me
C5H11
OH
OH
JACS, 1984, 107, xxxx
Transition Metal Catralyzed Cross-Coupling Reactions
Coupling of Vinyl Phosphonates and Triflates to Organometallic Reagents
- vinyl phosphates review: Synthesis 1992, 333.
O
OP(OEt)2
Li, NH3,
tBuOH
R2CuLi
R
R
O
O
O
R2CuLi
(EtO)2PCl
- preparation of enol triflates
Synthesis 1997, 735
O
OTf
LDA, THF, -78°C
kinetic
product
Tf2NPh
O
R
OP(OEt)2
O
Tf2O, CH2Cl2
tBu
N
tBu
OTf
thermodynamic
product
C=C BOND FORMATION
- reaction with cuprates.
112
Acc. Chem. Res. 1988, 25, 47
OTf
Ph
TL 1980,
21 , 4313
Ph2 CuLi
tBu
tBu
- palladium (0) catlyzed cross-coupling of vinyl or aryl halides or triflates with
organostannanes (Stille Reaction)
Angew. Chem. Int. Ed. Engl. 1986, 25, 508.; Organic Reactions 1997, 50, 1-652
SnMe3
O
O
1)LDA, Tf2NPh
2) Pd(PPh3)4
O
JOC 1986,
51 , 3405
O
O
OH
OTBS
Bu3Sn
I
I Bu Sn
3
O
O
O
OH
J. Am. Chem. Soc.
1998, 120, 3935
OH
HO
TBSO
Bu3Sn
HO
TBSO
(-)-Macrolactin A
I
BOMe
TESO
CHO 1)
2
MgBr
TESO
O3, NaBH4
TESO
3) TBSCl, imidazole
(76%)
CHO
1) Dess-Martin
Periodinane
2) Ph3P+CH2I, INaHMDS
(38%)
TBSO
I
TESO
CO2H
SnBu3
(42%)
OH
TBSO
3) DIBAL
(50%)
Bu3SnCHBr2, CrCl2
PdCl2(MeCN)2
(65%)
TESO
Bu3SnH, AIBN
OH
I
OH
TBSO
TESO
OTES
I
TBSO
PdCl2(MeCN)2
TESO
OH b) I2
(83%
OPiv
Bu3Sn
1) HF, MeCN
2) Me4NBH(OAc)3
OH
a) nBu3SnH,
(Ph3P)2PdCl2
OTES
(77%)
OPiv
TBSO
3) Dess-Martin
Periodinane
(70%)
2) TESOTf, 2,6-lutidine
(52%)
TESO
1) O3, Me 2S
2) allyl bromide, Zn
I
Bu3Sn
CO2H
TESO
CO2H
C=C BOND FORMATION
113
I
TESO
OH
+
Bu3Sn
Ph3P, DEAD
TESO
CO2H
(50%)
TESO
TESO
Bu3Sn
OH
I
O
1) Pd2(dba)3
iPr2NEt
2) TBAF
O
O
HO
(35%)
TESO
O
HO
TESO
(-)-Macrolactin A
palladium (0) catalyzed carbonylations- coupling of a vinyl triflate with a
organostanane to give α,b-unsaturated ketones.
O
OTf
Ph
Me3SnPh
Pd(0), CO
But
JACS 1994,
106 , 7500
tBu
Nickel (II) Catalyzed Cross-Coupling with Grignard Reagents (Kumada
Reaction):
Pure Appl. Chem. 1980, 52, 669
Bull Chem. Soc. Jpn. 1976, 49, 1958
R-MgBr
(dppp)NiCl2
X
R
R= alkyl, vinyl or aryl
Aryl or vinyl halide or triflate
Br
AcO
OMe
Br
Ni
1)
N
H
OAc
HO
Ni
R1
N
H
OH
(±)-Neocarazostatin B
Tetrahedron 1998, 54, 4413, Acc. Chem. Res. 1995, 25, 446.
+
olefin
metathesis
R2
catalyst
catalyst
(CH 2)n
R1
R2
Ring-Opening Metathesis
Polymerization (ROMP)
(CH 2)n
catalyst
(CH 2)n
Tetrahedron Lett.
1998, 39, 2537, 2947,
(2 equiv), 65 °C
2) LiAlH4 , Et2 O
(72%)
Olefin Metathesis
OMe
Br
(CH 2)n
n
Ring-Closing Metathesis
(RCM)
C=C BOND FORMATION
Metathesis Catalysts:
iPr
H3C
F3C
F3C
iPr
(C6H11)3P
Cl
N
Ru
O Mo
O
F3C
F3C
Ph
Ph
Ph
Cl
(C6H11)3P
(C6H11)3P
Cl
Ru
Cl
(C6H11)3P
CH3
Schrock' s Catalyst
Grubbs' Catalyst
Mechanism:
+
R1
R2
[M]
R1
[M]
R1
[M]
R1
R2
R2
+
[M]
Ph
114
C≡C BOND FORMATION
115
C≡C Bond Formation
1.
2.
3.
4.
5.
6.
From other acetylenes
From carbonyls
From olefins
From Strained Rings
Eschenmosher Fragmentation
Allenes
From Other Acetylenes
- The proton of terminal acetylenes is acidic (pKa= 25), thus they can be
deprotonated to give acetylide anions which can undergo substitution reactions
with alkyl halides, carbonyls, epoxides, etc. to give other acetylenes.
R
R1-X
R
_
R
H
R1
O
R2
OH
R3
R
R
R2 3
M+
O
Et2AlCl
OH
R
- Since the acetylenic proton is acidic, it often needs to be protected as a
trialkylsilyl derivative. It is conveniently deprotected with fluoride ion.
R
H
F-
B:, R3SiCl
R
SiR3
R
H
Acetylide anions and organoboranes
R1
_
Li+
R3B
_
R1
BR3
Li+
I2
R1
R
JOC 1974, 39 , 731
JACS, 1973, 95 , 3080
Palladium Coupling Reactions:
O
O
O
Cl
iPr3Si
SnBu3
O
JACS 1990,
112, 1607
Cl
(Ph3P)4Pd
iPr3Si
SiiPr3
C≡C BOND FORMATION
116
OTBS
OTBS
CO2Me
Br
CO2Me
C5H11
OTBS
OTBS
OTBS
Pd(Ph3P)4, CuI
iPr2NH, PhH
TBSO
JACS 1985,
107, 7515
C5H11
OH
HO
CO2H
OH
Copper Coupling- 1,3-diynes
+
R1
Cu(OAc)2
R2
R1
Adv. Org. Chem.
1963,4 , 63
R1
Nicholas Reaction
- acetylenes as their Co2(CO)8 complex can stabilize an α-positive charge,
which can subsequently be trapped with nucleophiles.
OR4
R1
(CO)6Co2
Co2(CO)8
OR4
R1
R2
(CO)6Co2
R1
R2
(CO)6Co2
Nu:
R1
oxidative
decomplexation
Nu
Nu
R1
R2
R3
R2
R3
N
CO2Me
R
R3 2
R3
R3
N
Tf2O, CH2Cl2,
MeNO2, -10°C
+
CO2Me
N
I2
CO2Me
JCSCC 1991, 544
O
TBSO
tBu
HO
N
tBu
O
(OC)6Co2
Co2(CO)6
Co2(CO)6-acetylene deocomplexation: JOC 1997, 62, 9380
From Aldehydes and Ketones
R'-X
RCHO
P3P, CBr4
Br
R
Br
2 eq. nBuLi
R
_
Li+
ClCO2Et
R
R'
R
CO2Et
R
H
H2O
C≡C BOND FORMATION
Br2C
OHC
OSitBuPh2
OHC
OSitBuPh2
OSitBuPh2
CBr4, Ph3P
OSitBuPh2
Br2C
a) nBuLi,
THF, -78°C
b) ClCO2Me
MeO2C
117
OSitBuPh2
OSitBuPh2
MeO2C
JACS 1992, 114 , 7360
- by conversion of ketones to gem-dihalides followed by elimination
O
PCl5
R'
R
Cl
Cl
R
Cl
tBuOK
R'
- HCl
R'
R
tBuOK
R
- HCl
R'
- by conversion of ketones to enol phosphates followed by elimination
LDA, (EtO)2P(O)Cl
O
O
R'
R
P(O)(OEt)2 LiNH , NH
2
3
R
JOC 1987,
52 , 4885
R'
R'
R
- Insertion reaction of a vinyl carbene (terminal acetylenes)
N2
R
C
N2CHP(O)(OMe)2
R'
tBuOK
R
+
-N2
R'
••
O
C
R
R
R'
JOC 1982,
47 , 1837
R'
O
Me3Si
_
N2
Synlett 1994, 107
THF,
-78°C → RT
Via Elimination Reactions of Vinyl Halides
- Treatment of vinyl halides with strong base gives acetylenes.
X
Base
R'
R
R
R'
- HX
H
X= Cl, Br, I, OTf, O3SR, OP(O)R2
Base= LDA; tBuOK; NaNH2, DBU
- Addition of Grignard reagents to 1,1-difluoroethylene yields an acetylide anion
which can be subsequently trapped with electrophiles.
H2C=CF2
R-MgX
E+
_
R
R
E
TL 1982, 23 , 4325
JOC 1976, 41 , 1487
Strained Rings
Topics in Current Chemistry 1983, 109, 189.
- Cyclopropenones and cyclobutendiones can be photolyzed or thermolyzed
(FVP) to give acetylenes.
O
O
O
hν (209 nm)
8 °K
C
- CO
O
- CO
JACS 1973,
95 , 6134
C
O
Benzyne
C≡C BOND FORMATION
1) 370 °C
2) 12°K
O
118
ACIEE 1988,27 , 398
JACS 1991, 113 , 6943
- CO
R1
R2
FVP
R1
(retro- D-A)
+
R2
CL 1982, 1241
Eschenmoser Fragmentation
Ph
N
O
Ph
N
R
R
O
R'
HCA 1972,
55 , 1276
O
R'
R'
HN NH2
iPr
iPr
O
O
R
Base
-orheat
O
tBuOOH,
triton B, C6H6
O
JOC 1992, 57, 7052
iPr
AcOH, CH2Cl2
Allenes
Tetrahedron 1984, 40, 2805
- from dihalocyclopropanes
Br
R
R'
Br
R'
R
_
••
R
Br
R'
H
R'
R
H
- From SN2' Reactions
Nu:
X
H
Nu
R
R'
R
R'
- from sigmatropic rearrangements from propargyl sulfoxides and phosphine
oxides.
Ph
OH
O
R
R'
Ph
:P
Ph2PCl, Et3N
R
O
Ph2P
R
R'
R'
H
TL 1990, 31 , 2907
JACS 1990, 112 , 7825
FUNCTIONAL GROUP INTERCONVERSIONS
119
Functional Group Interconversions
C&S Chapter 3 #1; 2; 4a,b, e; 5a, b, d; 6a,b,c,d; 8
1
2
3
4
5
6
7
sulfonates
halides
nitriles
azides
amines
esters and lactones
amides and lactams
Sulfonate Esters
- reaction of an alcohols (1° or 2°) with a sulfonyl chloride
R OH
O
R'SO2Cl
R'=
R O S
CH3
mesylate
CF3
triflate
R'
O
sulfonate ester
CH3
- sulfonate esters are very good leaving groups.
competing side reaction
tosylate
Elimination is often a
Halides
- halides are good leaving groups with the order of reactivity in SN2 reactions
being I>Br>Cl. Halides are less reactive than sulfonate esters, however
elimination as a competing side reaction is also reduced.
- sulfonate esters can be converted to halides with the sodium halide in acetone
at reflux. Chlorides are also converted to either bromides or iodides in the same
fashion (Finkelstein Reaction).
O
R O S
R'
X-
X= Cl, Br, I
R X
O
R Cl
NaI, acetone
reflux
- conversion of hydroxyl groups to halides:
R OH
- R-OH to R-Cl
- SOCl2
- Ph3P, CCl4
- Ph3P, Cl2
- Ph3P, Cl3CCOCCl3
R
I
Organic Reactions 1983, 29, 1
R X
FUNCTIONAL GROUP INTERCONVERSIONS
- R-OH to R-Br
- PBr3, pyridine
- Ph3P, CBr4
- Ph3P, Br2
- R-OH to R-I
- Ph3P, DEAD, MeI
Nitriles
- displacement of halides or sulfonates with cyanide anion
KCN, 18-C-6
DMSO
R X
R C
N
- dehydration of amides
O
R
-
R C N
NH2
POCl3, pyridine
TsCl, pyridine
P2O5
SOCl2
- Reaction of esters and lactones with dimethylaluminium amide
TL 1979, 4907
Me
H 3C
Me2AlNH2
O
O
OH
JOC 1987, 52, 1309
NC
Ar
Ar
- Dehydration of oximes
R CHO
N
H2NOH•HCl
R
OH
P2O5
R C
H
N
- Oxidation of hydrazones
O
O
N
NMe2
(97%)
- Reduced to aldehydes with DIBAL.
DIBAL
RC≡N
C
N
RCHO
Tetrahedron Lett.
1998, 39, 2009
120
FUNCTIONAL GROUP INTERCONVERSIONS
121
Azides
- displacement of halides and sulfonates with azide anion
LDA, THF
NBS
O
O
O
NaN3
O
O
O
SO2N(C6H11)2
SO2N(C6H11)2
Br
SO2N(C6H11)2 N3
O
O
O
TL 1986, 27, 831
HO
SO2N(C6H11)2
NH2
NH2
- activation of the alcohol
R OH
+
+
N
+
N
F
Me
N
Me
+
EtO2C N N CO2Et
O
Me
TsO -
R OH
+
R N3
OR
Ph3P, NaN3
+
PPh3
EtO2C
N N
_
CO2Et
DEAD
activated alcohol
+
R O PPh3
R-OH
R N3
+
Ph3P=O
JOC 1993, 58, 5886
HO
(PhO)2P(O)-N3
O
N3
O
O
DBU, PhCH3
SN2
P(OPh)2
+
+ DBU-H
-
+ N3
(91 %)
O
(97.5 % ee)
Ar
(99.6 % ee)
- Photolyzed to aldehydes
Amines
- Gabriel Synthesis
O
N - K+
O
R
O
- reduction of nitro groups
R NO2
H2, Pd/C
Al(Hg), H2O
NaBH4
LiAlH 4
Zn, Sn or Fe and HCl
H2NNH2
sodium dithionite
X
N R
H2NNH2
O
R NH2
R NH2
FUNCTIONAL GROUP INTERCONVERSIONS
- reduction of nitriles
R C N
R CH2 NH2
H2, PtO 2/C
B2H6
NaBH4
LiAlH 4
AlH3
Li, NH3
- reduction of azides
R N3
H2, Pd/C
B2H6
NaBH4
LiAlH 4
Zn, HCl
(RO)3P
Ph3P
thiols
R NH2
- reduction of oximes (from aldehydes and ketones)
N
R
OH
NH2
R'
R
R'
R'
R
N
H2, Pd/C
Raney nickel
NaBH4, TiCl4
LiAlH 4
Na(Hg), AcOH
- reduction of amides
O
R
N
R'
R''
R''
H2, Pd/C
B2H6
NaBH4, TiCl4
LiBH4
LiAlH 4
AlH3
- Curtius rearrangement
O
O
NaN3
R
R N O
isocyanate
H2O
O
∆
- N2
R
••
N
••
Cl
••
R
••
••
N N N
+
nitrene
R NH2
122
FUNCTIONAL GROUP INTERCONVERSIONS
- reductive aminations of aldehydes and ketones
- Borsch Reaction
- Eschweiler-Clark Reaction
- alkylation of sulfonamides
Tf
Tf
N
HN
N
HN
Tf
Tf
K2CO3, DMF
110°C
Tf
Br
Br
Tf
N
N
N
N
Tf
NH HN
Na, NH3
TL 1992, 33 , 5505
NH HN
Tf
cyclam
- transaminiation
O
Ph
N
PhCH2NH2
Ph
N
NH2
Can. J. Chem.
1970, 48, 570
H3O +
tBuOK
H+
Esters and Lactones
- Reaction of alcohols with "activated acids"
- Baeyer-Villigar Reaction
Organic Reactions 1993, 43, 251
- Pd(0) catalyzed carboylation of enol triflates
OTf
CO2R
CO, DMF
Pd(0), ROH
TL 1985, 26 , 1109
- Arndt-Eistert Reaction
O
R
Angew. Chem. Int. Ed. Engl. 1975, 15, 32.
O
CH2 N2
Cl
Et2 O
O
hν
N2
R
Wolff
Rearrangement
C O
H
O
R'OH
R
OR'
ketene
O
O
TsN3, Et3N
CO2Me
R
••
CH
R
ROH
R
diazo ketone
N2
R
- Diazoalkanes: carbene precursors
R-CHO
1) NH2NH2
2) Pb(OAc)4, DMF
R-N2
R3N
H 2N
R
- Halo Lactonizations
JOC 1995, 60, 4725
TsN3
N
N2
R
R
review: Tetrahedron 1990, 46 , 3321
+
I
I
I2-KI
CO2H
R
H2O, NaHCO3
O
O
H
O
O
123
FUNCTIONAL GROUP INTERCONVERSIONS
Pd(OAc)2
(5 mol %)
CO2H
JOC 1993, 58, 5298
O
DMSO, air
(86%)
O
- Selenolactonization
PhSe
O
H 2O 2
O
PhSeCl, CH2Cl2
O
O
JACS 1985,
107 , 1148
O
OH
- Mitsunobu Reaction
Synthesis 1981 , 1; Organic Reactions, 1991, 42, 335
Mechanism: JACS 1988, 110 , 6487
O
DEAD, Ph3P
OH
R
O
R''CO2H
R'
R
R''
Inversion of alcohol
stereochemistry
R'
Amides and Lactams
- reaction of an "activated acid" with amines
- Beckman Rearrangement
Organic Reactions 1988, 35, 1
O
R
N
R'
R
OH
PCl5
O
R'
R
NR'
- Schmidt rearrangement
O
R
O
HN3
R'
H+
R
NR'
- others
O
OTf
NR2
CO, DMF
Pd(0), R2NH
TL 1985, 26 , 1109
OH
O
O
O
PhCH2NH2
N
H
AlMe3
OTHP
Ph
TL 1977, 4171
OTHP
-Weinreb amide Tetrahedron Lett. 1981, 22, 3815
O
DIBAL
O
R
O
H3CNH(OCH3) •HCl
OR'
AlMe3
R
R
N
H
OCH3
O
CH3
R1-M
R
R1
124
THREE-MEMBERED RING FORMATION
3 Membered Rings
1.
2.
3.
epoxides
a. peracids, hydroperoxides and dioxiranes
b. transition metal catalyzed epoxidations
c. halohydrins
d. Darzen's condensation
e. sulfur ylides
cyclopropanes
a. Simmons-Smith reactions
b. diazo compounds
c. sulfur ylides
d. S N2 displacements
aziridines
a. nitrenes
b. SN2 displacements
Epoxides
- peracid, hydroperoxide and dioxirane oxidation of alkenes
- transition metal catalyzed epoxidation of alkenes
- Sharpless epoxidation
- Metal oxo reagents (Jacobsen's reagent)
- from halohydrins
Br
NBS, H2O
base
O
OH
bromohydrin
- Darzen's Condensation
O
_
BrCHCO2Et
B:
BrCH2CO2Et
R
R
R'
OO
CO2Et
R'
R'
Br
- sulfur ylides Chem. Rev. 1997,97, 2421.
O
O
Me S +
CH2Me
Corey
+
Me2SCH2-
Ph S
CH2-
+
_ SPh2
NMe2
Johnson
sulfoximine
CO2Et
R
Trost
125
THREE-MEMBERED RING FORMATION
126
- dimethylsulfoxonium methylide and dimethylsulfonium methylide
(Corey's reagent) review: Tetrahedron 1987, 43 , 2609.
O
O
R
-
O
DMSO, NaH
R'
O
SMe2
R
R
R'
R'
- cyclopropyldiphenylsulfonium ylide (Trost's reagent) ACR 1974, 7, 85.
+
O
R
_
O
SPh2
-
O
R'
O
SPh2
R
BF3
R
R'
- sulfoximine ylides (Johnson's reagent)
O
Ph S CH2NMe2
O
R
ZnCu +
R
ACR 1973, 6 , 341
O
R
R'
Cyclopropanes
- Simmons-Smith Reaction
R'
R'
R'
Org. Reactions 1973, 20, 1.
ICH2ZnI
CH2I2
- polar groups (-OH, -NR2- CO2R) can direct the cyclopropanation
OH
OH
JACS 1979, 101 , 2139
ZnCu, CH2I2
- sulfur ylides
O
O
O
Me2S CH2-
Ph
Ph
O _
Ph S
NMe2
Tetrahedron
1987, 43 , 2609
Ph
Ph
O
O
ACR 1973,
6 , 341
THREE-MEMBERED RING FORMATION
127
- diazo alkanes and diazo carbonyls
Synthesis 1972, 351; 1985, 569
- cyclopropanation with diazoalkanes; olefin requires at least on electron
withdrawing group.
N N
CO2Me
MeO2C
CH2N2
(MeO)2HC
CO2Me
CO2Me
JACS 1975,
97 , 6075
MeO2C
hν
MeO2C
(MeO)2HC
(MeO)2HC
- diazoketones; photochemical or metal catalyzed decomposition of
diazoketones to carbenes followed by cyclopropanation of olefins.
Org. Rxns. 1979, 26, 361; Tetrahedron 1981, 37, 2407
CuSO4 , ∆
N2
JACS 1970, 92 , 3429
JACS 1969, 91 , 4318
O
O
Rh2(OAc)2
O
Ph
O
O
O
Ph
Ph
N2
O
JACS 1993, 115, 9468
O
O
O
Ph
91 % yield
89 % de
Asymmetric cyclopropanation: Doyle, Chem Rev. 1998, 98, 911
Aldrichimica Acta 1997, 30, 107
O
O
CO2Et
CO2Et
TsN3 , Et3N
N2
C5H11
C5H11
O
O
CO2Et
- SPh
JACS 1977,
99, 1940
CO2Et
C5H11
C5H11
SPh
- SN2 Reactions
DBU
O
O
O
Br
O
O
O
CO2Et
JACS 1978,
99 , 1940
CO2Et
- Electrophillic Cyclopropanes review: ACR 1979, 12 , 66
in many ways, cyclopropanes react silmilarly to double bonds
- homo-1,4-addition
CO2R
Nu:
CO2R
Nu
CO2R
CO2R
ACR 1979,12 , 66
THREE-MEMBERED RING FORMATION
Nu:= malonate anion, amines, thiolate anion, enamines, cuprates
(usually requires double activation of cyclopropane)
Me
H
MeO2C
MeO2C
Me2CuLi
O
TL 1976, 3875
O
- hydrogenation
CO2Me
CH2I2, Zn-Cu
O
ArCO3H,
Na2HPO4
CO2Me
CO2Me
H
H
H
O
CO2Me
H2, PtO2,
AcOH
TL 1982, 23 , 1871
H
- hydrolysis
OH
OH
OH
CH2I2, Zn-Cu
H , MeOH
MeO
OMe
JACS 1967
89 , 2507
+
O
Aziridines
R2
R1
Ts
Cu(acac)2
R3
PhN=ITs
N
R2
R1
J. Org. Chem. .1991, 56, 6744
R3
128
FOUR-MEMBERED RING FORMATION
4 Membered Rings
1.
2.
cyclobutanes & cyclobutenes
oxatanes
Cyclobutanes
- [2+2] cycloadditions
- photochemical cycloadditions (2πs +2πs)
Acc. Chem. Res. 1968, 1, 50; Synthesis 1970, 287; Acc. Chem. Res. 1971, 4, 41;
Organic Photochemistry 1981, 5, 123; Angew. Chem. Int. Ed. Engl. 1982, 21, 820;
Acc. Chem. Res. 1982, 15, 135; Organic Photochemistry 1989, 10, 1
Organic Reactions 1993, 44, 297
- for synthetic purposes, cyclic α,β-unsaturated carbonyl are the most
useful.
intersystem
crossing
1
E*
E
3
E*
- symmetry requirements
2πs + 2πs
*
hν
O
O
~ 340 nm
HOMO
LUMO
- enones with olefins
O
O
+
CH2
O
O
O
hν
+
CH2
JCSCC
1966, 423
+
(90%)
O
O
O
O
hν
H
E.J. Corey
JACS 1963,
85 , 362
+
+
H
Caryophellene
Base
Hot Ground State?
O
O
O
H2C=CH2
hν
2πs + 2πa
thermal cyclization
CH3
N
H
H2C=CH2
hν, CH2Cl2
CH3
O
N
CH3
isomerization
cis
ring-fusion
H
CH3
O
Ph
H
H
CH3
O
JACS 1986,
108 , 306
HO
CH3
grandisol
129
FOUR-MEMBERED RING FORMATION
O
O
O
130
H
H
+
JACS 1984
106 , 4038
hν
H
H
2:1
- enones with acetylenes
O
O
hν
O
O
O
JACS 1982
104, 5070
- DeMayo Reaction
O
O
O
hν, pyrex
O
O
CO2Me
H
pTSA, MeOH
reflux
JACS 1986,
108 , 6425
O
O
H
O
R
hν
O
R
O
R
O
O
MeO2C
H
O
O
70-80 % de
favored
O
O
O
controls
conformation O
O
hν
H 3C
O
O
CH3
O
H
CH3
O
disfavored
TL 1993, 34, 1425
H 3C
H 3C
H
- Photochemistry of Ketones (Norrish Type I and II reactions)
H
H
hν
Norrish I
cleavage
O
•
•
254, 307 nm
Norrish II
cleavage
HO
Yang
reaction
OH
•
•
H
H H
O
O•
•
H
H 3C
O
O
H
O
CH3
CH3
controls
addition
O
H
O
O
H 3C
H
OH
+
FOUR-MEMBERED RING FORMATION
- filtering photchemical reaction to prevent Norrish reactions
quartz
180 nm
Vycor
200 nm
Pyrex
280 nm
Uranium glass
320 nm
- Yang Reaction
MOMO
MOMO
O
hν (254 nm)
OH
C6H6
SEMO
CH3
JACS 1987,
109 , 3017
CH3
SEMO
- thermal cycloadditons (2πa + 2πs)
- symmetry requirements
2πa + 2πs
O
HOMO
LUMO
- ketenes
O
∆
H
O
O
H
O
Cl
Et3N
O
H
O
Cl
Zn
O
Cl
O
N2
R
R
hν
O
H
- thermal cyclization of ketene with olefins
Tetrahedron 1986, 42, 2587; 1981, 37, 2949; Organic Reactions 1994, 45, 159.
LUMO
of ketene
2πa
pz
py
O
HOMO
of olefin
2πs
131
FOUR-MEMBERED RING FORMATION
Cl
O
CH3
Ph
O
Cl
Cl
CH2N2
CH3
O
O
O
Cl
CCl3
CH3
Zn-Cu, Et2O
Cl
Ph
H 3C
H 3C
N+
2) H2O
H 3C
Ar
JACS 1987,
109 , 4753
H
O
1)
CH3
R*O
CH3
H 3C
132
N+
JACS 1982, 104, 2920
H
OMe
OMe
-reaction of ketene with enamines
H
NR2
O
O
H
NR2
O
O
NR2
R'
R'
- reaction of ketene with imines to give β-lactams (Staudinger Reaction)
R
H
N
O
H
NR
O
O
O
R
N
+
H
CH2Cl2
-78°C → 0°C
N
Ph O
Bn
N
O
N
Ph
N
Ph
H
O
O
R
O
H
O
Bn
R
NBn
O
(90-96 % de)
- reaction of difluorodihaloethylene with olefins
Organic Reactions 1962, 12, 1
F
F2C=CX2
R
F
X= F, Cl
X
X
R
- reaction of difluorodihaloethylene with acetylenes- biradical mechanism
F
F2C=CX2
R
F
X= F, Cl
R
X
hydrolysis
O
X
R
O
FOUR-MEMBERED RING FORMATION
133
- SN2 Reaction
O
O
KH, DMSO
JACS 1980,
102 , 1404
OTs
O
_
CN
CN
HO
OR
JACS 1974, 96,
5268, 5272
OH
OR
Grandisol
- acyloin reaction
Organic Reactions 1976, 23, 259
CO2Me
OTMS
Na, TMS-Cl
(CH2)n
O
F-
(CH2)n
CO2Me
(CH2)n
OTMS
OH
R
CO2Me
R
O
R
CO2Me
R
OH
- benzocyclobutanes
ACIEE 1984, 23, 539; Synthesis 1978, 793
O
O
O
O
O
benzocyclobutane
O
o-quinodimethane
- cyclotrimerization of 1,5-diyenes with an acetylenes
SiMe3
Me3Si
SiMe3
CpCo(CO)2
Me3Si
- sulfur ylides
+
O
R
_
SPh2
R'
O
BF3
O
R
R'
R'
R
Oxatanes
Organic Photochemistry 1981, 5, 1
- [2+2] cycloaddition (Paterno-Buchi Reaction)
O
R
R
R'
O
R
O
hν
R'
hν (254 nm)
R'
O
R
R'
FOUR-MEMBERED RING FORMATION
O
O
hν
O
Me2C CMe2
O
O
134
TL 1975
1001
O
- SN2 reaction
O
NBS
OH
JCSCC 1979, 421
SiMe3
SiMe3
Br
OH
CO2Me
Tf2O, CH2Cl2
CO2Me
O
C5H11
HO
TL 1980,21 , 445
C5H11
OTBS
OH
OH
Br
Br
CO2Me
HO
(MeO)3P, DEAD
O
CO2Me
O
C5H11
C5H11
O
OH
JACS 1984,
107 , 6372
OH
- sulfur ylides
O
O
-
O
O
NTs
H2C_ S Me
S
NTs
O
JACS 1979,
101 , 6135
CH3
β-Lactones
R2
R1
LDA, THF
SPh
H
BnO
N
H
O
R3
CO2H
R1
R2 R1
O
BnO
JOC 1991,
56 , 1176
SPh
R4
DEAD, Ph3P
O
O
R4
O
R3
OH
O
O-
O
R2CuLi
O
N
H
R2
R3
R4
CO2H
R
BnO
NH
JACS 1987,
109 , 4649
O
O
1) MgBr2•OEt2
2) KF, H2O, CH3CN
O
OBn O
OBn
O
O
+
H
SiMe3
H
96 % de
TL 1995, 36, 4159
BALDWIN'S RULES FOR RING CLOSURE
Baldwin's Rules (Suggestions) for Ring Closure
JOC 1977, 42 , 3846
JCSCC 1976, 734, 736, 738
135
Approach Vector Analysis
- for an SN2 displacement at a tetrahedral center, the approach vector of the
entering nucleophile is 180° from the departing leaving group
Nu:
Nu
L
Nu
L
:L
- for the addition of a nucleophile to an Sp2 center, the nucleophile approaches
perpendicular to the π-system.
Nu
Nu
Nomenclature
1. indicate ring size being formed
3 membered ring = 3
4 membered ring = 4
etc.
2. indicate geometry of electrophilic atom
if Y= Sp3 center; then Tet (tetrahedral)
if Y= Sp2 center; then Trig (trigonal)
if Y= Sp center; then Dig (digonal)
Z
Y
X:
3. indicate where displaced electrons end up
- if the displaced electron pair ends up out side the ring being formed;
then Exo
- if the displaced electron pair ends up within the ring being formed;
then Endo
Z
Y
X:
Exo
Z
Y
X:
Endo
BALDWIN'S RULES FOR RING CLOSURE
136
4. Ring forming reaction is designated as Favored or Disfavored
disfavored does not imply the reaction can't or won't occur- it only means
the reaction is more difficult than favored reactions.
Rules (Suggestions) for Ring Closure
- All Exo-Tet reactions are favored
X:
Y
Z
X: Y
X:
Z
X:
Y
Z
3-
4-
X:
Y
Y
Z
5-
Z
6-
7-
- - - - - - - - - - - - - - - - - - - - - - - - --Favored- - - - - - - - - - - - - - - - - - - - - - - - -
- 5-Endo-Tet and 6-Endo-Tet are disfavored
Z
Z
Y
Y
X:
X:
5-
6-
- - - - -Disfavored- - - - -
- All Exo-Trig reactions are favored
X:
Y
Z
X: Y
Z
X:
X:
Y
Z
3-
4-
5-
X:
Y
Y
Z
Z
6-
7-
- - - - - - - - - - - - - - - - - - - - - - - - --Favored- - - - - - - - - - - - - - - - - - - - - - - - -
- 3-Endo-Trig, 4-Endo-Trig and 5-Endo-Trig are disfavored; 6-Endo-Trig, 7Endo-Trig, etc. are favored
Z
Z
X:
Y
3-
Z
X: Y
4-
Z
Y
X:
5-
- - - - - - - - - - - - -Disfavored- - - - - - - - - - - -
X:
Y
X:
Z
Y
76- - - - - -Favored- - - - - -
BALDWIN'S RULES FOR RING CLOSURE
137
- 3-Exo-Dig and 4-Exo-Dig are disfavored; 5-Exo-Dig, 6-Exo-Dig, 7-Exo-Dig, etc.
are favored
X:
X:
Y
X:
Y
Z
Z
3-
4-
Y
X:
Z
Z
5-
- - - - - -Disfavored- - - - - -
X:
Y
Y
Z
6-
7-
- - - - - - - - - - - - -Favored- - - - - - - - - - - -
- All Endo-Dig are favored
Z
Z
X:
Z
Y
Y
X:
X:
3-
4-
Z
Y
5-
X:
6-
Y
X:
Z
Y
7-
- - - - - - - - - - - - - - - - - - - - - - - - - - --Favored- - - - - - - - - - - - - - - - - - - - - - - - - - - - -
EXCEPTION:
There are many !!!
(see March p 212-214)
FIVE-MEMBERED RING FORMATION
5 Membered Rings
HO
O
CO2H
HO
CO2H
HO
OH
OH
PGF2α
PGE2
Me H
Me
Me
Me
Me
Me Me
Me
Hirsutene
1.
2.
3.
4.
5.
6.
7.
8.
9.
Isocumene
Modhephane
Intramolecular SN2 Reactions
Intramolecular Aldol Condensation and Michael Addition
Intramolecular Wittig Olefination
Ring Expansion and Contraction Reactions
a. 3 → 5
b. 4 → 5
c. 6 → 5
1,3-Dipolar additions
Nazarov Cyclization
Arene-Olefin Photocyclization
Radical Cyclizations
Others
Synthesis 1973, 397; ACIEE 1982, 21 , 480;
Intramolecular SN2 Reaction
5-exo-tet: favored
O
LDA
O
JCSCC 1973, 233
Br
O
O
CO2Me
O
CO2Me
Br
CN
CN
OH
JACS 1974, 96 , 5268
O
138
FIVE-MEMBERED RING FORMATION
O
Cl
JACS 1979,
101 , 5081
O
RO2C
RO2C
Cl
CO2R
CO2R
O
O
NaH, DMSO
JCSCC 1979, 817
TsO
Intramolecular Aldol Condensation 5-exo-trig: favored
intramolecular aldol condensation of 1,4-diketones
O
O
R
R
O
R'
R'
O
O
CH3
CH3
O
TL 1982,
29, 2237
CH3
CH3
Br
Br
O
NaOMe, MeOH
O
O
CO2Me
JACS 1980,
102 , 4262
CO2Me
O
O
JOC 1983,
48 , 1217
NaOH
O
O
O
Intramolecular Michael Addition
5-exo-tet: favored
Organic Reactions 1995, 47, 315-552
O
O
MeO2C
O
O
JACS 1979,
101 , 7107
139
FIVE-MEMBERED RING FORMATION
Tetrahedron 1980, 36, 1717
Intramolecular Wittig Olefination
O
O
1) (MeO) 2P(O)CH2 -
O
P(OMe)2
O
2) Collins
O
C5H11
THPO
OTHP
C5H11
THPO
OTHP
CO2H
O
JOC 1981, 46 , 1954
base
C5H11
THPO
OTHP
C5H11
HO
OH
Ring Expansion Reactions
- 3 → 5: Vinyl Cyclopropane Rearrangement
_
O
SPh2
O
Organic Reactions 1985, 33, 247.
O
H
OTMS
O
O
O
H
O
H
O
TMSO
JACS 1979, 101 , 1328
O
H
O
- 4 → 5: Reaction of cyclobutanones with Diazomethane
Cl
Me
CH2N2
Me
Me
Cl
O
O
O
Me
Cl
Me
Me
CH2N2
TL 1980,
21 , 3059
O
Cl
Cl
Cl
Cl
O
O
Cl
Cl
CH3
O
CH3
CH2N2
Cl
CH3
O
CH3
JACS 1987,109 , 4752
Ph
Ph
140
FIVE-MEMBERED RING FORMATION
Ring Contraction Reactions
- 6 → 5: Favorskii Reaction
Organic Reactions 1960, 11 , 261
_
O
O
Cl
OH
-
O OH
CO2H
_
1,3-Dipolar Addition to Olefins
1,3-Dipolar Cycloaddition Chemistry , vol 1 & 2 (A.
Padwa ed.) (Wiley, NY 1984); ACIEE 1977, 16 , 10. Chem Rev. 1998, 98, 863.
1,3-Dipole (4π)
LUMO
+
_
b
c
a
4πs + 2πs
- trimethylenemethane (TMM)
Alkene (2π)
HOMO
ACIEE 1986, 25 , 1. Synlett 1992, 107.
high
temperature
•
•
TMM
CO2Me
MeO2C
∆, MeCN
N
MeO2C
H
•
•
•
-N2
N
CO2Me
CH3
H
•
JACS 1981,
103 , 2744
note: TMM usually reacts poorly w/ electron defficient olefins
_
Pd(0)
Me3Si
OAc
+
PdLn
O
O
Me3Si
H
OAc
O
O
TL 1986, 27 , 4137
Pd(0), 80°C
MeO2C
CO2Me
Ni(COD)2, 35 °C
CO2Me
ACIEE 1985, 24 , 316
TL 1983, 24 , 5847
CO2Me
- α,α'-dihaloketones
O
OFeLn
O
Fe(CO)9
Ar
ACR 1979, 12 , 61
+
Br
Br
Ar
141
FIVE-MEMBERED RING FORMATION
ACR 1979, 12 , 396; Organic Reactions, 1988, 36 , 1
- nitrones
O+N
O
R1 N
R2
R1
JACS 1964,
86 , 3756
O
O
N
N
Me
O
N
MeNHOH,
EtONa
H
O
Me
NH
N
S
Me
NH
JOC 1984,
49 , 5021
H
H
MeO2C
Bn
OH
Me2N
1) MeI
2) H2, Pd/C
H
H
MeO2C
R3
R2
R3
Nitrone
-
142
Bn
N
O-
JACS 1983,
104 , 6460
O
S
C 4H 9
C 4H 9
- nitrile oxides
O
O
+ O
N
N
N O
ArNCO
CO2Et
CO2Et
O
O
N
O
O
O
N
O
O
OTHP
+ _
N O
THPO
N
O
OH
OTHP
Bu3Sn
(80%)
OTHP
H2, Raney Ni, H2 O,
MeOH, B(OMe)3
(88%)
1) TBS-Cl, DMF,
imidazole
2) nBuLi, THF
OH
O
O
OH
O
HF•pyridine,
CH3 CN, pyridine
3) Swern
(48%)
TBSO
O
(88%)
O
1) PPTS, EtOH, ↑↓
2) (CH3 )2C(OMe)2 ,
PPTS
O
O
(89%)
OH
Ph
(99%)
N
1) Swern
2) NH2OH•HCl
AcONa, MeOH
OTHP
(60%)
OTHP
NaOCl, H2 O,
CH2 Cl2
CO2Et
1) DHP, PPTS
CH2 Cl2, ↑↓
2) LAH
CH3
H
Ph
JACS 1992,
104 , 4023
CO2Et
OH
Bu 2BOTf, iPr 2EtN,
CH2 Cl2, -78°C
OH
O
H2
Cassiol
TL 1996, 37, 9292
(73%)
O
TBSO
O
HO
OH
OH
FIVE-MEMBERED RING FORMATION
OTBS
OTBS
OTBS
LAH
N +
O _
TL 1993, 34, 3017
O N
OH
NH2
Arene -Olefin Photocyclization
Organic Photochemistry 1989, 10 , 357
- the photochemistry of benzene is dominated by the singlet state
1
*
hν
*
*
hν
OAc
OAc
*
Me2CuLi
O
O
JACS 1982,
104 , 5805
JCSCC, 1986 247
Me
1) FVP
2) H2, Pd/C
*
*
hν
+
isocume
+
*
Tetrahedron
1981,37 , 4445
*
*
*
hν
OMe
OMe
JACS 1981,
103 , 688
OMe
cedrane
AcO
hν
AcO
*
*
HO
H+
H
TL 1982, 23 , 3983
TL 1983, 24 , 5325
143
FIVE-MEMBERED RING FORMATION
144
Intramolecular Photochemical [2+2]
"Rule of Five"
O
O
hν
O
O
O
O
JOC 1975, 40 , 2702
JOC 1979, 44 , 1380
hν
O
O
Nazarov Cyclization
review: Synthesis 1983, 429
- cyclization of allyl vinyl or divinyl ketones
Organic Reaction 1994, 45, 1
O
O
H3PO4/HCO2H
O
O
H3PO4/HCO2H
- 1,4-hydroxy-acetylenes
HO
H2SO4, MeOH
OH
H
O
OH
JOC 1989,
54 , 3449
H
OH
- Silicon-Directed Nazarov
O
O
Tetreahedron
1986, 42 , 2821
FeCl3
SiMe3
Me
- Tin -directed Nazarov
Me
TL 1986, 27 , 5947
Radical Cyclization
B. Giese Radicals in Organic Synthesis: Formation of Carbon-Carbon Bonds
(Pergamon Press; NY) 1986; Bull. Soc. Chim. Fr. 1990, 127 , 675; Tetrahedron 1981, 37 ,
3073; Tetrahedron 1987, 43, 3541; Advances in Free Radical Chemistry 1990, 1, 121.
Organic Reactions 1996, 48, 301-856.
Radical Addition to multiple bonds:
1. Free radical addition is a two stage process involving an addition step followed by an
atom transfer step.
2. In general, the preferred regioselectivity of the addition is in a manor to give the
most stable radical (thermodynamic control)
FIVE-MEMBERED RING FORMATION
Advantages of free radical reactions:
1. non-polar, little or no solvent effect
2. highly reactive- good for hindered or strained sysntems
3. insensitive to acidic protons in the substrates (i.e. hydroxyl groups do not
necessarily need to be protected
Mechanism of radical chain reactions
1. initiation
2. propagation
3. termination (bad)
Formation of carbon centered radicals:
tin hydride reduction of
alkyl, vinyl and aryl halides,
alcohol derivatives:
xanthates, thionocarbonate, thiocarbonylimidazolides
organosselenium & boron compounds
carboxylic acid derivatives (Barton esters)
reduction of organomercurials
thermolysis of organolead compounds
thermolysis or photolysis of azoalkanes.
Radical Ring Closure
For irreversible ring closure reaction, the kinetic product will predominate.
Both the 5-exo-trig and 6-endo trip are favored reactions, with the 6 exo-trig mode
producing the most stable radical. However, the 5-exo-trig is about 50 time faster
•
•
k1,5
•
k1,6
thermodynamically
favored
kinetically
favored
•
•
+
•
100 : 0
•
•
+
•
•
•
100 : 0
•
3-exo-trig
vs
4-endo-trig
4-exo-trig
vs
5-endo-trig
5-exo-trig
vs
6-endo-trig
+
98:2
•
•
•
+
6-exo-trig
vs
7-endo-trig
85:15
•
•
•
+
100 : 0
7-exo-trig
vs
8-endo-trig
145
FIVE-MEMBERED RING FORMATION
•
146
•
and
radicals open up fast and are not synthetically useful;
often used as probes for radical reaction
Effects of substituent on the regiochemistry of the 5-hexenyl radical cyclization
•
•
•
+
48:1
•
•
•
+
>200:1
•
•
•
+
2:3
•
•
•
+
< 1:100
Stereochemistry of 5-hexenyl radical cyclization
1-, or 3-substitued 5-hexenyl radicals give cis disubstituted cyclopentanes
2-, or 4-substituted 5-hexenyl radicals give trans disubstitued cyclopentanes
2
Br
Bu3SnH
H
R
•
R
H
R
+
R
65 : 35
prefered transition state
R
•
3
R
R
Br
R
+
H
H
75 : 25
H
4
Br
R
H
R
1
Bu3SnH
Br
+
R
•
R
R
•
83 : 17
R
+
H
R
73 : 27
R
FIVE-MEMBERED RING FORMATION
Br
OH
nBuSnH,
AIBN
147
JACS 1983,
105 , 3720
OH CN
CN
Bu3SnH,
AIBN
CO2Me
JACS 1990,
112, 5601
Br
CO2Me
Br
OH
Si
O
H Si
O
Bu3SnH
THPO
H
H2O2
THPO
OH
THPO
multiple cyclizations: D. Curran Advances in Free Radical Chemistry 1990, 1, 121.
OTBS
O
OAc
1) NaBH4, CeCl3
2) Ac2O, Et3N
1) LDA, THF
2) TBSCl
O
70 °C
O
OTBS
PhSe
H2O2
O
PhSeCl, CH2Cl2
O
O
THPO
O
1) PPTS, EtOH
2) LAH
CO2H
I
CuSMe2 Li+
THPO
I
1) Me3Si
Li
(1.0 equiv)
I
3) (CF3SO2)2O
pyridine
4) Bu4NI, PhH
2) CsF
Me
H
•
Bu3SnH, AIBN
PhH, reflux
JACS 1985,
107 , 1148
H
H
(±)-hirsutene
O
O
CH3MgBr,
CuBr•SMe2
HO
O
O
1) I2
2) DBU
O
O
O
MgBr
CuBr•SMe2, THF
O
O
HO
O
1) LAH
2) CH3SO2Cl
3) NaI
CO2Me
1) CH3MgBr
(excess)
Li
4)
5) CrO3, H2SO4, H2O
Bu3SnH, AIBN
PhH, reflux
Br
2) Me3SiBr
H H
•
H 3C
H
(±)-capnellene
Tetrahedron Lett.
1985, 41, 3943
FIVE-MEMBERED RING FORMATION
O
O
Me
nBuSnH,
AIBN
O
148
O
JACS 1986,
108, 1106
Me
Me
Me
Br
OH
H
CHO
O
JACS 1988,
110 , 5064
O
SmI2, THF
O
O
H
radical trapping
OEt
I
OEt
OEt
O
O
nBuSnH,
AIBN
RO
O
tBuNC
JACS 1986,
108 , 6384
•
RO
CN
RO
can also be trapped with acrylate esters or acrylonitrile.
OEt
OEt
I
O
O
nBuSnH,
AIBN
O
Me3Si
OEt
C5H11
OEt
O
SiMe3
C5H11
•
RO
RO
RO
1) (S)-BINAL-H
2) HCl, H2O, THF
O
C5H11
RO
1) Wittig
2) deprotect
CHO
RO
CO2H
HO
OH
(+)-PGF2α
O
Paulson-Khand Reaction
Tetrahedron 1985, 41 , 5855; Organic Reactions 1991, 40 , 1.
O
O
R
R''
R'
R
R
R''
Organometallics
1982, 1 , 1560
+
Co2(CO)6
R'
R'
R''
O
JOC 1992,
57 , 5277
Co2(CO)8, NMO
O
O
O
O
OTMS
HO
C5H11
C5H11
RO
HO
Brook
rearrangement
O
OEt
Pd(OAc)2,
CH3 CN
O
140°
FIVE-MEMBERED RING FORMATION
Ph
Cp2TiCl2,
EtMgBr, CO
EtO2C CO2Et
CO2Et
Ph
Ring-Closing Metathesis
CO2Et
Tetrahedron 1998, 54, 4413, Acc. Chem. Res. 1995, 25, 446.
O
O
N
O
OH
Bu2BOTf,
CH2Cl 2, -78°C
P(C6H11)3
Ph
Cl
Ru
Cl
P(C6H11)3
O
O
N
O
(97%)
CHO
(82 %)
Ph
Ph
OH
JOC 1992,
57 , 5803
O
O
O
N
LiBH4,
THF, MeOH
OH
J. Org. Chem. 1996, 61, 4192
O
OH
(78%)
Ph
Diazoketones
Tetrahedron 1981, 37 , 2407; Organic Reactions 1979, 26 , 361
O
R1
O
N2
BF3
R1
TL 1975, 4225
R2
R2
FVP of Acetylenic Ketones
O
O
R1
R2
FVP
H
R3
R1
R2
O
R3
H ••
R1
R2
R3
TL 1986,
27 , 19
149
6-MEMBERED RING FORMATION
Six Membered Rings
1.
2.
3.
4.
5.
Diels-Alder Reaction
o-Quinodimethanes
Intramolecular ene reaction
Cation olefin cyclizations
Robinson annulation
Diels-Alder Reaction
ACIEE 1984, 23 , 876; ACIEE 1977, 16 , 10; Organic Reactions 1984, 32 , 1
W. Carruthers Cycloadditions Reactions in Organic Synthesis (Pergamon Press, Oxford) 1990
- reaction of a 1,3-diene with an olefin to give a cyclohexene.
- thermal symmetry allowed pericyclic reaction
- diene must reaction is an s-cis conformation
- highly stereocontrolled process- geometry of starting material is preserved in the
product
- possible control of 4 contiguous stereocenters in one step
B
B
A
A
X
B
A
Y
X
Y
Y
Y
X
+
X
Y
X
X
+
Y
B A
B
A
B A
- Alder Endo Rule: In order to maximize secondary orbital interactions, the endo TS is
favored in the D-A rxn.
Tetrahedron 1983, 39, 2095
X
X
B
Y
Y
B
A
A
A
X
Y
Y
X
B
B A
O
O
exo
O
O
minor
O
H
H
O
H
endo
O
H
O
major
O O O
O
Orientation Rules
X
X
+
Y
X
Y
+
Y
major
minor
150
6-MEMBERED RING FORMATION
X
Y
+
X
+
X
151
Y
Y
major
minor
- when both the diene and dienophile are "unactivated" the D-A rxn is sluggish
- D-A rxns with electron rich dienes and electron defficient dienophiles work the best.
Some electron deficient dienophiles are quinones, maleic ahydride, nitroalkenes, α,βunsaturated ketones, esters and nitriles.
- D-A rxns with electron deficient dienes and electron rich dienophiles also work well.
These are refered to as reverse demand D-A rxns.
- D-A rxns are sensitive to steric effects of the dienephiles, particularly at the 1- and 2postions. Steric bulk at the 1-position may prevent approach of the dienophile while steric
bulk at the 2-position may prevent the diene from adopting the s-cis conformation.
- The D-A rxn is promoted by Lewis acids (TiCl4, BF3 AlCl3, AlEt2Cl, SnCl4,...)
- The D-A rxn is promoted by high pressure (1 kbar ~ 14200 psi)
Synthesis 1985, 1.
O
OMe
OMe
5 Kbar
Me
Me
O
+
JACS 1986,
108 , 3040
O
O
H
NHBOC
20 kbars, 50°C
H
+
Eu(fod)3
O
OMe
O
NHBOC
H
+
O
OMe
NHBOC
Synthesis
1986, 928
OMe
- The D-A rxn is usually insenstive to solvent effects, except for water. ACR 1991, 24 , 159
O
isooctane
+
O
+
krel=1
O
endo
H
exo
4:1
H2O
JACS 1980, 102 , 7816
TL 1983, 24 , 1901
TL 1984 , 25 , 1239
20 : 1
krel= 700
CO2- Na+
CHO
OHC
CO2H
CO2H
H
H
+
OHC
H2O
(70%)
(15%)
JOC 1984, 49 , 5257
JOC 1985, 50 , 1309
Tetrahedron 1986, 42 , 2847
6-MEMBERED RING FORMATION
152
- The mechanism of the D-A rxn is believed to be a one-step, concerted, non-synchronous
process.
- concerted- bond making and bond breaking processes take place in a single kinetic step
(no dip in the transition state)
- synchronous- bond making and bond breaking take place at the same time and to the
same extent.
M.J.S. Dewar JACS
1984, 104 , 203, 209.
+
- study of secondary D-isotope effects have indicated a highly symmetrical T.S.
D
H
D
H
k1
D
D
H
H
+
D
JACS 1972,
94 , 1168
D
D
D
k2
D
D
D
D
+
Diels Alder Reactions:
O
O
PhH, reflux
O
O
H
MeO2C
H
CHO
MeO2C
CO2H
OAc
OMe
HO2C
MeO
Tetrahedron
1958, 2 , 1
N
N
H
H
O2CAr
MeO2C
reserpine
H
CHO
H
+
NHCO2Bn
OMe
H
O
O
trans
trans
H
CHO
BnO2CHN
BnO2CHN
NHCO2Bn
JACS 1978,
100 , 5179
N
H
pumiliotoxin C
CH3
H
6-MEMBERED RING FORMATION
OMe
MeO
O
R
+
O
O
R
MeO2C
CO2Me
O
MeO2C
R
O
MeO2C
PhS
SPh
PhS
SPh
MeO
OMe
O
O
JACS 1986,
108 , 5908
O
Compactin
OMe
OMe
R
R
R
+
Me3SiO
ACR 1981,
14 , 400
O
Me3Si-O
Danishefsky's
Diene
OMe
MeO2C
OH
O
CO2Me
H
O
+
H
Me3SiO
O
O
H
Vernolepin
O
Hetero Diels-Alder Reactions
- Heterodienophiles
OMe
CO2Bn
N
+
PhH, reflux
O
EtO2C
Me3SiO
N
CO2Bn
JACS 1982,
102 , 1428
CO2Et
O
CO2Me
Me3SiO
+
HO
N
N
H
TsN
Ts
TL 1987,
28 , 813
HO
PhH, 5°C
OH
CO2Me
CH(OMe)2
CH(OMe)2
O
+
CH3
N
O
CO2Bn
N
CH3
OAc
AcO
CO2Bn
AcO
OH
NAc
CH3
TL 1985
27 , 4727
153
6-MEMBERED RING FORMATION
Me
Me
Me
Me
O
+
S
Me
O
S
Me
NTs
Me
Me
Me
S
Ph
S
S
Ph
Ph
H
Ph
1) MgBr2
O
2) H3O+
OMe
Me3SiO
Tetrahedron,
1983, 39 , 1487
S
O
+
TL 1983,
24 , 987
Me
Me
NTs
NHTs
Me
O
H
OBn
JACS 1985,
107 , 1256
O
OBn
OMe
+
Ar
H
Yb(fod)3
Me
Me3SiO
OMe
OMe
O
Me
HF,
Pyridine
O
O
Ar
Me3SiO
Me
JACS 1985,
107 , 6647
Me
Ar
O
Me
Me
O
Me
C3F7
M
O
O
C3F7
M
O
3
3
M(fod)3
M(hfc)3
- Heterodienes
O
H
O
O
+
AcO
OAc
Me
O
PhS
OAc
+
OEt
O
O
H
O
ACIEE 1983,
22 , 887
+
MeO2C
MeO2C
H
EtO
endo:exo
10 : 1
O
H
endo: exo
1:2
Me
MeO2C
AcO
HO
H
O
OH
SPh
OAc
Me
OH
ACR 1986,
19 , 250
154
6-MEMBERED RING FORMATION
155
OR'
N
O
N
O
+
RO
O
O
RO
RO
RO
N
OR
OR
OR'
OR'
O
O
RO
N
OMe
RO
JACS 1987,
109 , 285
NHAc
OR
CO2R'
H
JOC 1985,
50 , 2719
∆
AcO
N
N
N
O
O
O
Intramolecular Diels-Alder Reactions
- Type I IDA rxns
(IDA)
Fused bicyclc
Bridged Bicyclic
- Generally, for E-dienes, the fused product is observed unless the connecting chain is
very long. For Z-dienes, either the fused or bicyclic products are possible.
- Type II IDA rxns: gives bridgehead olefin
395°C
JACS 1982,
104 , 5708, 5715
O
O
O
O
O
O
185°C
EtO2C
EtO2C
JACS 1987,
109 , 447
O
EtO2C
CO2Et
AcO
NHBz O
155°C
Ph
O
OH
ACIEE 1983,
24 , 419
O
OH
O
O
H
OH
BzO AcO
Taxol
O
6-MEMBERED RING FORMATION
156
- IDA reactions to give fused 6•5 (hydroindene) and 6•6 (hydronaphthalene) ring
systems are usually favorable reactions.
R'
R'
R
(CH2)n
R
n= 1 or 2
(CH2)n
- Intramolecular D-A rxns that give medium sized rings (7,8,9, 10) are much less
favorable.
- Intramolecular D-A rxn which form large rings are often favorable reactions with the
diene and olefin portions act as if they were seperate molecules
H
H
O
O
O
+
H
H
O
O
O
endo
+
JACS 1980,
102 , 1390
H
O
O
O
O
O
H
O
O
O
O
O
exo
6.2 : 6.8 : 1 (77% combined yield)
- Preference for endo or exo transition state depends on the substituition of the diene,
dieneophile and connecting chain.
- For intramolecular D-A rxns, geometric constraints can now reverse the normal
regiochemistry of the addition as compared to the intermolecular rxn.
+
CO2R'
CO2R'
R
R
CO2R'
CO2R'
- for intramolecular D-A reactions, we will use endo and exo to described the
disposition of the connecting chain
R
R'
R'
R
H
(CH2)n
R'
H
(CH2)n
R
(CH2)n
endo
R
R'
R'
R
H
(CH2)n
(CH2)n
exo
H
6-MEMBERED RING FORMATION
157
- Lewis acids can greatly effect the endo/exo ratio of IDA reactions especially when the
olefin portion is E. The effects for Z-olefins is much more subtle
MeO2C
MeO2C
MeO2C
H
H
+
JACS 1982,
104 , 2269
H
H
75 : 25
150°C
(75% combined yield)
100 : 0
(RO)2AlCl2, rt
CO2Me
(72% combined yield)
MeO2C
MeO2C
H
H
+
H
H
75 : 25
180°C
63 : 37
EtAlCl2, rt
(74% combined yield)
(60% combined yield)
- the effect of substituents on the connectinng chain can influence the stereochemical
course of the IDA reaction
HN
O
HN
EtAlCl2
O
JOC 1981,
46 , 1509
H
MeO2C
rt
MeO2C
H
R
O
R R
R
O
H
H
Intramolecular Diels-Alder Reactions:
CO2Me
MeO2C
130°C
TBSO
H
TBSO
JOC 1981,
46 , 1506
H
O
O
O
150°C
O
TL 1973,
4477
6-MEMBERED RING FORMATION
OTBS
OTBS
CF3CO2H
H
JACS 1981,
103 , 4948
H
H
H
O
TBSO
OTBS
CO2Me
H
CO2Me
JOC 1982,
47 , 180
EtAlCl2
H
Asymmetric Diels-Alder Reactions
- Chiral Auxillaries
Chem. Rev. 1992, 92 , 953; Tetrahedron 1987, 43 , 1969
OAc O
OAc
O
M
O
+
HO
OH
BF3•OEt2, -40°C
H
CO2H
HO
H
OH
OAc
OAc
(-)- Shikimic Acid
>98% d.e.
O
O
O
TiCl4, 0°C
+
JOC 1983, 48, 1137
JOC 1983, 43, 4441
ACIEE 1985, 24, 1
TL 1984,
25 , 2191
O
R*
O
O
(97:3)
X
O
∆ = 4.5 %ee
TiCl4= 78% ee
iBu2AlCl= 90% ee
CO2R*
O
O
CO2R*
O
O
Ph
O
O
O
Ph +
O
+
OR*
O
Ph
Ph
O
JOC 1989,
54 , 2209
OR*
18 : 82
w/ BF3•OEt2
O
Ar
O
O
Et2AlCl
+
OTMS
d.e > 95%
92 : 8
R*
CH2Cl2, -80°C
Chem Lett. 1989, 2149
O
up to 70% d.e.
O
Ph
H
O
Me
O
OR* +
Eu(hfc)3
OTMS
Ph
O
PhCHO
O
Me
8 : 92
OR*
O
Me
JACS 1986, 108, 7060
158
6-MEMBERED RING FORMATION
CH3
O
Ph
O
O
R*
SnCl4, -78C
N +
O
O
N
O
O S
S
O
R
O
R*
PhMgBr
R
97% d.e.
159
R*
N
H
O S
Ph
O
OH
N
R
R
JCSCC , 1985 , 1449
TL 1986, 27 , 1853
O
O
H
(iPrO)2TiCl2,
CH2Cl2, 0°C
O
H
O
SO2
N
Tetrahedron
1987, 43 , 1969
H
CO2R*
O
TL 1989,
30 , 6963
endo/exo= 86:4
98% de
X
EtAlCl2, -78°C
Oppolzer Auxillary
O
O
Evan's auxillaries
N
R
O
O
O
N
R
Me
O
Ph
+
O
O
CO2R*
Me2AlCl
N
O
CO2R*
endo (major)
endo
CO2R*
CO2R*+
exo
endo/exo
endo A/ endoB
k(rel)
O
O
+
O
O
Me2AlCl
N
O
2 equiv. Me2AlCl
60:1
20:1
100
_
Me2AlCl2
_
Me2ClAl
exo
1 equiv Me2AlCl
20:1
4:1
1
N
O
Me2AlCl
+ Al
O
O
N
O
O
H
N
Al
O
O
6-MEMBERED RING FORMATION
O
O
N
R
O
O
OH
R*
EtAlCl2, -100°C
Ph
O
O
O
R*
N
Ph
R*
H
Me2AlCl
H
+
TL 1984 , 25 , 4071
JACS 1988, 110 , 1238
Me
H
O
O
JACS 1984, 106, 4261
JACS 1988, 110 , 1238
(R)-(+)-α-terpineol
O
O
160
H
15 : 85
O
N
Me2AlCl
Ph
95 : 5
- Chiral Dienes
O
OMe
O
Ph
O
CHO
OHC
OMe
BF3•OEt2, -20°C
BF3•OEt2, -78°C
Ph
4:1
94:6
O
O
OH
OMe
O
π-stacking
arrangement
MeO
H
Ph
O
+
O
Ph
O
OMe
O
H
O
OH
O
O
H
approach of
dienophile
O
only product
- Chiral Catalysts Chem. Rev. 1992, 92 , 1007; Synthesis 1991, 1; OPPI 1994, 26, 129158
OH
CHO
CHO
+
EtAlCl2, -78°C
JCSCC
1979, 437
72% (ee)
OtBu
Me
+
Eu(hfc)3, -10°C
O
O
TMSO
Me
TL 1983,
24 , 3451
PhCHO
Ph
Me
6-MEMBERED RING FORMATION
161
Ph Ph
Ph O
O
O
OH
OH
Me O
Ph Ph
N
+
O
CL 1986, 1967
N
O
(iPrO)2TiCl2
O
O
O
O
Ph Ph
O
O
N
Ph O
OH
O
Me O
OH
H
O
Ph Ph
O
70% yield
95% ee
O
(iPrO)2TiCl2
O
N
O
(30 mol % catalyst)
H
Ph
O
O
OH
OH
+
OH
O
JACS 1986,
108 , 3510
Ph
BH3, -78°C
OAc
OH
O
OAc
(98%)
Ts
N
Et B
Br
O
CHO
O
N
H
CHO
+
-78°C
Br
JACS 1992,
114 , 8290
(> 99% ee)
H
N
O
Br
CHO
O
N B Bu
Ts
+
CHO
Br
CH2Cl2, -78 °C,
16 hrs
Br
Br
O
(81%, 99% ee)
OC
O
H
N
O
B O
N
Ts
OH
HO
approach of diene
CO2H
Gibberellic Acid
Br
Br
H
H
N
OTBS
H 3C
H
CHO
+
O
O
O
1) NaBH4
2) DDQ
OTBS
O O
N B H
Ts
CH2Cl2, PhCH3
-78 °C, 42 hrs
3) F
O
JACS 1994, 116, 3611
-
4) HCl
CHO
(83%, 97% ee) O
O
OH
HO
Cassiol
OH
6-MEMBERED RING FORMATION
162
Ketene Equivalents in the D-A reaction
- ketenes undergo thermal [2+2] cycloaddition with dienes to give vinyl cyclobutanones.
- 2-chloroacrylonitrile as a ketene equiv. for D-A rxns.
AcO
+
Cl
Cl
CN
AcO
ortho-Quinodimathanes
CN
KOH, tBuOH
70 °C
R
R
R
o-quinodimethane
benzocyclobutane
1)
Nature (London) 1994, 367, 630
ACIEE 1995, 34, 2079
Synthesis 1978, 793; Tetrahedron 1987, 43 , 2873
R
O
O
HO
O
OTMS
LiNH2, NH3, THF
MgBr
CuI, TMS-Cl
Me3Si
SiMe3
CpCo(CO)2
I
O
O
O
170 °C
Me3Si
Me3Si
H
H
H
Me3Si
Me3Si
O
1) CF3CO2H, CCl4, -30°C
2) Pb(O2CCF3)4
H
H
ACIEE 1984, 23 , 539
JACS 1977, 99 , 5483
JACS 1979, 101 , 215
JACS 1980, 102 , 241
H
HO
Estrone
O
O
HN
HN
JACS 1971,
93 , 3836
155 °C
H
O
Me
N
O
Me
N
155 °C
ACIEE 1971, 11 , 1031
O
O
Me
O
O
N
Me
N
155 °C
N
O
N
ACIEE 1971, 11 , 1031
N
OMe
Me
O
OMe
Me
N
ACIEE 1971, 11 , 1031
180 °C
N
H
Me3Si
Me3Si
NH
H
6-MEMBERED RING FORMATION
Photoenolization
Tetrahedron 1976, 22, 405
R
O
R
•
hν
H
R
O•
OH
R
MeO2C
OH
CO2Me
CO2Me
H
CO2Me
R
R
R
O
R
O
O
-H2O
R
OH O
R
CO2Me
O
CO2Me
O
R
Intramolecular Ene Reactions
R
ACIEE 1984, 23 , 876, Synthesis 1991, 1
H
SnCl4
CHO
JOC 1985,
50, 4144
OH
H
JACS 1991,
113 , 2071
Me2AlCl
BnO
H
OH
CHO
OBn
Binaphthol
Tetrahedron Lett. 1985, 26, 5535
CHO
OH
Me2Zn
90% ee)
Polyene Cyclization
+
Terpene Biosynthesis
terpenes
sesquiterpenes
diterpenes
steroids
C 10
C 15
C 20
C 30
+
geraniol
farnesol
geranylgeraniol
squalene
- isoprene- basic building block
OPP
isopentyl-PP
isoprene unit
OPP
OPP
OPP
geraniol-PP (C10)
Farnesyl-PP (C15)
geranylgeraniol-PP (C20)
squalene (C30)
163
6-MEMBERED RING FORMATION
164
O
HO2C
α-Cedrane
Camphor
Abietic Acid
Biosynthesis of camphor:
+
OPP
O
+
Biosynthesis of cedrane:
OPP
H
+
+
H
+
+
Stork-Eschenmoser Hypothesis- Olefin Geometry is preserved in the cyclization reaction, i.e.
trans olefin leads to a trans fused ring jucntion
A. Eschenmoser HCA 1955, 38, 1890; G. Stork JACS 1955, 77, 5068
Me
Me
+
Me
Me
R
H
Me
Me
Me
Me
R
H
H
H
Biosynthesis of Abietic acid:
OPP
OPP
+
OPP
H
+
H
H
H
+
H
+
H
H
H
H
H
HO2C
H
6-MEMBERED RING FORMATION
165
-Steroid Biosynthesis:
H+
H
squalene
cyclase
squalene
epoxidase
squalene
H
+
H
H
HO
+O
H
HO
H
H
Protosterol
squalene oxide
H
H
+
H
H
H
H
H
H
HO
H
HO
H
H
HO
H
Lanosterol
Cholesterol
- Polyene cyclization in synthesis ACR 1968, 1, 1; Bioorg. Chem. 1976, 5, 51; Asymmetric
Synthesis 1984, 3, 341-409; ACIEE 1976, 15, 9
SnCl4
CH3NO2, 0°C
H
E.E. van Tamelen
JACS 1972, 94 , 8229
(8%)
H
HO
O
H
δ- Amyrin
OH
CHO
SnCl4
PhH, rt
JACS 1974,
96 , 3333
H
(38%)
MeO
MeO
H
SnCl4
pentane
H
(27%)
O
O
HO
CO2Me
OPO(OEt)2
O
W.S. Johnson
JACS 1974, 96, 3979
H
H
CO2Me
1) Hg(O2CCF3)2
2) NaCl
O
ClHg
H
JACS 1980,
102 , 7612
6-MEMBERED RING FORMATION
Robinson Annulation
166
Synthesis 1976, 777; Tetrahedron 1976, 32, 3.
O
acid or base
(thermodynamic
conditions)
O
O
- unfavorable equillibium for the Michael addition under kinetic conditions
O
base
(kinetic conditions)
-
O
-O
O
- stabilizing the resulting enolate of the Michael Addition product can shift the
equilibrium as in the case of the vinyl silane shown below
O
Me3Si
Me3Si
-
O
-
O
O
O
base
(kinetic conditions)
OR
OR
OR
a) MeLi
Me3SiO
O
H
b)
Me3Si
O
O
JACS 1974,
96 , 6181
O
H
O
O
O
c) MeONa
- Methyl Vinyl Ketone equivalents
I
+
mCPBA
-
O
O
Me3Si
Me3Si
Me3Si
Aldol
O
O
O
H+
O
O
JACS 1974,
96 , 3862
I
+
O
O
O
-
O
O
O
O
6-MEMBERED RING FORMATION
Intramolecular Aldol Condensation of 1,5-Diketones
6-exo-trig; favored process
O
O
R'
R'
base
- H2O
R
R
O
- DeMayo reaction to 1,5-diketones
O
O
O
O
O
H
DIBAL-H
hν
O
O
H 3C
Intramolecular Alkylations (SN2 reaction)
Radical Cyclizations
Acyloin Reaction
Birch Reduction
Organic Reactions 1992, 42, 1.
Aromatic Substitution
(Carey & Sundberg, Chapter 11)
Intramolecular Wittig Reaction
Sigmatropic Rearrangements
O
167
7-MEMBERED RING FORMATION
168
Medium Sized Rings
7-Membered Rings
[4+2] cycloadditions
- [4+2] cycloadditions between dienes and allylcations leads to cycloheptadienes
review:
ACIEE 1984, 23 , 1; ACIEE 1973, 12 , 819
+
+
+
I
+
CF3CO2Ag
ACIEE 1973, 12, 819
ACIEE 1984, 23, 1
-78°C
Me
ZnCl2, -30°C
ACIEE 1982,
21, 442
+
+
H
H
F3CCO2
SiMe3
SiMe3
O
O
OTMS
+
JACS 1982,
104 , 1330
ZnCl2
Br
O
OMe
O O
+
O
O
O
+
O
O
O
O
O
JACS 1979, 101, 226
O
mCPBA
O
HO
- Noyori [4+2] cycloaddition of α,α'-dibromoketones and dienes
review: ACR 1979, 12 , 61
O
R
R
Br
Br
Fe2(CO)9
-orZn-Cu
O
OM
R
R
R
+
R
ACR 1979, 61
Organic Reactions
1983, 29, 163
7-MEMBERED RING FORMATION
O
O
Br
Fe2(CO)9
Br
Br
O
Br
Zn
Br
O
O
Br
169
O
CO2Me
O
N
Br
Br
Br
MeO2C
1)
N
JACS 1978, 100 , 1786
Tetrahedron 1985, 41, 5879
Fe2(CO)9
2) Zn
Br
O
O
OMe
O O
O
+
O
O
+
O
O
O
O
O
O
mCPBA
JACS 1979, 101, 226
O
HO
O
O
Me
Me
Br
Br
1) H2, Pd/C
2) CF3CO3H
O
1)
Fe2(CO)9
OH
O
O
O
H
O
JACS 1972, 94, 3940
JOC 1976, 41, 2075
H
O
O
O
1) CF3CO3H
2) LDA, MeI
O
O
Me
O
Me
CO2H
Me
OH
JCSCC 1985, 55
O
Me OBn
OBn
OH
OH
OBn
O
- [4+2] cycloaddition between pentadienyl cations and olefins
+
+
O
HO
OH
∆
O
-
O
+
O
+
O
O
Tetrahedron 1966, 22, 2387
JOC 1987, 52, 759
7-MEMBERED RING FORMATION
O
MeO OMe
OMe
SnCl4
OMe
O
O
JACS 1977, 99, 8073
JACS 1979, 101, 6767
JACS 1981, 103, 2718
Seven-Membered Rings from Funnctionalization of Tropone
Organic Reactions 1997, 49, 331-425
O-
O
O
Cl
R
R
JOC 1988, 53, 4596
JACS 1987, 109, 3147
O
- [6+4] cycloadditions of tropones with dienes
[6+4]
O
JACS 1986, 108, 4655
JOC 1986, 51, 2400
150°C
O
(88%)
O
HO HO
HO
OH
Ingenol
- [4+2] cycloaddition between tropone and olefins
O
O
[4+2]
150°C
(81%)
Radical Ring Expansion Reactions
- one carbon ring expansions
O
O
CO2Me
(CH2)n
n = 1,2,3
NaH, CH2Br2
Br
CO2Me
nBu3SnH
CO2Me
(CH2)n
(CH2)n
•
CO2Me
(CH2)n
(CH2)n
O
•O
O
nBu3SnH,
AIBN
•
CO2Me
O
JACS 1987,109, 3493
JACS 1987,109 , 6548
(CH2)n
CO2Me
170
7-MEMBERED RING FORMATION
O
O
Bu3SnLi
BrCH2SePh
CH3
171
O
•
SePh
SnBu3
SnBu3
Tetrahedron 1989, 45, 909
Tetrahedron 1991, 47, 6795
O
•O
+
Bu3Sn•
SnBu3
- more than one carbon expansion
O
R
O
X
(CH2)n
(CH2)n
SnBu3
Tetrahedron 1989, 45, 909
Tetrahedron 1991, 47, 6795
R
X= I, SePh
n= 1,2
Br
Bu3SnH, AIBN,
C6H6, reflux
Br
+
JOC 1992, 57, 7163
O
O
Eight-Membered Rings
[4+4] Cycloaddition of Dienes
O
review:Tetrahedron 1992, 48 , 5757.
Ni(COD)2, Ph3P
MeO2C
MeO2C
JACS 1986,
108 , 4678
MeO2C
MeO2C
60°C
O
O
O
O
O
O
Ni(COD)2, Ph3P
H
60°C
H
O
Asteriscanolide
Carbonyl Coupling Reactions
- Acyloin Reaction
CO2Me
O
Na
CO2Me
JACS 1971,
93 , 1673
OH
- McMurry Reaction
R
R
CO2Et
Ti (0)
O
O
JACS 1988,
110 , 5904
7-MEMBERED RING FORMATION
172
CHO OHC
TiCl3, Zn-Cu
H
JACS 1986,
108 , 3513
H
Aldol-like Condensations
O
O
TiCl4
O
OTMS
OH
JCSCC 1983, 703
O
SiMe3
SiMe3
(CH2)n
R
Lewis Acid
R
O
O
(CH2)n
OMe
O
JACS 1986,
108 , 3516
n= 1,2
Me3Si
Cl
SnCl4
OTs
O
tBuPh2SiO
Cl
Cl
Me3Si
OTs
O
O
tBuPh2SiO
OEt
Laurenyne
TiCl4
O
SiMe3
JACS 1988,
110 , 2248
JOC 1988,
53 , 50
O
Pinocol Rearrangement
O
Me3SiO
SnCl4
Me3SiO
+
OMe
JACS 1989,
111 , 1514
OMe
OMe
H
OMe
Tiffeneu-Demyanov Ring Expansion
- one carbon ring expansion for virtually any size ring
O
HO
1) Me3SiCN
2) LAH
N2
NH2
HONO
O H
O
JOC 1980,
45 , 185
- also see Beckman and Schmidt rearrangements as a one atom ring expansion
for the conversion of cyclic ketones to lactams.
7-MEMBERED RING FORMATION
173
DeMayo Reaction
O
O
O
O
hν
O
O
+
OAc
AcO
O
JCSCC 1984, 1695
O
O
O
O
O
O
hν, pyrex
O
O
CO2Me
H
pTSA, MeOH
reflux
JACS 1986,
108 , 6425
O
O
H
Ring Expansion/Contraction via Sigmatropic Rearrangements
- Cope Rearrangement
O
O
SMe
TL 1991,
32 , 6969
55°C
O
O
O
O
- Anion Accelerated Cope
KH, 18-C-6
115°C
O
JACS 1989,
111, 8284
OH
O
OCOCH2OH
Pleuromutilin
- Claisen Rearrangement
O
O
O
Tebbe
reagent
O
185°C
H
TL 1990,
31 , 6799
7-MEMBERED RING FORMATION
- Ester Enolate Claisen- 4 carbon ring contractions
O
1) LDA, TBS-Cl
2) 110°C
O
CO2H
JACS 1982, 104 , 4030
Tetrahedron 1986, 42 , 2831
MOMO
MOMO
H
O
O
1) LDA, TBS-Cl
2) 110°C
JOC 1988, 53 , 4141
H
H
CO2TBS
174
Exam 1
page 1 of 5
1.
Give the reagent(s) necessary to carry out the following transformations. The stereochemistry of
the products and reactants is as shown. (20 pts)
OH
HO
HO
CO2CH3
OH
OH
O
OH
OH
O
OH
O
N
CO2H
O
O
O
O
O
OH
O
O
OH
OH
page 2 of 5
2.
Give the product of the following reactions. The stereochemistry of the reactant is as shown.
Give the proper sterochemistry of the major steroisomer of the product. (20 pts)
OsO4, MNO
O
O
O
(CH3)2CuLi
tBuMe2SiO
O
mCPBA
O
O
1) LDA, THF
Ph
N
O
2)
H 3C
Ph
O
Ph
SO2Ph
N
O
Ph
H
O
OCH3
CH3MgBr
page 3 of 5
3. What are the following reagents used for. Please be specific. (8 pts each)
+
Ir
P(C6H11)3
PF6 -
N
PPh2
PPh2
H
H
N
N
Bu2BOTf, iPr2NEt
Mn
O
O
tBu
4.
O
tBu
Give the reagent(s) needed to selectively deprotect the substrate below to the desired product.
(16 pts)
BnO
O
OSitBuMe2
O
BnO
AcO
O
O
OSitBuMe2
BnO
O
OSitBuMe2
OH
AcO
O
OH
OH
BnO
O
OH
O
OH
OSitBuMe2
O
AcO
O
O
O
AcO
O
page 4 of 5
5.
Provide the product and all intermediates for the following sequence of reactions. (20 pts)
1) LiAlH4
2) MnO2
3) tBuMe2SiCl, pyridine
O
O
4) pCH3C6H3SO2NHNH2
H+, C6H6 (- H2O)
5) NaCNBH3
page 5 of 5
6.
Starting from cyclohexanone, provide a feasible synthesis target shown. Give all reagents and
intermediates. (16 pts)
O
~ 4 steps
Exam 2
1.
page 1 of 5.
Give the product of the following reactions. The stereochemistry of the reactant is as shown.
Give the proper sterochemistry of the major stereoisomer of the product. (24 pts)
_
+
MeO2CNSO 2NEt3
Me
PhH, ∆
Ph
OH
O
Cp2 Ti
O
CH2
Al(CH3 )2
Cl
NNHTs
a) BuLi (2 equiv)
b) CH3I
H
O
O
1) LDA, TMS-Cl
2) ∆
H
OTf
Pd(0), CO, MeOH
O
Cl
1) CH2 N2
2) hν, MeOH
2.
page 2 of 5.
Give the reagent(s) necessary to carry out the following transformations. The stereochemistry of
the products and reactants is as shown. (24 pts)
CHO
CO2 Me
O
CO2 Me
O
O
H
O
H
H
H
H
H
H
H
O
OH
O
O
Ph
O
O
H
H
page 3 of 5.
3.
Give the expected product from a thermal and photochemical [2+2] cycloaddition of
cycloheptenone with ethylene. Using MO's, clearly show why the photochemical and thermal
pathways give different stereochemical outcomes. (12 pts)
O
+
4.
page 4 of 5.
Provide the product and all intermediates for the following sequence of reactions. (20 pts)
1) cyclopentene, hν
O
2) DIBAL
O
4a) (CH3)2CuLi
3) tBuO-K+
O
b) Tf2NPh
5) Ph2CuLi
5.
page 5 of 5.
Complete the synthesis for the target shown. Give all reagents and intermediates. (20 pts).
OH
~5 steps
OH
O
O
Exam 3
1.
page 1 of 6.
Give the product of the following reactions. The stereochemistry of the reactant is as shown.
Give the proper sterochemistry of the major stereoisomer of the product. (20 pts)
CpCo(CO)2
R
R
CHO
NHBn
I2
CO2 H
H
H2 , Pd/C
O
N
H
CH3
O
O
N
Ph
O
a) Bu2BOTf, Et3N
b) PhCHO
2.
page 2 of 6.
Give the reagent(s) necessary to carry out the following transformations. The stereochemistry of
the products and reactants is as shown. (20 pts)
Ph
Ph
CHO
OH
CH3
O
OBn
OBn
OBn
OBn
OH
O
CO 2H
O
OMe
OMe
OMe
O
Me
O
O
OH
OH
OEt
O
OEt
I
O
CN
page 3 of 6.
3.
Short Answers. (5 points each, 20 points total)
a. Define enantioselective and enantiospecific and give examples of each.
b. What is the Stork-Eschenmoser hypothesis?
page 4 of 6.
c. What are Baldwin's Rules (be general, do not give each specific rule)? Be sure to explain what are
the basis of the rules and give one example each of a reaction that is favored and disfavored
according to Baldwin's rule.
d. Define umpolung and give an example.
page 5 of 6.
4. Provide the product and all intermediates for the following sequence of reactions. (20 pts)
OH
OH
1) (CH3)2 C(OCH3)2 , H+
2) MnO2
3) (EtO)2P(O)CH2 CO 2Me, base
4) AcOH, THF, H2 O
OH 5) tBuSiCl (1 equv), pyridine
6) TsCl, pyridine
7) Bu4NF
5.
page 6 of 6.
Complete the synthesis for the target shown. Give all reagents and intermediates. (20 pts).
O
~ 6 steps
O
OBn
O